Sunteți pe pagina 1din 214

University of the Philippines Diliman

MATHEMATICS 22
Elementary Analysis II
Course Module

Institute of Mathematics
MATHEMATICS 22
Elementary Analysis II
Course Module

Institute of Mathematics
University of the Philippines Diliman
iv

©2018 by the Institute of Mathematics, University of the Philippines Diliman.


All rights reserved.

No part of this document may be distributed in any way, shape, or form without prior written
permission from the Institute of Mathematics, University of the Philippines Diliman. Mathematics

22 Module Writers and Editors:

Rowena Alma Betty


Mark Lexter de Lara

s
Dennis Leyson

ic
Rolando Perez III

at
h em
at
M
of
e
ut
stit
In
UP
Contents

1 Techniques of Integration and Improper Integrals 1


1.1 Review of Formulas of Integration and Integration by Substitution . . . . . . . . . . 1
1.2 Integration by Parts . . . . . . . . . . . . . . . . . . . . . . . . . . . . . . . . . . . . 4
1.3 Trigonometric Integrals . . . . . . . . . . . . . . . . . . . . . . . . . . . . . . . . . . 8
1.3.1 Integrals of the form sinm x dx or cosm x dx . . . . . . . . . . . . . . . . .
R R
9
1.3.2 Integrals of the form sinm x cosn x dx . . . . . . . . . . . . . . . . . . . . . .
R
10

s
1.3.3 Integrals of the form tanm x dx or cotm x dx . . . . . . . . . . . . . . . .
R R
11

ic
at
1.3.4 Integrals of the form secn x dx or cscn x dx . . . . . . . . . . . . . . . . .
R R
12

em
1.3.5 Integrals of the form tanm x secn x dx or cotm x cscn x dx . . . . . . . . . .
R R
14
R h R R
1.3.6 Integrals of the form sin mx cos nx dx, sin mx sin nx dx, cos mx cos nx dx 15
at
1.4 Integration by Trigonometric Substitution . . . . . . . . . . . . . . . . . . . . . . . . 16
M


1.4.1 Integrand contains an expression of the form a2 − u2 . . . . . . . . . . . . . 16

of

1.4.2 Integrand contains an expression of the form u2 − a2 . . . . . . . . . . . . . 17



e

1.4.3 Integrand contains an expression of the form a2 + u2 . . . . . . . . . . . . . 19


ut

1.5 Integration of Rational Functions by Partial Fractions . . . . . . . . . . . . . . . . . 20


tit

1.6 Improper Integrals . . . . . . . . . . . . . . . . . . . . . . . . . . . . . . . . . . . . . 25


s
In

1.6.1 Integrals with Infinite Limits of Integration . . . . . . . . . . . . . . . . . . . 26


UP

1.6.2 Integrals with Infinite Discontinuity . . . . . . . . . . . . . . . . . . . . . . . 30


1.7 Chapter Exercises . . . . . . . . . . . . . . . . . . . . . . . . . . . . . . . . . . . . . 34

2 Parametric Curves 35
2.1 Parametric Equations of Plane Curves . . . . . . . . . . . . . . . . . . . . . . . . . . 35
2.2 Calculus of Parametric Curves . . . . . . . . . . . . . . . . . . . . . . . . . . . . . . 39
2.2.1 Tangent Line to a Parametric Curve . . . . . . . . . . . . . . . . . . . . . . . 39
2.2.2 Concavity of Parametric Curves . . . . . . . . . . . . . . . . . . . . . . . . . 41
2.2.3 Arc Length of a Parametric Curve . . . . . . . . . . . . . . . . . . . . . . . . 43
2.3 Chapter Exercises . . . . . . . . . . . . . . . . . . . . . . . . . . . . . . . . . . . . . 46

3 Polar Coordinate System 49


3.1 Basic Definitions . . . . . . . . . . . . . . . . . . . . . . . . . . . . . . . . . . . . . . 49
3.2 Conversion Formulas . . . . . . . . . . . . . . . . . . . . . . . . . . . . . . . . . . . . 51
3.3 Polar Curves . . . . . . . . . . . . . . . . . . . . . . . . . . . . . . . . . . . . . . . . 53

v
vi CONTENTS

3.3.1 Lines and Circles in Polar Form . . . . . . . . . . . . . . . . . . . . . . . . . . 53


3.3.2 Symmetry in the Polar Plane . . . . . . . . . . . . . . . . . . . . . . . . . . . 56
3.4 Special Polar Curves . . . . . . . . . . . . . . . . . . . . . . . . . . . . . . . . . . . . 58
3.4.1 Limaçons . . . . . . . . . . . . . . . . . . . . . . . . . . . . . . . . . . . . . . 58
3.4.2 Roses . . . . . . . . . . . . . . . . . . . . . . . . . . . . . . . . . . . . . . . . 61
3.4.3 Other Polar Curves . . . . . . . . . . . . . . . . . . . . . . . . . . . . . . . . 63
3.5 Calculus of Polar Curves . . . . . . . . . . . . . . . . . . . . . . . . . . . . . . . . . . 64
3.5.1 Tangent Line to a Polar Curve . . . . . . . . . . . . . . . . . . . . . . . . . . 64
3.5.2 Arc length of a Polar Curve . . . . . . . . . . . . . . . . . . . . . . . . . . . . 67
3.5.3 Area of a Polar Region . . . . . . . . . . . . . . . . . . . . . . . . . . . . . . . 69
3.6 Chapter Exercises . . . . . . . . . . . . . . . . . . . . . . . . . . . . . . . . . . . . . 76

4 Surfaces in R3 79
4.1 The Three-Dimensional Coordinate System . . . . . . . . . . . . . . . . . . . . . . . 79
4.2 Planes in R3 . . . . . . . . . . . . . . . . . . . . . . . . . . . . . . . . . . . . . . . . . 83

s
4.3 Spheres . . . . . . . . . . . . . . . . . . . . . . . . . . . . . . . . . . . . . . . . . . . 84

ic
at
4.4 Traces of Surfaces . . . . . . . . . . . . . . . . . . . . . . . . . . . . . . . . . . . . . 85

em
4.4.1 Review of Conic Sections . . . . . . . . . . . . . . . . . . . . . . . . . . . . . 85
4.4.2 Traces . . . . . . . . . . . . . . . . . . . . . . . . . . . . . . . . . . . . . . . . 87
h
at
4.5 Cylinders . . . . . . . . . . . . . . . . . . . . . . . . . . . . . . . . . . . . . . . . . . 89
M

4.6 Surfaces of Revolution . . . . . . . . . . . . . . . . . . . . . . . . . . . . . . . . . . . 91


of

4.7 Quadric Surfaces . . . . . . . . . . . . . . . . . . . . . . . . . . . . . . . . . . . . . . 95


e

4.8 Chapter Exercises . . . . . . . . . . . . . . . . . . . . . . . . . . . . . . . . . . . . . 102


ut
tit

5 Vectors, Lines, and Planes 105


s
In

5.1 Vector Notation and Geometric Representation . . . . . . . . . . . . . . . . . . . . . 105


5.2 Vector Operations . . . . . . . . . . . . . . . . . . . . . . . . . . . . . . . . . . . . . 109
UP

5.2.1 Scalar Multiplication and Vector Addition . . . . . . . . . . . . . . . . . . . . 109


5.2.2 Dot Product . . . . . . . . . . . . . . . . . . . . . . . . . . . . . . . . . . . . 112
5.2.3 Cross Product . . . . . . . . . . . . . . . . . . . . . . . . . . . . . . . . . . . 117
5.3 Equations of Lines in R3 . . . . . . . . . . . . . . . . . . . . . . . . . . . . . . . . . . 121
5.4 Equations of Planes in R3 . . . . . . . . . . . . . . . . . . . . . . . . . . . . . . . . . 124
5.5 Chapter Exercises . . . . . . . . . . . . . . . . . . . . . . . . . . . . . . . . . . . . . 130

6 Vector-Valued Functions 131


6.1 Introduction . . . . . . . . . . . . . . . . . . . . . . . . . . . . . . . . . . . . . . . . . 131
6.1.1 Parametric Curves in R3 . . . . . . . . . . . . . . . . . . . . . . . . . . . . . . 131
6.1.2 Vector-Valued Functions of One Parameter . . . . . . . . . . . . . . . . . . . 132
6.1.3 Graphs of Vector Functions . . . . . . . . . . . . . . . . . . . . . . . . . . . . 133
6.1.4 Operations on Vector Functions . . . . . . . . . . . . . . . . . . . . . . . . . . 135
6.2 Calculus of Vector Functions . . . . . . . . . . . . . . . . . . . . . . . . . . . . . . . 136
CONTENTS vii

6.2.1 Limits and Continuity of Vector Functions . . . . . . . . . . . . . . . . . . . . 137


6.2.2 Derivatives of Vector Functions . . . . . . . . . . . . . . . . . . . . . . . . . . 138
6.2.3 Integrals of Vector Functions . . . . . . . . . . . . . . . . . . . . . . . . . . . 140
6.3 The Moving Trihedral . . . . . . . . . . . . . . . . . . . . . . . . . . . . . . . . . . . 142
6.3.1 Unit Tangent Vector . . . . . . . . . . . . . . . . . . . . . . . . . . . . . . . . 142
6.3.2 Unit Normal Vector . . . . . . . . . . . . . . . . . . . . . . . . . . . . . . . . 144
6.3.3 Unit Binormal Vector . . . . . . . . . . . . . . . . . . . . . . . . . . . . . . . 145
6.3.4 Frenet Frame of Space Curves . . . . . . . . . . . . . . . . . . . . . . . . . . . 146
6.4 Arc length and Parametrization using Arc length . . . . . . . . . . . . . . . . . . . . 149
6.4.1 Arc length of Space Curves . . . . . . . . . . . . . . . . . . . . . . . . . . . . 149
6.4.2 Parametrizing Vector Functions Using Arclength . . . . . . . . . . . . . . . . 150
6.5 Curvature . . . . . . . . . . . . . . . . . . . . . . . . . . . . . . . . . . . . . . . . . . 152
6.6 Motion in Space . . . . . . . . . . . . . . . . . . . . . . . . . . . . . . . . . . . . . . 157
6.6.1 Basic Definitions . . . . . . . . . . . . . . . . . . . . . . . . . . . . . . . . . . 157
6.6.2 Tangential and Normal Components of Acceleration . . . . . . . . . . . . . . 159

s
6.7 Chapter Exercises . . . . . . . . . . . . . . . . . . . . . . . . . . . . . . . . . . . . . 161

ic
at
em
7 Sequences and Series 163
7.1 Sequences . . . . . . . . . . . . . . . . . . .
h . . . . . . . . . . . . . . . . . . . . . . . 163
at
7.2 Series of Constant Terms . . . . . . . . . . . . . . . . . . . . . . . . . . . . . . . . . 169
M

7.3 Convergence Tests for Series of Nonnegative Terms . . . . . . . . . . . . . . . . . . . 174


of

7.4 Alternating Series Test . . . . . . . . . . . . . . . . . . . . . . . . . . . . . . . . . . . 180


7.5 Tests for Absolute Convergence . . . . . . . . . . . . . . . . . . . . . . . . . . . . . . 182
e
ut

7.6 Power Series . . . . . . . . . . . . . . . . . . . . . . . . . . . . . . . . . . . . . . . . . 187


tit

7.7 Functions as Power Series . . . . . . . . . . . . . . . . . . . . . . . . . . . . . . . . . 191


s
In

7.8 Taylor and Maclaurin Series . . . . . . . . . . . . . . . . . . . . . . . . . . . . . . . . 195


7.9 Approximations Using Taylor polynomials . . . . . . . . . . . . . . . . . . . . . . . . 199
UP

7.10 Chapter Exercises . . . . . . . . . . . . . . . . . . . . . . . . . . . . . . . . . . . . . 204


Chapter 1

Techniques of Integration and


Improper Integrals

In the previous course, we have developed a collection of basic formulas for integration, most
of which followed directly from the corresponding differentiation formulas. Beyond these basic

ic s
formulas, we only have one technique−substitution, that we have used to rewrite certain integrals

at
in a simpler form. Unfortunately, integration is not as straightforward as differentiation and this

em
leaves us with many integrals that cannot be evaluated given our current knowledge. Hence we
h
at
must consider other techniques of integration.
M
of

1.1 Review of Formulas of Integration and Integration by Substi-


e

tution
ut
tit

Theorem 1.1.1. If f and g are differentiable functions of x and a is a real number, then:
s
In

1. Dx (a) = 0
UP

2. Dx ((f (x))n ) = n(f (x))n−1 Dx (f (x)), ∀n ∈ R

3. Dx (f (x) ± g(x)) = Dx (f (x)) ± Dx (g(x))

4. Product Rule: Dx (f (x)g(x)) = f (x)Dx (g(x)) + g(x)Dx (f (x))


 
f (x) g(x)Dx (f (x)) − f (x)Dx (g(x))
5. Quotient Rule: Dx =
g(x) (g(x))2
Theorem 1.1.2. The following differentiation formulas hold:

1 1
1. Dx (ln x) = 4. Dx (loga x) = , a > 0, a 6= 1
x (ln a)x
2. Dx (ex ) = ex 5. Dx (sin x) = cos x

3. Dx (ax ) = ax ln a, a > 0, a 6= 1 6. Dx (cos x) = − sin x

1
2 CHAPTER 1. TECHNIQUES OF INTEGRATION AND IMPROPER INTEGRALS

7. Dx (tan x) = sec2 x 18. Dx (cosh x) = sinh x

8. Dx (cot x) = − csc2 x 19. Dx (tanh x) = sech2 x

9. Dx (sec x) = sec x tan x 20. Dx (coth x) = − csch2 x

10. Dx (csc x) = − csc x cot x 21. Dx (sech x) = − sech x tanh x


1 22. Dx (csch x) = − csch x coth x
11. Dx (sin−1 x) = √
1 − x2
1
1 23. Dx (sinh−1 x) = √
12. Dx (cos−1 x) = − √ x2 +1
1 − x2
1
1 24. Dx (cosh−1 x) = √ ,x > 1
13. Dx (tan−1 x) = x2 −1
1 + x2
1
1 25. Dx (tanh−1 x) = , |x| < 1
14. Dx (cot−1 x) = − 1 − x2
1 + x2
1
1 26. Dx (coth−1 x) = , |x| > 1
1 − x2

s
15. Dx (sec−1 x) = √

ic
x x2 − 1
1

at
1 27. Dx (sech−1 x) = − √ , x ∈ (0, 1)

em
16. Dx (csc−1 x) = − √ x 1 − x2
x x2 − 1
1
h
28. Dx (csch−1 x) = − √ , x 6= 0
at
17. Dx (sinh x) = cosh x |x| 1 + x2
M

Theorem 1.1.3. The following antidifferentiation formulas hold:


of
e
ut

 R
un+1 11. sec u du = ln | sec u + tan u| + C
tit

 n + 1 + C, n 6= −1



R n
s

R
1. u du = 12. csc u du = ln | csc u − cot u| + C
In



 ln |u| + C, n = −1
 Z
1 u
UP

13. √ du = sin−1 + C, a > 0


a2 −u2 a
au
au du =
R
2. + C, a > 0, a 6= 1 Z
1 1 u
ln a 14. du = tan−1 + C, a 6= 0
R a2 + u2 a a
3. sin u du = − cos u + C Z
1 1 u
4.
R
cos u du = sin u + C 15. √ du = sec−1 + C, a > 0
2
u u −a2 a a
R
sec2 u du = tan u + C
R
5. 16. sinh u du = cosh u + C
R
csc2 u du = − cot u + C
R
6. 17. cosh u du = sinh u + C

sech2 u du = tanh u + C
R R
7. sec u tan u du = sec u + C 18.

csch2 u du = − coth u + C
R R
8. csc u cot u du = − csc u + C 19.
R R
9. cot u du = ln | sin u| + C 20. sech u tanh u du = − sech u + C
R R
10. tan u du = ln | sec u| + C 21. csch u coth u du = − csch u + C
1.1. REVIEW OF FORMULAS OF INTEGRATION AND INTEGRATION BY SUBSTITUTION3
R Z
22. tanh u du = ln cosh u + C 1 u
26. √ du = sinh−1 + C, a > 0
u2+a2 a
R
23. coth u du = ln | sinh u| + C
Z
1 u
27. √ du = cosh−1 + C, a ∈ (0, u)
u2 − a2 a
sech u du = 2 tan−1 eu + C
R
24. Z
1 1 u + a
28. du = ln + C,
R a2 − u2 2a u − a
25. csch u du = ln | csch u − coth u| + C u 6= a, a 6= 0

Z
5x etanh 5 sech2 5x dx.
x
Example 1.1.4. Evaluate

Solution. Let u = tanh5x . Then du = sech2 5x 5x ln 5 dx.


Z Z
x tanh 5x 2 x 1
5 e sech 5 dx = eu du
ln 5
1 u
= e +C

s
ln 5

ic
1 tanh 5x

at
= e + C.
ln 5

em
Z p
x3 x2 + 1 dx.
Example 1.1.5. Evaluate
h
at
M

Solution. Let u = x2 + 1. Then du = 2x dx.


of
e

Z Z
1
ut

p p
3
x x2 + 1 dx = x2 · 2x x2 + 1 dx
2
tit

Z
1 1
s

= (u − 1)u 2 du
In

2
Z
1  3 1

UP

= u 2 − u 2 du
2
" 5 3
#
1 u2 u2
= − +C
2 5/2 3/2
5 3
(x2 + 1) 2 (x2 + 1) 2
= − + C.
5 3
Z 3
1
Example 1.1.6. Evaluate dx.
3 4x2 − 12x + 18
2

Solution. We have

Z Z
1 1
2
dx = dx
4x − 12x + 18 4x2
− 12x + 9 + 9
Z
1
= dx.
(2x − 3)2 + 32
4 CHAPTER 1. TECHNIQUES OF INTEGRATION AND IMPROPER INTEGRALS

3
Let u = 2x − 3. Then du = 2 dx. Note that when x = , u = 0, and when x = 3, u = 3. Thus,
2
Z 3 Z 3
1 1 1
2 2
dx = du
3 (2x − 3) + 3 2 0 + 32 u2
2
1 1 −1 u
  3 π
= · tan = .
2 3 3 0 24

EXERCISES. Evaluate the following integrals.

1
Z
ln(5x)
Z
1
1. dx 7. √ dx
x 0 1+ x

Z Z p
2. 1 − 2x dx 8. sin 4x 1 + cos2 2x dx
π
0
Z
cos x
2
Z

s
3. dx 9. 2 cosh [ln(1 − 2x)] dx

ic
0 sin x + 1 −1

at

em
e x (5x6 + 10x4 − 6x2 − 4)
Z Z
4. √ dx 10. dx
x x3 + 2x
h
at
Z
x
Z
πx x
M

5. 2 π dx 11. dx
x2 + 2x + 2
of

Z
Z 2 ln x + 3
6. x|x − 1|dx 12. dx
e

p
x 5 − 4 ln x − ln2 x
ut

0
s tit
In

1.2 Integration by Parts


UP

Let f and g be differentiable functions of x. By the Product Rule of differentiation, we have

Dx [f (x)g(x)] = f (x)g 0 (x) + f 0 (x)g(x)


f (x)g 0 (x) = Dx [f (x)g(x)] − f 0 (x)g(x).

Applying integration on each side of the equation, we get


Z Z Z
f (x)g (x) dx = Dx [f (x)g(x)] dx − f 0 (x)g(x) dx
0

Z
= f (x)g(x) − f 0 (x)g(x) dx.

Letting u = f (x) and v = g(x), we can re-write the above equation as


Z Z
udv = uv − vdu.

Thus, we have the following formula:


1.2. INTEGRATION BY PARTS 5

Theorem 1.2.1. (Integration by Parts (IBP) Formula) Let u and v be differentiable functions.
Then
Z Z
udv = uv − vdu,
Z b b Z b
udv = uv − vdu.

a a a
Z
Example 1.2.2. Find xex dx.

Solution. To apply integration by parts, consider the possible choices:

1. Let u = x and dv = ex dx.

2. Let u = 1 and dv = xex dx.

3. Let u = ex and dv = xdx.

4. Let u = xex and dv = dx.

s
• Choice 1: Let u = x and dv = ex dx. Then

ic
at
du = dx, v = ex + K.

We have
h em
at
Z Z
xex dx = x(ex + K) − (ex + K)dx
M

| {z } | {z }
uv vdu
of

Z Z
x x
= xe + Kx − e dx − Kdx
e
ut

= xex + Kx − ex − Kx + C
tit

= xex − ex + C.
s
In

• Choice 2: This does not work because we do not know how to integrate dv = xex dx to get v.
UP

• Choice 3: Let
u = ex dv = xdx
2
du = ex dx v = x2 .
The new integral Z
x2 x 2 Z
x xx
xe dx = e e dx −
2 2
is more complicated than the integral we started with.

• Choice 4: Let
u = xex dv = dx
du = (ex + xex )dx v = x.
This choice leads to Z Z
x 2 x
xe dx = x e − (xex + x2 ex )dx,

which is another complicated integral.


6 CHAPTER 1. TECHNIQUES OF INTEGRATION AND IMPROPER INTEGRALS

Remark 1.2.3.

1. In general,
Z Z
uv − vdu = u(v + K) − (v + K)du, for any constant K.

Therefore, we can omit the constant of integration when calculating v in IBP.

2. There may be several choices for the function u and the differential dv. A wise choice for u is
one that is simpler when differentiated, and for dv is one that is readily integrated. In most
cases, the following mnemonic is applicable when choosing the function u:

L − logarithmic function
I − inverse trigonometric function
A − algebraic function

s
ic
T − trigonometric function

at
E − exponential function
h em
at
Z 1
M

2
Example 1.2.4. Find sin−1 x dx.
0
of
e

Solution. Let
ut

u = sin−1 x,
tit

dv = dx
1
s

du = √ dx, v = x.
In

1 − x2
UP

Z 1 1 Z 1
2
−1 −1 2 x2
sin x dx = x sin x − √
dx
0 0 0 1 − x2
1 p 1
= x sin−1 x + 1 − x2
2 2
√ 0 0
π 3
= + − 1.
12 2
Z
Example 1.2.5. Find x tan−1 xdx.

Solution. Let
u = tan−1 x, dv = xdx
1 x2
du = dx, v = .
1 + x2 2
1.2. INTEGRATION BY PARTS 7

x2 x2
Z Z
−1 −1
x tan xdx = tan x − dx
2 2(x2 + 1)
x2
Z 2
x +1−1
= tan−1 x − dx
2 2(x2 + 1)
x2
Z Z 
−1 1 1
= tan x − dx − dx
2 2 2(x2 + 1)
x2 tan−1 x
 
−1 1
= tan x − x− +C
2 2 2
1 2 1
= (x + 1) tan−1 x − x + C.
2 2

There are integration problems that require the use of the integration by parts formula several
times, as illustrated in the next examples.
Z
Example 1.2.6. Evaluate x2 e2x dx.

ic s
Solution. Let

at
u = x2 ,dv = e2x dx

em
e2x
du = 2x dx, v = h .
2
at
So
x2 2x
M

Z Z
2 2x
x e dx = e − xe2x dx.
2
of

Let
e

u = x, dv = e2x dx
ut

e2x
tit

du = dx, v = .
2
s
In

We now have
UP

x2 2x
Z Z
2 2x
x e dx = e − xe2x dx
2
x2 2x
 Z 2x 
x 2x e
= e − e − dx
2 2 2
x2 2x x 2x e2x
= e − e − + C.
2 2 4
Z
Example 1.2.7. Evaluate ex sin x dx.

Solution. Let
u = sin x, dv = ex dx
du = cos xdx, v = ex .
So
Z Z
ex sin x dx = ex sin x − ex cos x dx.
8 CHAPTER 1. TECHNIQUES OF INTEGRATION AND IMPROPER INTEGRALS

Let
u = cos x, dv = ex dx
du = − sin xdx, v = ex .
We now have,
Z Z
x x
e sin x dx = e sin x − ex cos x dx
 Z 
x x x
= e sin x − e cos x + e sin x dx
Z Z
e sin x dx = e sin x − e cos x − ex sin x dx.
x x x

Observe that the desired integral also appears on the right-hand side of the equation after the second
application of the integration by parts formula. By combining this to integral to the left-hand side
of the equation, we obtain
Z
1
ex sin x dx = [ex sin x − ex cos x] + C.
2

s
ic
EXERCISES. Evaluate the following integrals.

at
em
Z e Z e
1. ln x dx 9. x3 ln x dx
h
at
1 1
M

3x2 − 1
Z Z
2. cot−1 x dx 10. √ tan−1 x dx
2x x
of

Z
e

Z
2
3. ln x dx log(x2 + 1) dx
ut

11.
tit

Z 1 Z
s

2 −1 12. sin(ln x) dx
4. x sin x dx
In

1
2
UP

Z Z π
2
5. cosh −1
(x + 1)dx 13. ex cos x dx
0
Z π
Z
6. 3
x cos 2x dx 14. (e2x + ex ) ln(ex + 1) dx
0
x2x
Z

Z
7. 2
(x + 2x) 1 − 3x dx 15. dx
[(ln 2)x + 1]2
Z Z 1 √
x
8. x csc2 5x dx 16. e dx
0

1.3 Trigonometric Integrals


Evaluating an integral whose integrand contains powers of one or more trigonometric functions often
involves making an appropriate substitution. In this section, we look at techniques of integration
that work for powers of each of the six trigonometric functions, or product of such powers.
1.3. TRIGONOMETRIC INTEGRALS 9

sinm x dx or
R R
1.3.1 Integrals of the form cosm x dx

Table 1.1: Integrating Powers of Sines and Cosines

sinm x dx, cosm x dx


R R
Procedure Identities Used
· Split off a factor of sin x or cos x
m odd · Apply the identity. sin2 x = 1 − cos2 x,
· Make the substitution u = cos x or cos2 x = 1 − sin2 x
u = sin x.
m is even · Use identities to reduce the powers sin2 x = 12 (1 − cos 2x),
of sin x or cos x. cos2 x = 12 (1 + cos 2x)

s
Z
sin3 x dx.

ic
Example 1.3.1. Evaluate

at
Solution. Note that

sin3 x = sin2 x sin x


h em
at
= (1 − cos2 x) sin x
M

= sin x − cos2 x sin x.


of

Z Z
e

3
sin x − cos2 x sin x dx.

Thus, sin x dx =
ut
tit

Let u = cos x, du = − sin x dx. Therefore,


s

Z Z
1 1
In

3
1 − u2 du = −u + u3 + C = − cos x + cos3 x + C.

sin x dx = −
3 3
UP

Z
Example 1.3.2. Evaluate cos5 x dx.

Solution.
Z Z
cos5 x dx = cos4 x cos x dx
Z
2
= 1 − sin2 x cos x dx
Z
1 − 2 sin2 x + sin4 x cos x dx.

=

Let u = sin x, so du = cos x dx. Then,


Z Z
5
1 − 2u2 + u4 du

cos x dx =
2 1
= u − u3 + u5 + C
3 5
2 1
= sin x − sin3 x + sin5 x + C.
3 5
10 CHAPTER 1. TECHNIQUES OF INTEGRATION AND IMPROPER INTEGRALS

sinm x cosn x dx
R
1.3.2 Integrals of the form

Table 1.2: Integrating Products of Sines and Cosines

sinm x cosn x dx
R
Procedure Identities Used
· Split off a factor of sin x or cos x,
whichever has an odd exponent
m or n is odd · Apply the identity. sin2 x = 1 − cos2 x,
· Make the substitution u = cos x cos2 x = 1 − sin2 x
or u = sin x.
m and n are even · Use identities to reduce the powers sin2 x = 21 (1 − cos 2x)
of sin x and cos x. cos2 x = 21 (1 + cos 2x)

ic s
· Use the rule for cosm x dx.
R

at
h em
at
M

Z
Example 1.3.3. Evaluate cos3 x sin2 x dx.
of
e
ut

Solution. We have
s tit
In

Z Z
3 2
cos x sin x dx = cos2 x sin2 x cos x dx
UP

Z
1 − sin2 x sin2 x cos x dx

=
Z Z
= sin2 x cos x dx − sin4 x cos x dx.

Let u = sin x. Then du = cos xdx. We have

Z Z Z
3 2 2
cos x sin x dx = u du − u4 du
1 1
= u3 − u5 + C
3 5
1 1
= sin x − sin5 x + C.
3
3 5
1.3. TRIGONOMETRIC INTEGRALS 11
Z
Example 1.3.4. Find sin2 x cos4 x dx.

Solution.
Z Z
2 4
sin x cos x dx = sin2 x(cos2 x)2 dx

1 − cos 2x 1 + cos 2x 2
Z  
= dx
2 2
Z
1
= (1 + cos 2x − cos2 2x − cos3 2x) dx
8
Z    
1 1 + cos 4x 2
= 1 + cos 2x − − (1 − sin 2x) cos 2x dx
8 2
sin3 2x
    
1 sin 2x 1 sin 4x 1
= x+ − x+ − sin 2x − + C.
8 2 2 4 2 3

s
R R
1.3.3 Integrals of the form tanm x dx or cotm x dx

ic
at
em
Table 1.3: Integrating Powers of Tangents and Cotangents
h
at
M

Integral Procedure Identities Used


of

· Split off a factor of tan2 x or cot2 x.


e

tanm x dx or cotm x dx tan2 x = sec2 x − 1


R R
· Apply the identity.
ut
tit

· Make the substitution u = tan x cot2 x = csc2 x − 1


s

or u = cot x.
In
UP

Z
Example 1.3.5. Evaluate tan3 x dx.

Solution. We have
Z Z
2
tan x sec2 x − 1 dx

tan x tan x dx =
Z Z
2
= tan x sec x dx − tan x dx.

Let u = tan x. Then du = sec2 x dx.


Z Z
2
tan x tan x dx = u du − ln | sec x| + C
1
= u2 − ln | sec x| + C
2
1
tan2 x − ln | sec x| + C.

=
2
12 CHAPTER 1. TECHNIQUES OF INTEGRATION AND IMPROPER INTEGRALS
Z
Example 1.3.6. Evaluate cot4 3x dx.

Solution. We have
Z Z
2 2
cot2 3x csc2 3x − 1 dx

cot 3x cot 3x dx =
Z
cot2 3x csc2 3x − cot2 3x dx

=
Z
cot2 3x csc2 3x − csc2 3x + 1 dx

=
Z
1
cot2 3x csc2 3x dx + cot 3x + x + C.

=
3

Let u = cot 3x. Let du = −3 csc2 3x dx.

−1
Z Z
1
cot2 3x cot2 3x dx = u2 du + cot 3x + x + C
3 3

ics
−1 3 1
= u + cot 3x + x + C

at
9 3

em
−1 1
= cot3 3x + cot 3x + x + C.
9 3 h
at
M
of

R R
1.3.4 Integrals of the form secn x dx or cscn x dx
e
ut
tit

Table 1.4: Integrating Powers of Secants and Cosecants


s
In
UP

secn x dx or cscn x dx
R R
Procedure Identities Used
· Split off a factor of sec2 x or csc2 x.
n even · Apply the identity. sec2 x = 1 + tan2 x
· Make the substitution u = tan x csc2 x = 1 + cot2 x
or u = cot x.
· Split off a factor of sec2 x or csc2 x.
· Apply the integration by parts with
n odd u = secn−2 x and dv = sec2 x dx or tan2 x = sec2 x − 1

u = cscn−2 x and dv = csc2 x dx. cot2 x = csc2 −1


· Apply the identity.
1.3. TRIGONOMETRIC INTEGRALS 13
Z
Example 1.3.7. Evaluate csc6 x dx.

Solution. We have

Z Z
6
csc x dx = (csc2 x)2 csc2 x dx
Z
= (1 + cot2 x)2 csc2 x dx
Z
= (1 + 2 cot2 x + cot4 x) csc2 x dx.

Let u = cot x. Then du = − csc2 x dx.

2 cot3 x cot5 x
Z Z  
6 2 4
csc x dx = − (1 + 2u + u ) du = − cot x + + + C.
3 5
Z

s
Example 1.3.8. Find sec3 x dx.

ic
at
em
Z Z
3
Solution. Note that sec x dx = sec x sec2 x dx. Applying integration by parts, let
h
at
u = sec x and dv = sec2 x dx.
M
of

Then
e
ut

du = sec x tan x, and v = tan x dx.


s tit
In

We have
UP

Z Z
3
sec x dx = sec x tan x − tan x(sec x tan x) dx
Z
= sec x tan x − tan2 x sec x dx
Z
(sec2 x − 1) sec x dx
= sec x tan x −
Z Z Z
3 3
sec x dx = sec x tan x − sec x dx + sec x dx.

Z
Transposing sec3 x dx on the left-hand side, we have

Z
2 sec3 x dx = sec x tan x + ln | sec x + tan x| + C
Z
1
sec3 x dx = (sec x tan x + ln | sec x + tan x|) + C.
2
14 CHAPTER 1. TECHNIQUES OF INTEGRATION AND IMPROPER INTEGRALS
R R
1.3.5 Integrals of the form tanm x secn x dx or cotm x cscn x dx

Table 1.5: Integrating Products of Tangents and Secants or Cotangents and Cosecants

tanm x secn x dx or
R
Procedure Identities Used
cotm x cscn x dx
R

· Split off a factor of sec2 x or csc2 x.


n even · Apply the identity. sec2 x = 1 + tan2 x
· Make the substitution u = tan x csc2 x = 1 + cot2 x
or u = cot x.
· Split off a factor of sec x tan x
m odd or csc x cot x.
· Apply the identity. tan2 x = sec2 x − 1

ic s
· Make the substitution u = sec x cot2 x = csc2 x − 1

at
em
or u = csc x.
· Use identities to reduce the tan2 x = sec2 x − 1
m even, n odd
h
at
integrand to powers of sec x cot2 x = csc2 x − 1
M

or csc x only.
of
e
ut
tit

Z
Example 1.3.9. Evaluate tan3 x sec2 x dx.
s
In
UP

Solution.
Z Z
3 2
tan x sec x = tan2 x sec x sec x tan x dx
Z
= (sec2 x − 1) sec x sec x tan x dx
Z
= (sec3 x − sec x) sec x tan x dx.

Let u = sec x. Then du = sec x tan x.


Z Z
tan3 x sec2 x = (u3 − u) du
1 1
= sec4 x − sec2 x + C.
4 2
1.3. TRIGONOMETRIC INTEGRALS 15
Z
Example 1.3.10. Evaluate cot2 x csc x dx.

Solution.
Z Z
2
cot x csc x dx = (csc2 x − 1) csc x dx
Z
= (csc3 x − csc x) dx
Z
= csc3 x dx − ln | csc x − cot x|.
Z
1 1
Note that csc3 x dx = − csc x cot x + ln | csc x − cot x| + C. Hence, we have
2 2
Z
1 1
cot2 x csc x dx = − csc x cot x − ln | csc x − cot x| + C.
2 2
Z √
Example 1.3.11. Evaluate tan x sec4 x dx.

s
Solution.

ic
Z √ Z √

at
4
tan x sec x dx = tan x sec2 x sec2 x dx

em
Z √
tan x(1 + tan2 x) sec2 x dx
=
h
at
Z √ √
M


= tan x + tan5 x sec2 x dx
of

2√ 3 2√ 7
= tan x + tan x + C.
e

3 7
ut
tit

R R R
1.3.6 Integrals of the form sin mx cos nx dx, sin mx sin nx dx, cos mx cos nx dx
s
In

We willl use the following identities:


UP

1
sin mx cos nx = 2 [sin(m − n)x + sin(m + n)x]
1
sin mx sin nx = 2 [cos(m − n)x − cos(m + n)x]
1
cos mx cos nx = 2 [cos(m − n)x + cos(m + n)x]
Z
Example 1.3.12. Evaluate cos 3x cos 5x dx.

Solution.
Z Z
1
cos 3x cos 5x dx = (cos(3x + 5x) + cos(3x − 5x)) dx
2
Z
1
= (cos 8x + cos 2x) dx
2
 
1 1 1
= sin 8x + sin 2x + C
2 8 2
1 1
= sin 8x + sin 2x + C.
16 4
16 CHAPTER 1. TECHNIQUES OF INTEGRATION AND IMPROPER INTEGRALS

EXERCISES. Evaluate the following integrals.

Z
sin4 x − 2 sin2 x cos2 x + cos4 x
Z
1. cos3 3x dx 10. dx
sin2 x cos2 x
Z
tan4 (cosh−1 2x)
Z
2. cos2 x sin3 x dx 11. √ dx
4x2 − 1
Z
sin5 2x cos 2x dx
Z
3.
12. sin 7x cos 3x dx
Z 1 Z
4. sin2 πx cos2 πxdx 13. cos 4x cos 3xdx
0
Z   
1 3 1
3
Z
cos3 5x sin2 5x − sin 7x
5. cot x csc x dx 14. dx
2 2 csc 5x
Z    
3 1 3 1 csc4 x
Z
6. cot x csc x dx 15. dx
2 2 cot2 x

ic s
Z
π
7. (3 − 2 cot2 5x) csc2 5x dx
Z

at
4
16. tan4 x sec3 x dx

em
− π4
Z π
2
csc3 x cos2 x dx
8. Z h
tan3 (ln x) sec8 (ln x) dx
at
π
6 17.
x
M


cos3 x
Z
4 Z
9. √ dx
of

π sin x 18. (sec 9x + csc 9x)2 dx


4
e
ut
stit
In

1.4 Integration by Trigonometric Substitution


UP

√ √ √
If the integrand contains an expression of the form a2 − u2 , a2 + u2 or u2 − a2 , where a is
a positive constant, it is often possible to perform the integration by making a substitution that
results in an integral involving trigonometric functions.


1.4.1 Integrand contains an expression of the form a2 − u2
−π π
Let u = a sin θ, where ≤ θ ≤ . Then du = a cos θ dθ. Moreover, since cos θ ≥ 0, we have
2 2
p q
a − u = a2 (1 − sin2 θ)
2 2

= a| cos θ|
= a cos θ.
1.4. INTEGRATION BY TRIGONOMETRIC SUBSTITUTION 17

The triangle in Figure 1.1 will be helpful in obtaining other trigonometric functions when u = a sin θ.

Figure 1.1: Triangle for substitution u = a sin θ

Z √
49 − x2 dx
Example 1.4.1. Find .
x
Solution. Let x = 7 sin θ. Then dx = 7 cos θ dθ.
Z √
49 − x2
Z
7 cos θ
dx = 7 cos θ dθ

s
x 7 sin θ

ic
7 x
Z
7 cos2 θ

at
= dθ
sin θ

em
θ
√ 7(1 − sin2 θ)
Z
= dθ
49 − x 2
h
sin θ
at
R
= 7 (csc θ − sin θ) dθ
M

= 7 ln | csc θ − cot θ| + 7 cos θ + C


of

7 √49 − x2 p

e

= 7 ln − + 49 − x2 + C
ut


x x
s tit
In


UP

1.4.2 Integrand contains an expression of the form u 2 − a2


π 3π
Let u = a sec θ, where 0 ≤ θ < and π ≤ θ < .
2 2
Then du = a sec θ tan θ dθ. Moreover,

p p
u2 − a2 = a2 sec2 θ − a2
p
= a2 (sec2 θ − 1)

= a tan2 θ
= a tan θ
Figure 1.2: Triangle for substitution u = a sec θ
18 CHAPTER 1. TECHNIQUES OF INTEGRATION AND IMPROPER INTEGRALS
p
e4
ln2 x − 4
Z
Example 1.4.2. Find dx.
e2 x ln x

1
Solution. Let ln x = 2 sec θ. Then dx = 2 sec θ tan θ dθ.
x

e4
p Z π
ln2 x − 4
Z
3 2 tan θ

ln x dx = · 2 sec θ tan θ dθ
p
e2 x ln x 0 2 sec θ
ln2 x − 4 Z π
3
θ = 2 tan2 θ dθ
0
2 Z π
3
=2 (sec2 θ − 1) dθ
0
π
3
= 2(tan θ − θ)
0
√ π
=2 3−
3

ic s
R1√
Example 1.4.3. Evaluate 1 − x2 dx

at
0

Solution. From the definition of the definite integral, the expression h em R1√
0 1 − x2 dx is the area of
at
the region
M
of
e
ut
s tit
In
UP

Indeed, by trigonometric substitution, let x = sin θ. Then dx = cos θdθ

π
Z 1p Z
2
1− x2 dx = cos θ · cos θdθ
0 0
Z π
1 2
= (1 + cos 2θ)dθ
0 2
1 1 π
2
= (θ + sin 2θ)
2 4 0
π
=
4
1.4. INTEGRATION BY TRIGONOMETRIC SUBSTITUTION 19

1.4.3 Integrand contains an expression of the form a2 + u 2
−π π
Let u = a tan θ, where < θ < . Then du = a sec2 θ dθ. Moreover,
2 2

p p
a2 + u2 = a2 + a2 tan2 θ
q
= a2 (1 + tan2 θ)

= a sec2 θ
= a sec θ.
Figure 1.3: Triangle for substitution u = a tan θ
Z
dx
Example 1.4.4. Evaluate p .
(x − 6x + 25)3
2

Solution. Observe that x2 − 6x + 25 = x2 − 6x + 9 + 16 = (x − 3)2 + 42 . Let x − 3 = 4 tan θ. Then


dx = 4 sec2 θ dθ.
Z Z
dx dx

s
=

ic
p
2 3
p 3
(x − 6x + 25)
p
(x − 3)2 + 42

at
(x − 3)2 + 42
x−3

em
4 sec2 θ
Z
θ = dθ
(4 sec θ)3
h
at
4 1
Z
= cos θ dθ
M

16
1
of

= sin θ + C
16
e

1 x−3
ut

= √ +C
16 x2 − 6x + 25
s tit

The following table summarizes these forms:


In
UP

Table 1.6: Summary of Rules for Trigonometric Substitution

Integrand contains Substitution


√ √
a2 − u2 let u = a sin θ du = a cos θ dθ, a2 − u2 = a cos θ
√ √
a2 + u2 let u = a tan θ du = a sec2 θ dθ, a2 + u2 = a sec θ
√ √
u2 − a2 let u = a sec θ du = a sec θ tan θ dθ, u2 − a2 = a tan θ

EXERCISES. Evaluate the following integrals.

Z Z
1 1
1. dt 2. dy
(1 + t2 )3/2 (4 − y 2 )3/2
20 CHAPTER 1. TECHNIQUES OF INTEGRATION AND IMPROPER INTEGRALS
Z
1 x3
Z
3. dx 8. √ dx
(5 + 4x − x2 )5/2 1 − x2
Z
z
Z
1
4. dz 9. dx
(1 + z 4 )2 2x2 + 4x − 7
Z
1
Z p
5. √ dx 10. sin x 9 + cos2 x dx
x2 + 2x − 15
Z √
16 − e2x
Z
1
6. dx 11. √ dx
ex 1 + 1 − x2
4
√ Z
x2 − 4 1
Z
7. dx 12. √ dx
2 x ( x − 9)5
2

1.5 Integration of Rational Functions by Partial Fractions

In these section, we will introduce an algebraic method for rewriting certain rational functions.

s
f (x)

ic
Recall. A function h is a rational function if h(x) = , where f and g are polynomial functions.
g(x)

at
Consider the integral
h em
at
Z
1
M

dx.
x2 + 5x + 6
of

By completing the squares, we have


e
ut
tit

Z Z
1 1
s

dx = 1 dx
In

2
x + 5x + 6 x2 + 5x + 25 4 − 4
Z
1
UP

= 2 dx
5 2
x + 2 − 12


1 x + 3
=− ln +C (by Theorem 1.1.3, no. 28)
2 · (1/2) x + 2

x + 2
= ln
+C
x + 3
= ln |x + 2| − ln |x + 3| + C.

Alternatively,
Z Z Z
1 dx dx
2
dx = − = ln |x + 2| − ln |x + 3| + C.
x + 5x + 6 x+2 x+3

Our goal is to decompose a rational expression as a sum of two or more simpler quotients, called
partial fractions.
1.5. INTEGRATION OF RATIONAL FUNCTIONS BY PARTIAL FRACTIONS 21

Remark 1.5.1.
f (x)
1. We consider rational functions h(x) = where deg(f (x)) < deg(g(x)). If deg(f (x)) ≥
g(x)
deg(g(x)), we first divide the numerator by the denominator so that

r(x)
h(x) = q(x) + ,
g(x)

where deg(r(x)) < deg(g(x)).

2. Recall that any polynomial with real number coefficients can be expressed as a product of
linear and quadratic polynomials.

Case 1. The denominator of the rational function can be written as a product of distinct linear
factors.
If g(x) = (a1 x + b1 )(a2 x + b2 ) · · · (an x + bn ), where all factors are distinct, then

f (x) A1 A2 An
= + + ··· + , Ai ∈ R, i = 1, 2, . . . , n.

s
g(x) a1 x + b1 a2 x + b2 an x + bn

ic
at
Z
x
Example 1.5.2. Find dx.

em
2
x − 5x + 6
Solution. Note that
h
at
M

x x
=
of

x2 − 5x + 6 (x − 3)(x − 2)
e

A B
ut

= + .
x−3 x−2
tit

Multiplying both sides of the equation by (x − 3)(x + 2), we get


s
In

x = A(x − 2) + B(x − 3).


UP

We can solve for A and B by comparing the coefficients of like powers of x in the equation

x = A(x − 2) + B(x − 3) = (A + B)x − (2A + 3B).

x : 1 = A+B
constant : 0 = 2A + 3B

Thus, A = 3 and B = −2. Another way of solving for the values of A and B is to use the following
substitution:
let x = 2: 2 = B(−1) which gives B = −2
let x = 3: A = 3
Therefore,
Z Z  
x 3 2
dx = − dx = 3 ln |x − 3| − 2 ln |x − 2| + C.
x2 − 5x + 6 x−3 x−2
22 CHAPTER 1. TECHNIQUES OF INTEGRATION AND IMPROPER INTEGRALS

2x3 − 4x2 − 15x + 5


Z
Example 1.5.3. Evaluate dx.
x2 − 2x − 8
Solution. Since the degree of the numerator is greater than the denominator, we first divide the
numerator by the denominator. We have
2x3 − 4x2 − 15x + 5 x+5
= 2x +
x2 − 2x − 8 (x − 4)(x + 2).
x+5
We then apply the previous technique to the expression . We write
(x − 4)(x + 2)
x+5 A B
= + and obtain x + 5 = A(x + 2) + B(x − 4).
(x − 4)(x + 2) x−4 x+2
We use the following substitution:
1
let x = −2: 3 = B(−6) so that B = −
2
3
let x = 4: 9 = A(6) so that A =
2

s
Hence,

ic
at
2x3 − 4x2 − 15x + 5
Z Z  
x+5
dx = 2x + dx

em
x2 − 2x − 8 (x − 4)(x + 2)
2x2
Z  h 
3 1
= + − dx
at
2 2(x − 4) 2(x + 2)
M

3 1
= x2 + ln |x − 4| − ln |x + 2| + C.
of

2 2
e
ut

Case 2. The factors of the denominator of the rational function are all linear but some are repeated.
tit

If (ax + b)n , n > 1 is a factor of the denominator, then the partial fractions corresponding to this
s
In

factor are
A1 A2 An
+ + ··· + , Ai ∈ R, i = 1, 2, . . . , n.
UP

ax + b (ax + b) 2 (ax + b)n

(3x − 2)
Z
Example 1.5.4. Find dx.
2x3 − x2
Solution. Note that

3x − 2 3x − 2
3 2
= 2
2x − x x (2x − 1)
A B C
= + 2+
x x 2x − 1

Multiplying both sides by x2 (2x − 1), we get

3x − 2 = Ax(2x − 1) + B(2x − 1) + Cx2 .


1.5. INTEGRATION OF RATIONAL FUNCTIONS BY PARTIAL FRACTIONS 23

Solve for the coefficients:

letting x = 0 gives B = 2
1
letting x = gives C = −2
2
letting x = 1 (with the computed values of B and C) gives A = 1

Hence,

2x−1
Z  
(3x − 2)
Z
1 2 2
dx = + 2− dx = ln |x| + − ln |2x − 1| + C.
2x3 − x2 x x 2x − 1 −1

(x − 5) dx
Z
Example 1.5.5. Evaluate .
(x + 1)2 (x − 2)
Solution. We write
x−5 A B C
= + + ,

s
2
(x + 1) (x − 2) x + 1 (x + 1) 2 x−2

ic
at
and obtain

em
x − 5 = A(x + 1)(x − 2) + B(x − 2) + C(x + 1)2 .
h
at
We use the following substitution:
M

let x = −1: −6 = B(−3) which gives B = 2


of

1
let x = 2: −3 = C(3)2 which gives C = −
e

3
ut

−1 2 1
let x = 0: −5 = A(1)(−2) + 2(−2) + (1) which gives A =
tit

3 3
s
In

Therefore, we have
UP

Z  
x−5
Z
1 2 1
dx = + − dx
(x + 1)2 (x − 2) 3(x + 1) (x + 1)2 3(x − 2)
1 2(x + 1)−1 1
= ln |x + 1| + − ln |x − 2| + C.
3 −1 3
Case 3. The factors of the denominator of the rational function are linear and quadratic and none
of the quadratic factors is repeated.
If ax2 + bx + c is a quadratic factor of the denominator that is not repeated, then the corre-
Ax + B
sponding partial fraction to this factor is .
ax2 + bx + c
5x2 + 3x − 2
Z
Example 1.5.6. Find dx.
x3 − 1
Solution. The integrand can be written as

5x2 + 3x − 2 5x2 + 3x − 2 A Bx + C
= = + .
x3 − 1 (x − 1)(x2 + x + 1) x − 1 x2 + x + 1
24 CHAPTER 1. TECHNIQUES OF INTEGRATION AND IMPROPER INTEGRALS

Multiplying both sides of the equation by (x − 1)(x2 + x + 1), we get

5x2 + 3x − 2 = A(x2 + x + 1) + (Bx + C)(x − 1).

Solving for the coefficients:

let x = 1: 6 = A(3) so that A = 2


let x = 0: − 2 = 2(1) + C(−1) so that C = 4
let x = −1: 0 = 2(1) + (−B + 4)(−2) so that B = 3

Therefore,
5x2 + 3x − 2
Z Z Z
2 3x + 4
3
dx = dx + 2
dx
x −1 x−1 x +x+1
Z Z 3 (2x + 1) Z 5
2 2 2
= dx + dx + dx
x−1 x2 + x + 1 x2 + x + 1
!
3 5 1 x + 12
= 2 ln |x − 1| + ln |x2 + x + 1| + · √ tan−1 √ + C.

ic s
2 2 3/2 3/2

at
em
Case 4. Some quadratic factors of the denominator of the rational function are repeated.
h
If (ax2 + bx + c)n , n > 1 is a factor of the denominator, the corresponding partial fractions are
at
M

A1 x + b1 A2 x + b2 An x + bn
+ + ··· + .
ax2 + bx + c (ax2 + bx + c)2 (ax2 + bx + c)n
of
e

x3 + 1
Z
ut

Example 1.5.7. Find dx.


(x2 + 4)2
tit

Solution. The denominator of the integrand contains a repeated factor (x2 + 4)2 , so we write
s
In

x3 + 1 Ax + B Cx + D
UP

= 2 + 2 .
(x2 + 4)2 x +4 (x + 4)2
Multiplying both sides of the equation by (x2 + 4)2 , we get

x3 + 1 = (Ax + B)(x2 + 4) + (Cx + D) = Ax3 + Bx2 + (4A + C)x + D.

By comparing coefficients, we have A = 1, B = 0, D = 1. Also, with A = 1 and 4A + C = 0, we get


C = −4. Thus,
x3 + 1 1 − 4x
Z Z Z
x
2 2
dx = 2
dx + dx
(x + 4) x +4 (x2 + 4)2
1 − 4x
Z Z
1 2x
= 2
dx + dx
2 x +4 (x2 + 4)2
Z Z Z
1 2x 2x 1
= dx − 2 dx + dx
2 x2 + 4 (x2 + 4)2 (x2 + 4)2
(x2 + 4)−1
Z
1 2 1
= ln |x + 4| − 2 + dx.
2 −1 (x + 4)2
2
1.6. IMPROPER INTEGRALS 25
Z
1
We apply trigonometric substitution to evaluate dx.
(x2 + 4)2
Let x = 2 tan θ. Then dx = 2 sec2 θ dθ and x2 + 4 = 4(tan2 θ + 1) = 4 sec2 θ. Thus,

2 sec2 θ
Z Z
√ 1
x2 + 22 dx = dθ
x (x2 + 4)2 16 sec4 θ
Z
1
= cos2 θ dθ
θ 8
Z
2 1 1 + cos 2θ
= dθ
8 2
1 1
= θ+ sin 2θ + C
16 32
1 1
= θ+ · 2 sin θ cos θ + C.
16 32
Hence,
x3 + 1
Z
1 2 2 1 −1 x
  1 x 2
2 2
dx = ln |x + 4| + 2
+ tan + √ √ + C.
(x + 4) 2 x + 4 16 2 16 x + 4 x + 4
2 2

ic s
EXERCISES. Evaluate the following integrals.

at
1.
Z
8x − 2
dx
h em
7.
Z
(4x − 3)
dx
at
x3 − x x2− 2x − 3
M

17x2 + 7x − 3
Z Z
1
2. dx 8. dx
of

(x − 1)2 (2x + 1) x3 (x − 1)
e

2x2 − 8x + 12
Z
2x + 1
ut

Z
3. dx 9. dx
x4 + 4x2 x2 (x2+ 2)
stit

−x3 − x2 − 9x + 3 (x3 − 2x)


Z Z
In

4. dx 10. dx
x4 − 2x2 − 3 (x2 + 1)2
UP

x3 + 3x2 + 4x + 1 tan−1 x
Z Z
5. dx 11. dx
(x2 + 2x + 2)2 x2
3x4 + 5
Z Z
x
6. dx 12. √ dx (Hint: Let x = z 3 .)
(x2 + 1)2 1+ x

1.6 Improper Integrals


Rb
In previous calculus course, the definite integral a f (x)dx was defined for a function f and a finite
interval [a, b] such that f is continuous on [a, b]. Geometrically, the value of the definite integral
gives the net signed area of the region bounded by the graph of y = f (x) on the interval [a, b]. In
particular, if f (x) ≥ 0 for all x ∈ [a, b], then the area of the region under the graph of y = f (x),
bounded below by the x-axis, between x = a and x = b (see Figure 1.4) is given by
Z b
A= f (x)dx.
a
26 CHAPTER 1. TECHNIQUES OF INTEGRATION AND IMPROPER INTEGRALS

Figure 1.4: Region bounded by y = f (x), x-axis, x ∈ [a, b]

We will extend the concept of definite integrals to include cases where

1. one or both of the limits of integration are infinite or

ic s
at
2. f has an infinite discontinuity on the interval [a, b].
em
h
at
M
of

1.6.1 Integrals with Infinite Limits of Integration


e
ut
tit

1
Consider the infinite region R bounded above by the graph of y = 2 , bounded below by the x-axis
x
s
In

and to the right of the line x = 1. See Figure 1.5.


UP

1
Figure 1.5: Region bounded by y = , x-axis and to right of the line x = 1
x2
1.6. IMPROPER INTEGRALS 27

1
Figure 1.6: Region bounded by y = , x-axis , x ∈ [1, t]
x2
The area of the portion of the region on [1, t], for some t ∈ (1, +∞) (see Figure 1.6), is

s
Z t
1 1

ic
At := dx = − + 1.

at
x 2 t
1

em
Suppose that we move t towards the right indefinitely, that is, we let t → +∞. Then
h
at
Z t  
1 1
lim At = lim dx = lim − + 1 = 1.
M

t→+∞ t→+∞ 1 x2 t→+∞ t


of

Thus, we define the area of the infinite region R to be


e
ut

Z +∞ Z t
1 1
A= dx = lim dx = 1.
tit

1 x2 t→+∞ 1 x2
s
In

Observe that the fist integral above contains +∞ as a bound of integration. Such integral is an
UP

example of an improper integral.

Definition 1.6.1. Let a, b, c ∈ R.

1. If f is continuous on [a, +∞), then the improper integral of f over [a, +∞) is defined as
Z +∞ Z t
f (x) dx = lim f (x) dx.
a t→+∞ a

2. If f is continuous on (−∞, b], then the improper integral of f over (−∞, b] is defined as
Z b Z b
f (x) dx = lim f (x) dx.
−∞ t→−∞ t

The above equalities hold provided the limits exist. In which case, the improper integrals are
said to be convergent. If the limits do not exist, the corresponding integrals are said to be
divergent.
28 CHAPTER 1. TECHNIQUES OF INTEGRATION AND IMPROPER INTEGRALS

3. If f is continuous on R, then the improper integral of f over (−∞, +∞) is defined as


Z +∞ Z c Z +∞
f (x) dx = f (x) dx + f (x) dx
−∞ −∞ c
Rc R +∞
provided that both −∞ f (x) dx and c f (x) dx are convergent. If at least one of the
improper integrals on the right-hand side is divergent, then the improper integral on the
left-hand side is divergent.

Example 1.6.2. Determine whether each of the following improper integrals is convergent or
divergent. Evaluate the improper integrals that are convergent.

Z +∞ Z +∞
−x ln x
1. e dx 3. dx
0 1 x
Z −3 Z +∞
x 1
2. dx 4. dx
−∞ (x − 4)2
2
−∞ x2 + 2x + 5

ic s
at
Solution.

1. Using item 1 of Definition 1.6.1, we have h em


at
Z +∞ Z t
M

e−x dx = lim e−x dx


0 t→+∞ 0
of

t
= lim −e−x

e

t→+∞ 0
ut

−t
= lim (−e + 1) = 1
tit

t→+∞
s
In

Hence, the improper integral is convergent.


UP

2. Using item 2 of Definition 1.6.1, we have


−3 −3
−1 −3
Z Z
x x dx
dx = lim = lim
(x − 4)2
2 (x2 − 4)2 t→−∞ 2(x2 − 4) t

−∞ t→−∞ t
 
−1 −1 −1
= lim + 2
=
t→−∞ 2 · 5 2(t − 4) 10

Hence, the improper integral is convergent.

3. Using item 1 of Definition 1.6.1, we have


+∞ t
ln2 x t
Z Z
ln x ln x
dx = lim dx = lim
x x t→+∞ 2

1 t→+∞ 1 1
 2 
ln t
= lim − 0 = +∞
t→+∞ 2

Hence, the improper integral is divergent.


1.6. IMPROPER INTEGRALS 29

4. Using item 3 of Definition 1.6.1 with c = 1, we can write

Z +∞ Z 1 Z +∞
1 1 1
2
dx = 2
dx + dx.
−∞ x + 2x + 5 −∞ x + 2x + 5 1 x2 + 2x + 5

Meanwhile,

Z 1 Z 1  
1 1 1 −1 x + 1
1
2
dx = lim 2
dx = lim tan
x + 2x + 5 t (x + 1) + 4 t→−∞ 2 2

−∞ t→−∞ t
  
1 t + 1 1 h π  π i 3π
= lim tan−1 (1) − tan−1 = − − = ,
t→−∞ 2 2 2 4 2 8

+∞ k  

s
Z Z
1 1 1 −1 x + 1
k

ic
2
dx = lim 2
dx = lim tan
x + 2x + 5 1 (x + 1) + 4 k→∞ 2 2

k→∞ 1

at
1
   
1 k+1 1 π π π

em
 
= lim tan−1 − tan−1 (1) = − = .
k→∞ 2 2 h 2 2 4 8
at
M
of

Z +∞
1 3π π π
Therefore, dx = + = .
e

x2 + 2x + 5 8 8 2
ut

−∞
s tit
In

One must solve carefully for an improper integral


Z +∞where both bounds are infinite.
UP

In particular, consider f (x) = x. Evaluating f (x)dx using Definition 1.6.1 , we have


−∞

Z +∞ Z 0 Z +∞
x dx = x dx + x dx
−∞ −∞ 0
Z 0 Z b
= lim x dx + lim x dx
a→−∞ a b→+∞ 0
x2 0 x2 b
= lim + lim

2 a b→+∞ 2 0

a→−∞
a2
   2 
b
= lim 0 − + lim +0 .
a→−∞ 2 b→+∞ 2

Since both limits do not exist, the improper integral is divergent.


30 CHAPTER 1. TECHNIQUES OF INTEGRATION AND IMPROPER INTEGRALS

Figure 1.7: Region bounded by x−axis and f (x) = x


Z c
Consider now the expression lim f (x) dx. We have
c→+∞ −c

c
x2 c c2 (−c)2
Z

s
ic
lim x dx = lim = lim − = 0.
c→+∞ −c c→+∞ 2 −c c→+∞ 2 2

at
em
This illustrates the following.

Remark 1.6.3. Let f be continuous on R. Then


h
at
Z +∞ Z c
M

f (x) dx 6= lim f (x) dx


−∞ c→+∞ −c
of
e

1.6.2 Integrals with Infinite Discontinuity


ut

1
tit

Consider the region under the graph of y = √ , above the x−axis, between the lines x = t and
x
s
In

x = 1, for some 0 < t < 1, as shown in Figure 1.8.


The area of the shaded region is
UP

Z 1
dx √ 1 √
At := √ = 2 x = 2 − 2 t.
t x t

If we let t → 0+ , we get
Z 1
dx √
lim At = lim √ = lim 2 − 2 t = 2.
t→0+ t→0+ t x t→0+
1
Thus, we define the area of the infinite region bounded by y = √ , the x−axis, and lines x = 0
x
and x = 1 (see Figure 1.9) to be
Z 1 Z 1
1 dx
A= √ dx = lim √ = 2.
0 x t→0+
t x

Observe that the above integral does not satisfy continuity of the function on the interval of
integration. This is another example of an improper integral.
1.6. IMPROPER INTEGRALS 31

1
Figure 1.8: Region bounded by y = √ , x−axis and lines x = t and x = 1
x

Definition 1.6.4. Let a, b, c ∈ R.

ic s
at
1. If f (x) is continuous on (a, b] and lim f (x) = ±∞, then the improper integral of f over [a, b]

em
x→a+
is defined as Z b h Z b
f (x) dx = lim f (x) dx.
at
a t→a+ t
M
of

2. If f (x) is continuous on [a, b) and lim f (x) = ±∞, then then the improper integral of f over
x→b−
e

[a, b] is defined as
ut

Z b Z t
tit

f (x) dx = lim f (x) dx.


a t→b− a
s
In

The above equalities hold if the limits exist. In which case, we say the improper integrals
UP

are convergent. If the limits do not exist, then the corresponding integrals are said to be
divergent.
Rc Rb
3. If f has an infinite discontinuity at c, where a < c < b, and both a f (x) dx and c f (x) dx
are convergent, then we define
Z b Z c Z b
f (x) dx = f (x) dx + f (x) dx.
a a c
Z b
If either integral on the right-hand side is divergent, then f (x) dx is divergent.
a

Example 1.6.5. Determine whether each of the following improper integral is convergent or di-
vergent. Evaluate the improper integrals that are convergent.
32 CHAPTER 1. TECHNIQUES OF INTEGRATION AND IMPROPER INTEGRALS

1
Figure 1.9: Region bounded by y = √ , x−axis and lines x = 0 and x = 1
x

s
2 π
+∞
e− x
Z
1
Z Z

ic
2
1. √ dx 2. tan x dx 3. √ dx

at
−4 16 − x2 0 0 x

Solution. em
h
at
M

1. On the given interval, the integrand has an infinite discontinuity at x = −4 since


of
e

1
ut

lim √ = +∞.
16 − x2
tit

x→−4+
s
In

Applying item 1 of Definition 1.6.4, we have


UP

Z 2 Z 2
dx dx
√ = lim √
−4 16 − x2 t→−4+ t 16 − x2
 x  2
= lim sin−1
4 t

t→−4+
    
−1 1 −1 t
= lim sin − sin
t→−4+ 2 4
 
1 2π
= sin−1 − sin−1 (−1) = .
2 3

Hence, the improper integral is convergent.

π
2. On the given interval, the integrand has an infinite discontinuity at x = 2 since

lim tan x = +∞.


x→ π2 −
1.6. IMPROPER INTEGRALS 33

Applying item 1 of Definition 1.6.4, we have


π
Z
2
Z t
tan x dx = lim tan x dx
0 t→ π2 − 0
t
= lim (ln | sec x|)

t→ π2 − 0

= lim (ln | sec t| − ln | sec 0|)


t→ π2 −

= +∞.

Hence, the improper integral is divergent.

3. The upper bound of integration is infinite, which falls under the first general case of an
improper integral. Furthermore, The integrand has an infinite discontinuity at x = 0 since

e− x
lim √ = +∞.
x→0+ x

ic s
We can write the given integral as a sum of two improper integrals by choosing any number

at
larger than 0. In particular, we havE
Z +∞

e−

x
dx =

e− x
Z
√ dx +
1 Z +∞ −√x
e
h em
√ dx
at
0 x 0 x 1 x
M

√ 1 √ k
= lim −2e− x + lim −2e− x

of

t→0+ t k→+∞ 1
√ √
   
−2 2
e

− t − k
= lim + 2e + lim −2e +
ut

t→0+ e k→+∞ e
tit

= 2 + 0 = 2.
s
In

Hence, the improper integral is convergent.


UP

EXERCISES. Determine whether each of the following improper integral converges or diverges.
Evaluate the improper integrals which are convergent.

Z 0 Z +∞
1 1
1. dx 5. √ dx
−∞ x2 + 16 2 x x2 − 4
Z 1 Z 1
1
2. ln x dx 6. dx
0 −1 x2 − 2x

2 +∞
x3
Z
1
Z
3. dx 7. dx
−∞ x4 − 4 −∞ 1 + 4x2
Z 2 Z +∞
1 1+x
4. dx 8. dx
0 (2x − 1)2/3 −∞ 1 + x2
34 CHAPTER 1. TECHNIQUES OF INTEGRATION AND IMPROPER INTEGRALS
Z 1 Z 0
1
9. −x
dx 11. xe3x dx
0 e − ex −∞
Z +∞ √
Z 2
(3x + 2)
10. e− x
dx 12. dx
0 0 (x − 2)(x2 + 4)

1.7 Chapter Exercises


I. Find the following antiderivatives.

Z Z √
1. x
e cos(6x) dx 16 − e2x
7. dx
ex
Z Z √
2. e 2x
sin 4x dx 4 − x4
8. dx

s
x

ic
Z √

at
Z
2 x4 − 4
3. x ln(2x + 1) dx 9. dx

em
x
17x2 + 4x + 2
Z Z h
cos2 x cos x sin2 x − 1 dx

4. 10. dx
at
(4x − 1)(x2 + 4)
M

4x2 + x − 2
Z Z
5. tan5 x sec3/2 x dx 11. dx
of

(3x − 4)(x2 + 2x + 2)
√ 6x2 + 2x + 1
e

Z Z
sec6 x tan x dx
ut

6. 12. dx
x2 (4x2 + 1)
tit
s
In

II. Evaluate the following improper integrals.


UP

+∞
sin x1
Z Z +∞

2
1. xe1−x dx 4. dx
1 1/π x2
1 Ze
dx
Z
dx
2. 5.
0 x(1 + ln2 x) 1 x(ln x)1/2
0
e x2
Z Z
3. ln x dx 6. dx
0 −∞ e x3
Chapter 2

Parametric Curves

2.1 Parametric Equations of Plane Curves


Consider a particle moving along a curve where the x- and y- coordinates vary with respect to

ic s
time, as shown in the figure below.

at
h em
at
M
of
e
ut
s tit
In

The curve cannot be expressed as the graph of an equation of the form y = f (x) since it fails the
UP

vertical line test. Sometimes, it is not possible to find a single equation relating the x-coordinate
and y-coordinate of the position of the particle at a given time. But we can express the coordinates
of the particle’s position as functions of the time t; that is,

x = x(t) and y = y(t).

In this process, we say that we are parametrizing the curve above. The equations defining x and
y as functions of t are called parametric equations, and t is called a parameter. The curve is
referred to as a parametric curve.

A parametric curve is always traced according to increasing values of the parameter. Moreover,
if t ∈ [a, b], and [a, b] is a subset of the domains of both x(t) and y(t), then (x(a), y(a)) is called
initial point while (x(b), y(b)) is called terminal point of the parametric curve.

Example 2.1.1. Consider the curve C defined by x = t, y = t2 , where t ∈ R.

35
36 CHAPTER 2. PARAMETRIC CURVES

Giving several values for t, we get the points:

t (x, y)

−2 (−2, 4)
−1 (−1, 1)
0 (0, 0)
1 (1, 1)
2 (2, 4)
3 (3, 9)

Some points of the curve x = t, y = t2 , t ∈ R

If we relate x and y directly by eliminating t, we get y = x2 . We have just obtained the Cartesian

s
ic
form of the parametric curve. Note that for some parametric curves, the Cartesian form may be

at
difficult to compute or may not even exist. We have the following observations.

Remark 2.1.2. h em
at
1. A curve given by a set of parametric equations has an orientation, given by the direction by
M

which the curve is traced out as the parameter increases.


of
e

2. The parametrization of a given curve is not unique. Starting from the set of parametric
ut

equations, eliminating the parameter gives the Cartesian equation of the curve. However, the
tit

corresponding Cartesian curve may have points which are not on the parametric curve.
s
In
UP

Let us look at some examples.

Example 2.1.3. Consider the points A(0, 1) and B(3, 7). Parametrize the line through A and B;
line segment from A to B.

Solution. Using the two-point form of a line, the Cartesian equation of the line through A and B is
7−1
y−1= (x − 0) which simplifies to y = 2x + 1.
3−0
Hence, by setting x = t, we can parametrize the line through A and B as

x = t, y = 2t + 1, t ∈ R.

The line segment from A to B is parametrized by making an appropriate restriction on the param-
eter t. Since x = t and x varies from 0 to 3, we have

x = t, y = 2t + 1, t ∈ [0, 3].
2.1. PARAMETRIC EQUATIONS OF PLANE CURVES 37

Example 2.1.4. Sketch the curve defined by x = 2 cos t and y = 2 sin t, where 0 ≤ t ≤ 2π.

Solution. By looking at several values of t we get:

t (x, y)

0 (0, 2)
π √ √
( 2, 2)
4
π
(0, 2)
2
π (−2, 0)

(0, −2)
2
2π (2, 0)

s
The parametric curve is a circle traced counterclockwise with initial and terminal points at (2, 0).

ic
at
Indeed, by squaring both parametric equations and adding them up, we get

em
x2 + y 2 = 4 cos2 t + 4 sin2 t = 4,
h
at
a circle centered at the origin of radius 2.
M

Example 2.1.5. Sketch the curve defined by x = 2 sin t and y = 2 cos t, where 0 ≤ t ≤ 2π.
of

Solution. By looking at several values of t we get:


e
ut
tit

t (x, y)
s
In

0 (0, 2)
UP

π √ √
( 2, 2)
4
π
(2, 0)
2
π (0, −2)

(−2, 0)
2
2π (0, 2)

The parametric curve is a circle traced clockwise with initial and terminal points at (0, 2). Doing
the same technique as illustrated in Example 2.1.4, we get the same Cartesian equation. This
clearly shows that the parametrization of a curve is not unique.

Example 2.1.6. Consider the parametric curve defined by x = 3 cos t and y = 2 sin t, where
0 ≤ t ≤ 2π.
38 CHAPTER 2. PARAMETRIC CURVES

Solution. Note that


x y
= cos t and = sin t.
3 2
Hence
x 2 y 2  x 2  y 2
cos2 t + sin2 t = + = + = 1.
9 4 3 2
x2 y2
Thus, + = 1, which is the equation of an
9 4
ellipse.

Example 2.1.7. Consider the parametric curve defined by x = cosh t, y = sinh t where t ∈ R.

Solution. Note that

s
cosh2 t − sinh2 t = x2 − y 2 = 1.

ic
at
Recall that this equation describes a hyperbola.
h em
at
Since cosh t > 0 for all t ∈ R, we only consider
M

the portion of the hyperbola where x > 0, that


of

is, the right branch of the hyperbola.


e
ut

Let us look at some generalizations on parametric curves.


s tit
In

Remark 2.1.8.
UP

1. A curve given by y = f (x) can be parametrized by x = t, y = f (t). Similarly, a curve given


by x = g(y) can be parametrized by x = g(t), y = t.

2. A circle centered at (h, k) with radius a

(a) traced counterclockwise can be parametrized by x = h + a cos t, y = k + a sin t, where


t ∈ [0, 2π];

(b) traced clockwise can be parametrized by x = h + a sin t, y = k + a cos t, where t ∈ [0, 2π].

3. An ellipse with equation


(x − h)2 (y − k)2
+ =1
a2 b2
can be parametrized by x = h + a cos t, y = k + b sin t, where t ∈ [0, 2π].
2.2. CALCULUS OF PARAMETRIC CURVES 39

EXERCISES. Do as indicated.

1. Consider the points P (−1, 3) and Q(3, 5).

(a) Parametrize the line through P and Q.


(b) Parametrize the line segment from P to Q.
x2
2. Parametrize the ellipse + 4y 2 = 1 traced in the clockwise direction.
4
3. Sketch the curve defined by the parametric equations x = cos t and y = ecos t .

4. Consider the parametric curve C defined by x(t) = sin2 t and y(t) = cos t, 0 ≤ t ≤ π.

(a) Sketch the graph of the portion of the given parametric curve.
(b) Find the corresponding Cartesian equation for C.

5. Consider the curve C with parametric equations x = sinh t and y = cosh t.

(a) Find the corresponding Cartesian equation of C.

ic s
(b) Sketch a portion of the graph of C.

at
em
6. Parametrize the upper semi-circle of x2 + y 2 = 1 using the slope of the tangent line to a point
h
as the parameter.
at
M

2.2 Calculus of Parametric Curves


of
e
ut

In this section, we shall study tangent lines, concavity, and arclength of parametric curves. We
tit

denote by C a parametric curve given by equations x = x(t) and y = y(t).


s
In

Recall that for a curve with equation y = f (x), where f is differentiable at x = x0 , the slope of the
UP

tangent line at x = x0 is given by f 0 (x0 ).

2.2.1 Tangent Line to a Parametric Curve


We first define the smoothness of parametric curves.

Definition 2.2.1. A parametric curve C defined by parametric equations x = x(t) and y = y(t) is
said to be smooth if both x0 (t) and y 0 (t) are continuous and no value of t satisfies x0 (t) = y 0 (t) = 0,
for any allowable values of t.

If C is smooth, then x0 (t) and y 0 (t) are not both zero. In particular, if x0 (t) 6= 0, the chain rule
implies that
dy
dy y 0 (t)
= dt = 0 .
dx dx x (t)
dt
Hence, we arrive at the following result.
40 CHAPTER 2. PARAMETRIC CURVES

Theorem 2.2.2 (Tangent Lines to Parametric Curves). If C is a smooth curve that is described
by the parametric equations x = x(t) and y = y(t), then
dy
dy dx
= dt , provided 6= 0.
dx dx dt
dt

dy
The slope of the tangent line to a given parametric curve C at (x(t0 ), y(t0 )) is given by .
dx t=t0

Example 2.2.3. Find the equation of the tangent line to the right branch of the hyperbola defined
π π π
by the parametric equations x = sec t, y = tan t, where − ≤ t ≤ , at the point where t = .
2 2 4
Solution. Using the formula in Theorem 2.2.2, we get

dy dy/dt
=
dx dx/dt

ics
sec2 t
=

at
tan t sec t

em
= csc t.
π
h
at
The slope of the tangent line at t = is
4
M



dy
of

= csc(π/4) = 2.
dx t=π/4

e
ut

π  √ π 
Moreover, x =
2 and y = 1.
tit

4 4
s

Thus, the equation of the tangent line is


In

√  √ 
UP

y−1= 2 x− 2 .

Example 2.2.4. Find the equation of the tangent line to the curve parametrized by x = 3t − cos t
and y = t2 + 2e3t + 1 at the point where t = 0.

Solution. At t = 0, x = 3(0) − cos 0 = −1 and y = 02 + 2e3(0) + 1 = 3. Thus, the point of tangency


has coordinates (−1, 3).
dx dy
Now, = 3 + sin t and = 2t + 6e3t . This means,
dt dt
dy dy/dt 2t + 6e3t
= = .
dx dx/dt 3 + sin t
In particular,
2(0) + 6e3(0)

dy
= = 2.
dx t=0 3 + sin 0
2.2. CALCULUS OF PARAMETRIC CURVES 41

Hence, the tangent line has equation

(y − 3) = 2(x + 1).

Remark 2.2.5. Let C be a curve defined by the parametric equations x = x(t) and y = y(t).
dy dx
1. The tangent lines to C are horizontal at those points where = 0 and 6= 0.
dt dt
dx dy
2. The tangent lines to C are vertical at those points where = 0 and 6= 0.
dt dt
dx dy
3. If both and are zero at t = t0 , then C makes a sharp turn at the point (x(t0 ), y(t0 )).
dt dt
dy
In such case, we define the slope of the tangent line to be lim , if this limit exists.
t→t0 dx

Example 2.2.6. Find all values of t where the tangent line to the parametric curve defined by the
equations x = 2t3 + 3t2 − 12t and y = 3t4 − 4t3 + 7 is horizontal; vertical.

ic s
dx dy
= 6t2 + 6t − 12 and = 12t3 − 12t2 . Note that

at
Solution. We have
dt dt

if
dx
dt
h em
= 0, then 6t2 + 6t − 12 = 6(t − 1)(t + 2) = 0, i.e. , t = 1, t = −2,
at
dy
M

and if = 0, then 12t3 − 12t2 = 12t2 (t − 1) = 0, i.e. , t = 0, t = 1.


dt
of

Thus, the parametric curve has a horizontal tangent line at t = 0 and a vertical tangent line at
e

t = −2.
ut
s tit

Observe that both derivatives are zero at t = 1. For this, we compute:


In

dy dy/dt
UP

lim = lim
t→1 dx t→1 dx/dt

12t3 − 12t2
= lim 2
t→1 6t + 6t − 12
12t2 (t − 1)
= lim
t→1 6(t − 1)(t + 2)

2t2
= lim
t→1 t + 2
2
= .
3
Therefore, the curve has a tangent line at t = 1 that is neither horizontal nor vertical.

2.2.2 Concavity of Parametric Curves


d2 y
In this section, we describe the concavity of a parametric curve using the second derivative .
dx2
Recall from the previous calculus course that for a curve described by the equation y = f (x), the
42 CHAPTER 2. PARAMETRIC CURVES

second derivative gives the concavity of the graph of f . In particular, if f 00 (x) > 0 for all x on an
open interval I, then the graph of f is concave up on I. On the other hand, if f 00 (x) < 0 for all x
on an open interval I, then the graph of f is concave down on I.

dy
Let C be a smooth parametric curve defined by x = x(t) and y = y(t). Note that is also a
dx
dy
function of the parameter t. Let ỹ(t) = . Reapplying Theorem 2.2.2, we get
dx
 
dy
d /dt
dỹ dỹ/dt dx
= = .
dx dx/dt dx/dt
dy dỹ d2 y
Moreover, since ỹ = , then = 2 . Thus,
dx dx dx
 
dy
d /dt
d2 y dx
= .
dx2 dx/dt

ic s
Theorem 2.2.7 (Second Derivative). Consider the curve C described by parametric equations

at
x = x(t) and y = y(t). The second derivative of y with respect to x is

em
 
dy h
d /dt
d2 y dx
at
= .
dx2 dx/dt
M

Furthermore,
of

d2 y
e

• if > 0 for all t on an open interval I, then the curve is concave up on I;


ut

dx2
tit

d2 y
• if < 0 for all t on an open interval I, then the curve is concave down on I.
s

dx2
In
UP

Example 2.2.8. Determine the values of t for which the curve C described by the parametric
equations x = t − t2 and y = t − t3 is concave up; concave down.
dy dy/dt 1 − 3t2
Solution. First, = = . Thus,
dx dx/dt 1 − 2t

1 − 3t2
 
d
1 − 2t
d2 y dt
=
dx2 dx/dt
(1 − 2t)(−6t) − (1 − 3t2 )(−2)
(1 − 2t)2
=
1 − 2t
6t2 − 6t + 2
= .
(1 − 2t)3
1 1
For all t, 6t2 − 6t + 2 > 0. Thus, C is concave up when t < and concave down when t > .
2 2
2.2. CALCULUS OF PARAMETRIC CURVES 43

d2 y y 00 (t) y 00 (t)
Remark 2.2.9. In general, 6
= . It can be seen from the previous example that = 3t
dx2 x00 (t) x00 (t)
d2 y 6t2 − 6t + 2
is not the same as = .
dx2 (1 − 2t)3

2.2.3 Arc Length of a Parametric Curve

Consider a smooth parametric curve C defined on an interval [a, b]. Suppose that C is traced exactly
once as t varies from a to b. First, we divide [a, b] into n subintervals [t0 , t1 ], [t1 , t2 ], . . . , [tn−1 , tn ]
where
a = t0 < t1 < t2 < · · · < tn−1 < tn = b

and denote ∆ti = ti − ti−1 for i = 1, 2, . . . , n. Next, denote

Pi : (xi , yi ) = (x(ti ), y(ti )), ∆xi = xi − xi−1 and ∆yi = yi − yi−1

for i = 0, 1, . . . , n.

ic s
at
h em
at
M
of
e
ut
s tit
In

By the distance formula between two points, the length of the line segment from Pi−1 to Pi is given
by
UP

s
∆xi 2 ∆yi 2
  
p
2 2
(∆xi ) + (∆yi ) = + · ∆ti
∆ti ∆ti
for i = 1, 2, . . . , n. Thus, the length of C on [a, b] is approximately
s
n 2  2
X ∆xi ∆yi
+ · ∆ti .
∆ti ∆ti
i=1

To lessen the error of this approximation, we let n → +∞ in such a way that ∆ti → 0 for
i = 1, 2, . . . , n. Thus, the length L of the curve C on [a, b] is
s s
n 2  2 Z b 2  2
X ∆xi ∆yi dx dy
L = lim + · ∆ti = + dt.
n→+∞ ∆ti ∆ti a dt dt
i=1
44 CHAPTER 2. PARAMETRIC CURVES

Theorem 2.2.10 (Arc length). Let C be a smooth parametric curve described by parametric
equations x = x(t) and y = y(t) that is traced exactly once as t varies from a to b. The length L
of C on [a, b] is given by s
Z b  2  2
dx dy
L= + dt.
a dt dt
4t3
Example 2.2.11. Find the length of C : x = + 1, y = 2 − t3 , where t ∈ [0, 3].
3
dx dy
Solution. Note that = 4t2 and = −3t2 . Thus,
dt dt
Z 3p
L= (4t2 )2 + (−3t2 )2 dt
0
Z 3p
= 16t4 + 9t4 dt
0
Z 3
= 5t2 dt
0
3

s
5t3

ic
=

at
3

0
= 45. h em
at
The length of the parametric curve is 45 units.
M

Remark 2.2.12. Let C be a piecewise-smooth parametric curve on an interval [a, b]; that is,
of

C is a union of n smooth parametric curves C1 , C2 , . . . , Cn , corresponding to the subintervals


e
ut

[a, a1 ], [a1 , a2 ], . . . , [an−1 , b]. The arc length of C is the sum of the arc lengths of C1 , C2 , . . . , Cn .
tit

Example 2.2.13. Find the length of the astroid x = cos3 t, y = sin3 t, where 0 ≤ t ≤ 2π.
s
In

π 3π
Solution. Note that the astroid is not smooth at t = 0, , π, , 2π. Thus, we cannot use the
2 2
UP

formula directly from t = 0 to t = 2π. Instead, we can partition the astroid into four smooth
parametric curves corresponding to the intervals [0, π2 ], [ π2 , π], [π, 3π 3π
2 ], [ 2 , 2π]. Let us consider the
smooth portion of the astroid [0, π2 ]. If we denote the arc length of this portion by L1 , we have
Z π q
2 2
L1 = (−3 cos2 t sin t)2 + 3 sin2 t cos t dt
0
Z π
2
= 3 sin t cos t dt
0
π !
3 2
= sin2 t
2 0
3
= .
2

In the same manner, it can be shown that the portion on [ π2 , π], [π, 3π 3π
2 ] and [ 2 , 2π] is each of length
3
2 . Hence, the arc length of the astroid is 6 units.
2.2. CALCULUS OF PARAMETRIC CURVES 45

Example 2.2.14. Set up the necessary definite integral that will find the following.

1. the circumference of the circle x = 4 cos t, y = 4 sin t

2. the perimeter of the ellipse x = 2 + 9 cos t, y = 1 − 4 sin t

3. the arc length of the curve x = 4et , y = 5 − t2 , with t ∈ [1, 3]

Solution. All given curves are smooth and thus, using Theorem 2.2.10, we have
Z 2π p
1. L = (−4 sin t)2 + (4 cos t)2 dt
0
Z 2π p
2. L = (−9 sin t)2 + (−4 cos t)2 dt
0
Z 3p
3. L = (4et )2 + (−2t)2 dt
1

ic s
EXERCISES. Do as indicated.

at
em
1. Given the parametric curve C with equations
h
at
x(t) = ln(1 + t2 ), y(t) = tan−1 t.
M

(a) Determine the values of t where C has a vertical or a horizontal tangent line.
of

d2 y
e

(b) Find at t = 1.
ut

dx2
tit

(c) Is C concave up or concave down when t = 2?


s

(d) Set up an integral for the arclength of C from (0, 0) to (ln 2, π4 ).


In
UP

2. Given the parametric equations x = t2 − t − 6 and y = t2 + t3 .

(a) Find all values of t where the curve has vertical tangent lines.
d2 y
(b) Evaluate at the point where (x, y) = (0, −4).
dx2
(c) Set up the integral that would give the length of the arc from t = 1 to t = 2.

3. Given the parametric curve C defined by


1
x= , y = ln(1 + t).
1+t

dy d2 y
(a) Determine and .
dx dx2
(b) Determine the equation of the tangent line to C at t = 0.
(c) Set up the integral needed to find the arclength of C from (1, 0) to (0.25, ln 4).
46 CHAPTER 2. PARAMETRIC CURVES

2.3 Chapter Exercises


I. Sketch the graph of the following parametric equations:

1. x = t2 , y = t4 4. x = et , y = e2t , t ∈ [0, +∞)


2. x = cos t, y = cos2 t 5. x = 3 sin t, y = 4 cos t, t ∈ [0, π]
3. x = 4, y = sin t 6. x = 2 sec t, y = tan t, t ∈ [− π4 , π4 ]

II. Find a parametrization for the following curves:

1. The circle centered at (2, 1) of radius 3.


2. The branch of the hyperbola (y − 1)2 − x2 = 1 below the x−axis.
3. The triangle with vertices (1, 0), (0, 5) and (0, 0).

III. Find the slope of the tangent line to the given curve at the specified point.

ic s
1

at
1. x = 2t, y = at (0, 1)
1 + t2
2. x = et cos t, y = et sin t at (0, eπ/2 ) hem
at
3. x = t − 1, y = t4 − t3 + t2 − t + 1 at (−2, 1)
M

IV. Find the length of the following curves.


of


e

1. x = 9 − t2 , y = t, where −3 ≤ t ≤ 3
ut
tit

2. x = 2t, y = cos 4t, where 0 ≤ t ≤ π/2


s

3. x = cos5 t, y = sin5 t, where 0 ≤ t ≤ 2π


In
UP

V. Let C be the curve defined by the parametric equations x = t3 − 3t + 2 and y = 3t2 − 9.

1. Determine the points where C has a horizontal or a vertical tangent line.


2. Find the equation of the tangent line to C at the point (4, 3).
d2 y
3. Evaluate at t = 2.
dx2
4. Determine the length of the portion of C on t ∈ [0, 2].

VI. Given the set of parametric equations of a curve C defined by


h π
x = ln(cos t), y = sin t, t ∈ 0, .
2
1. Find the corresponding Cartesian equation for C.
2. Set up the integral needed to find the length of the portion of C when t ∈ [0, π4 ].
π
3. Without eliminating the parameter, find the second derivative when t = .
6
2.3. CHAPTER EXERCISES 47

VII. Given the set of parametric equations of a curve C defined by


h π
x = 4t + 7, y = 4 ln(cos t), t ∈ 0, .
2
1. Determine the points where C has a horizontal or vertical tangent line.
2. Find the corresponding Cartesian equation for C.
h πi
3. Find the arclength if t ∈ 0, .
4
ln t 1
VIII. Given the parametric curve x = ,y= where t ∈ (0, +∞) − {3}.
2 3−t
d2 y

1. Evaluate without eliminating the parameter.
dx2 t=2
1
2. Set up the integral(s) equal to the arclength of the curve from y = −1 to y = − .
2
IX. Consider the following set of parametric equations:

s
1 − t2 2t

ic
x= , y= , t∈R
1 + t2 1 + t2

at
em
1. Show that the above set of parametric equations traces out the unit circle centered at
h
the origin except a single point. What is this point?
at
2. Using another parametrization of the unit circle, derive the following equations:
M

1 − t2
of

2t 2dt
cos θ = , sin θ = , dθ =
1 + t2 1 + t2 1 + t2
e
ut

3. Using the above equations, evaluate the following integrals.


stit

Z Z Z
1 1
In

dθ dθ sec θdθ
2 + cos θ 1 + cos θ + sin θ
UP

The above substitution is known as Weierstrass substitution.

X. Consider two particles moving on the plane according to the following sets of parametric
equations:

C1 : x = 3 sin t, y = 2 cos t, 0 ≤ t ≤ 2π
C2 : x = 3 + cos t, y = 1 + sin t, 0 ≤ t ≤ 2π.

1. Find all intersection points of the two paths.


2. Determine all collision points of the two particles, that is, points at which both particles
are located at the same time.
3. Are intersection points necessarily collision points?
48 CHAPTER 2. PARAMETRIC CURVES

s
ic
at
hem
at
M
of
e
ut
stit
In
UP
Chapter 3

Polar Coordinate System

Let P be a point on the xy-plane. We know that the Cartesian coordinate system assigns an
ordered pair (x, y) to the point P where x and y are the directed distances of P from the y-axis
and the x-axis respectively.

ics
In this chapter, we introduce another way of assigning coordinates to points on a plane.

at
3.1 Basic Definitions h em
at
−−→
Consider a point O and a ray OK that co-
M

incide with the origin and the positive x-axis


−−→
of

respectively. We call O the pole and OK the


e

polar axis. The polar coordinate system


ut

assigns an ordered pair (r, θ) to a point P on


tit

the plane where


s
In

• r is the directed distance of the point P


UP

from the pole, and

• θ is an angle formed from the polar axis


−−→
to the ray OP .

We adapt the convention that a positive value for θ indicates counterclockwise measurement from
the polar axis, while a negative value for θ indicates a clockwise measurement from the polar axis.
Remark 3.1.1. Let P (r, θ) be a point in the polar plane.
• If r = 0, the point P is the pole.

• If r > 0, the point P is located r units from the pole on the terminal side of the angular
coordinate θ.

• If r < 0, the point P is located |r| units from the pole on the ray directed opposite of the
terminal side of the angular coordinate θ.

49
50 CHAPTER 3. POLAR COORDINATE SYSTEM

Let us plot some points on the polar plane.

Example 3.1.2. Plot the following points.


 π
1. P 1,
2
 

2. Q 3,
4
 π
3. R −2,
6
 

4. S −1,
2
 
13π
5. T −2,
6

As we have seen above, the point S coincides with the point P and the point R coincides with the

s
point T . This illustrates that the polar coordinates of a point is not unique. The following remark

ic
at
generalizes this fact.

em
Remark 3.1.3. Polar representation of a point is not unique. In fact, a point P (r, θ) may also
h
at
be represented by
M

(r, θ + 2kπ), k ∈ Z or (−r, θ + (2k + 1)π), k ∈ Z.


of
e

EXERCISES. Do as indicated.
ut
tit

1. Plot the following points on a single polar plane.


s
In
UP

 π  π
(a) 3, (d) 2, −
3 6
(b) (4, 0) (e) (−5, π)
 π
(c) −5, (f) (0, 1000π)
2
 π
2. Give equivalent polar coordinates (r, θ) of 4, that satisfies the following conditions:
6

 
(a) r > 0 and θ > 0 7π 11π
(f) r < 0 and θ ∈ ,
(b) r > 0 and θ < 0 2 2
 
(c) r < 0 and θ > 0 5π π
(g) r > 0 and θ ∈ − ,
3 3
(d) r < 0 and θ < 0
   
7π 11π 5π π
(e) r > 0 and θ ∈ , (h) r < 0 and θ ∈ − ,
2 2 3 3
3.2. CONVERSION FORMULAS 51

3.2 Conversion Formulas


Let P be point with Cartesian coordinates (x, y)
and polar coordinates (r, θ). From the definition
of the coordinate r, by the distance formula,

r 2 = x2 + y 2 .

Now, we construct a right triangle as seen in the


figure. By trigonometry, we have

x = r cos θ and y = r sin θ.

If one wants to transform a Cartesian coordinate (x, y) to polar coordinates (r, θ), choose r (re-
gardless of the sign) such that x2 + y 2 = r2 and then solve for the value of θ such that x = r cos θ
and y = r sin θ.

s
Example 3.2.1. Find a pair of polar coordinates for the point having Cartesian coordinates

ic
√ 

at
− 3, 1 with

h em
at
1. r > 0, θ > 0 ; 3. r < 0, θ > 0 ;
M

2. r > 0, θ < 0 ; 4. r < 0, θ < 0 .


of

√ 2
e
ut

Solution. Recall that r2 = x2 + y 2 . Thus, r2 = − 3 + 12 = 4.


tit


s

x 3 y 1 5π
In

1. Take r = 2. Then cos θ = = − and sin θ = = . So, we may take θ = . Hence, a


r 2  r 2 6

UP

polar representation of the point is 2, .


6

5π 7π
2. Take r = 2. We find a negative angle coterminal with . Take θ = − . Hence, a polar
  6 6

representation of the point is 2, − .
6

x 3 y 1 11π
3. Take r = −2. Then cos θ = = and sin θ = = − . So, we may take θ = . Hence,
r 2  r 2 6
11π
a polar representation of the point is −2, .
6

11π π
4. Take r = −2. We find a negative angle coterminal with . Take θ = − . So, we may take
π  6 π
6
θ = − . Hence, a polar representation of the point is −2, − .
6 6
52 CHAPTER 3. POLAR COORDINATE SYSTEM

Example 3.2.2. Convert the following pairs of polar coordinates to Cartesian coordinates.

 π 


1. 2, 2. −3, −
6 4

Solution.
π √ π
1. Using the conversion formulas, we have x = 2 cos = 3 and y = 2 sin = 1. Thus, the
√ 6 6
Cartesian coordinates are ( 3, 1).
  √   √
3π 3 2 3π 3 2
2. Using the conversion formulas, x = −3 cos − = and y = −3 sin − = .
4! 2 4 2
√ √
3 2 3 2
Thus, the Cartesian coordinates are , .
2 2

The conversion formulas can also be used to transform equations from polar form to rectangular

ic s
form and vice versa.

at
em
Example 3.2.3. Find the polar form of the following equations.
h
at
1. x = 2 2. xy = 1
M
of

Solution.
e
ut

1. Using the conversion formulas, we have r cos θ = 2 and thus, r = 2 sec θ since cos θ 6= 0.
tit

2. Note that xy 6= 0. Using the conversion formulas, we have r2 cos θ sin θ = 1. So r2 = sec θ csc θ.
s
In

Example 3.2.4. Find the rectangular form of the following equations:


UP

1. r = 2 4
2. r =
2 cos θ − sin θ

Solution.

1. Since x2 + y 2 = r2 , we get x2 + y 2 = 4.

2. We rewrite the given equation as 2r cos θ − r sin θ = 4. Thus, 2x − y = 4.

EXERCISES. Do as indicated.

1. Give the Cartesian coordinates of the following points in polar coordinates.

(a) (3, π4 ) (3.) (4, π)



(b) (−5, π2 ) (4.) ( 2, − π6 )
3.3. POLAR CURVES 53

2. Find the polar coordinates of the following points in Cartesian coordinates,


√ √
(a) ( 2, 2) (3.) (0, 0)

(b) (−1, 3) (4.) (0, −4)

3. Find the equivalent Cartesian equation of the following polar equations. Sketch the graph of
each curve.

2π (d) r cos θ + 6 = 0
(a) θ =
3
2π (e) r2 − 8r cos θ − 4r sin θ + 11 = 0
(b) θ = −
3 4
(c) r = 5 and r = −5 (f) r =
1 − cos θ

3.3 Polar Curves

ic s
In this section we consider graphs of polar equations. The graph of a polar equation is a curve on

at
the polar plane consisting of all points (r, θ) that satisfy the equation.

3.3.1 Lines and Circles in Polar Form


hem
at
M

Recall that in Cartesian coordinates, the graph of the equation x = a, where a is a constant, is the
of

line parallel to the y-axis through the point (a, 0). On the other hand, the graph of y = b, where b
is a constant, is the line parallel to the x-axis through the point (0, b).
e
ut
tit

Analogously, we shall first consider the graph of r = k or θ = k, where k is a constant.


s
In

1. Consider the polar equation r = k, where k is a nonzero constant. Suppose that a point P
UP

with Cartesian coordinates (x, y) lies on the graph of r = k. Using the conversion formula
x2 + y 2 = r2 , we get the corresponding Cartesian equation x2 + y 2 = k 2 . Therefore, r = k
represents a circle centered at the pole of radius |k|. Note that the polar equations r = k and
r = −k represent the same circle.

2. Consider the polar equation θ = θ0 , where θ0 is a constant. Let P with Cartesian coordinates
y
(x, y) lie on the graph of θ = θ0 . Since x = r cos θ and y = r sin θ, we have = tan θ = tan θ0 .
x
Thus, the polar curve has corresponding Cartesian equation y = (tan θ0 )x, which represents
a line through the pole that makes an angle θ0 radians with the polar axis.

Let us look at some particular examples.

Example 3.3.1. Sketch the graph of r = 3.

Solution. By the first generalization above, r = 3 represents a circle centered at the pole of radius
3, as shown in the figure below.
54 CHAPTER 3. POLAR COORDINATE SYSTEM

π
Example 3.3.2. Sketch the graph of θ = .
4
π
Solution. By the second generalization, θ = represents a line through the pole that makes an
4
π
angle radians with the polar axis, as shown below. In fact, its Cartesian equation is given by
4
y = x.

ic s
at
h em
at
M
of
e
ut
s tit
In

In Cartesian coordinates, constructing lines with equations x = a and y = b for varying values
UP

of a and b enables us to partition the Cartesian plane into rectangles, as shown in the left figure
below. Thus, we also use the term rectangular coordinates to refer to the Cartesian coordinates.
In a similar manner, in polar coordinates, by constructing circles r = k and lines θ = θ0 for varying
values of k and θ0 , the plane is partitioned into polar grids, as shown in the right figure below. The
polar grids makes it easier for us to plot points and polar curves in the polar plane.
3.3. POLAR CURVES 55

Example 3.3.3. Sketch the graph of r = 4 cos θ.

Solution. To sketch the polar curve, we consider several points on the curve by assigning values for
θ and solving for the corresponding values of r. We consider the following points.

θ r θ r

0 4 −2
3
π √ 3π √
2 3 2 2
6 4
π √ 5π √
2 2 −2 3
4 6
π
2 π −4
3
π
0
2

ic s
Example 3.3.4. Sketch the graph of r = 6 sin θ.

at
em
Solution. As we did in the previous example, we consider the following points.
h
at
M

θ r θ r
of

2π √
0 0 3 3
e

3
ut

π 3π √
tit

3 3 2
6 4
s

π √ 5π
In

3 2 3
4 6
UP

π √
3 3 π 0
3
π
6
2

We generalize the circles described in Examples 3.3.3 and 3.3.4.

Remark 3.3.5.
π
1. The graph of the equation r = 2a cos θ is a circle of radius |a| tangent to the -axis at the
2
pole. Moreover, the polar coordinates of the center of this circle is (a, 0). This circle has a
Cartesian equation (x − a)2 + y 2 = a2 .

2. The graph of the equation r = 2a sin θ is a circle of radius |a| tangent


 toπthe
 polar axis at the
pole. Moreover, the polar coordinates of the center of this circle is a, . This circle has a
2
Cartesian equation x2 + (y − a)2 = a2 .
56 CHAPTER 3. POLAR COORDINATE SYSTEM
h π πi
These circles are traced exactly once as θ varies on [0, π] or − , .
2 2
Remark 3.3.6.

1. The vertical line with Cartesian equation x = a has a polar equation r = a sec θ.

2. The horizontal line with Cartesian equation y = a has a polar equation r = a csc θ.

3.3.2 Symmetry in the Polar Plane


Recall that a curve with equation y = f (x) is symmetric with respect to

• the x-axis if an equivalent equation is obtained when y is replaced by −y;

• the y-axis if an equivalent equation is obtained when x is replaced by −x; and

• the origin if an equivalent equation is obtained when x and y are replaced by −x and −y,
respectively.

ic s
Furthermore, if a curve is symmetric with respect any two of the following: x-axis, y-axis, origin,

at
then the curve is symmetric with respect to all three.
h em
We now consider the analogues of these tests in the polar coordinate system.
at
M

Symmetry About the Polar Axis


of
e
ut

A polar curve is symmetric about the polar


tit

axis whenever an equivalent equation is obtained


s

when (r, θ) is replaced by


In
UP

(r, −θ) or (−r, π − θ).

π
Symmetry About the -Axis
2
π
A polar curve is symmetric about the -axis
2
whenever an equivalent equation is obtained
when (r, θ) is replaced by

(r, π − θ) or (−r, −θ).


3.3. POLAR CURVES 57

Symmetry About the Pole


A polar curve is symmetric about the pole when-
ever an equivalent equation is obtained when
(r, θ) is replaced by

(−r, θ) or (r, θ + π).

Example 3.3.7. Test for the symmetries of the polar curve r2 = 2 cos θ.

Solution.

• Symmetry about the polar axis:


When (r, θ) is replaced by (r, −θ), we get

s
r2 = 2 cos(−θ) which gives r2 = 2 cos θ.

ic
at
An equivalent equation is obtained. Hence, the curve is symmetric about the polar axis.
π
• Symmetry about the -axis:
2
h em
at
When (r, θ) is replaced by (r, π − θ), we get
M

r2 = 2 cos(π − θ) which gives r2 = −2 cos θ.


of
e

The obtained equation is not equivalent to the given. The test fails. When (r, θ) is replaced
ut

by (−r, −θ), we get


tit

(−r)2 = 2 cos(−θ) ⇒ r2 = 2 cos θ.


s
In

π
An equivalent equation is obtained, hence, the curve is symmetric about the -axis.
2
UP

• Symmetry about the pole:


When (r, θ) is replaced by (−r, θ), we get

(−r)2 = 2 cos θ which gives r2 = 2 cos θ,

which is equivalent to the given equation. Thus, the curve is symmetric about the pole.

r2 = 2 cos θ
58 CHAPTER 3. POLAR COORDINATE SYSTEM

π
Remark 3.3.8. If a polar curve is symmetric about two of the three: polar axis, -axis, pole, then
2
it is symmetric with respect to all three.

EXERCISES. Do as indicated.

1. Sketch the following curves.

(a) r = 5 (d) r = 3 sin θ

(b) r = 6 cos θ (e) r = −2 sin θ



(c) r = − cos θ (f) θ =
4

2. Determine the symmetry of the following polar curves.

(a) r = θ (d) r2 = sin(2θ)


2
(b) r = (e) r = 3 sin 2θ

s
1 − cos θ

ic
4
(f) r2 =
p

at
(c) r = cos(2θ) cos2 θ + 4 sin2 θ

em
3. Give two different polar equations r = f (θ) and r = g(θ) having the same graph. Can this
h
at
happen in the rectangular coordinates?
M
of

3.4 Special Polar Curves


e
ut

3.4.1 Limaçons
tit

Definition 3.4.1. The graph of an equation of the form r = a ± b cos θ or r = a ± b sin θ, where
s
In

a > 0, b > 0, is called a limaçon.


UP

Let us look at some properties of limaçons.

Remark 3.4.2.

1. A limaçon is traced exactly once as θ varies on [0, 2π].

2. Testing for symmetry, we have the following:

• r = a ± b cos θ : Replacing (r, θ) by (r, −θ), we get

r = a ± b cos(−θ) which is equivalent to r = a ± b cos θ.

Thus, it is symmetric with respect to the polar axis.


• r = a ± b sin θ : Replacing (r, θ) by (r, π − θ), we get

r = a ± b sin(π − θ) which is equivalent to r = a ± b sin θ.


π
Thus, it is symmetric with respect to the -axis.
2
3.4. SPECIAL POLAR CURVES 59

Example 3.4.3. Sketch the graph of r = 1 + 2 cos θ.

Solution. We consider the following points.

θ r θ r θ r
π 3π √
0 3 2 1− 2
3 4
π √ π 5π √
1+ 3 1 1− 3
6 2 6
π √ 2π
1+ 2 0 π −1
4 3
Then, we apply symmetry to complete the graph.
a
The graph is called a limaçon with a loop. Note that 0 < < 1.
b

Example 3.4.4. Sketch the graph of r = 1 + cos θ.

ics
Solution. We consider the following points.

at
θ r θ r θ r
√ em
h
at
π 3 3π 2
0 2 1−
M

√ 3 2 4 √2
π 3 π 5π 3
of

1+ 1 1−
6 √2 2 6 2
e

π 2 2π 1
ut

1+ π 0
4 2 3 2
stit

Then, we apply symmetry to complete the graph.


In

a
The graph is called a cardioid. Note that = 1.
UP

Example 3.4.5. Sketch the graph of r = 3 + 2 cos θ.

Solution. We consider the following points.

θ r θ r θ r
π 3π √
0 5 4 3− 2
3 4
π √ π 5π √
3+ 3 3 3− 3
6 2 6
π √ 2π
3+ 2 2 π 1
4 3
Then, we apply symmetry to complete the graph.
a
The graph is called a limaçon with a dent. Note that 1 < < 2.
b
60 CHAPTER 3. POLAR COORDINATE SYSTEM

Example 3.4.6. Sketch the graph of r = 2 + cos θ.

Solution. We consider the following points.

θ r θ r θ r

π 5 3π 2
0 3 2−
√ 3 2 4 √2
π 3 π 5π 3
2+ 2 2−
6 √2 2 6 2
π 2 2π 3
2+ π 0
4 2 3 2
Then, we apply symmetry to complete the graph.
a
The graph is called a convex limaçon. Note that ≥ 2.
b

Types of Limaçons

ic s
at
In general, for r = a ± b cos θ or r = a ± b sin θ, with a, b > 0, the polar curve is a

a
em
h
at
• limaçon with a loop if 0 < < 1.
M

b
of

a
• cardioid if = 1.
e

b
ut
tit

a
• limaçon with a dent if 1 < < 2.
s
In

b
UP

a
• convex limaçon if ≥ 2.
b

The graph of r = a±b cos θ is a limaçon oriented horizontally, i.e. symmetric about the polar axis.

r = a + b cos θ r = a − b cos θ
3.4. SPECIAL POLAR CURVES 61

π
The graph of r = a ± b sin θ is a limaçon oriented vertically, i.e. symmetric about the -axis.
2

r = a + b sin θ r = a − b sin θ

3.4.2 Roses

ic s
Definition 3.4.7. The graph of an equation of the form r = a cos nθ or r = a sin nθ, where a > 0

at
and n ∈ N, is called a rose.

Example 3.4.8. Here are some examples of roses.


h em
at
M
of
e
ut
stit
In
UP

r = 2 sin 3θ r = 2 cos 2θ

r = 3 cos 9θ r = 3 sin 4θ
62 CHAPTER 3. POLAR COORDINATE SYSTEM

Let us look at some properties of roses.

Remark 3.4.9.

1. A rose has n petals if n is odd, and 2n petals if n is even. Each petal of a rose is of
length/radius a.

2. The following can be shown using the symmetry tests:


π
• If n is even, the rose is symmetric about the polar axis, the -axis and the pole.
2
• If n is odd, r = a cos nθ is symmetric about the polar axis and r = a sin nθ is symmetric
π
about the -axis.
2
3. A rose with an odd number of petals is traced exactly once as θ varies on [0, π]. A rose with
an even number of petals is traced exactly once as θ varies on [0, 2π].

4. To get the graph of r = −a cos nθ (or r = −a sin nθ), we reflect each petal of the graph of
r = a cos nθ (or r = a sin nθ) about the pole.

ic s
Here are useful tips on how to sketch a rose. Try to look for the following:

at
em
• Tangent lines to the rose. The tangent lines to the rose at the pole are the lines which
squeeze the petals. To get these lines, solve the equation r = 0, θ ∈ [0, 2π].
h
at
• Tips of the petals of the rose. The tips of the petals are the points which have the longest
M

distance from the pole. To locate the tips of the petals, solve the equation r = a if n is odd,
of

or r = |a| if n is even, θ ∈ [0, 2π].


e
ut

Let us look at some examples.


tit
s

Example 3.4.10. Sketch the graph of r = 2 sin 3θ.


In

Solution. The rose has 3 petals, each of radius 2.


UP

π
It is symmetric about the -axis. First, we plot
2
a portion of the rose using the following points.

θ r θ r
π
0 0 0
3
π √ 5π √
2 − 2
12 12
π π
2 −2
6 2
π √
2
4
Then, we apply the symmetry of the rose. We
π
reflect the portion about the -axis to complete
2
the graph.
3.4. SPECIAL POLAR CURVES 63

Another way is to use the tip stated above. Note that the tangent lines to the rose at the pole are
π 2π π 5π 3π
θ = 0, , and the tips of the petals are located at θ = , , .
3 3 6 6 2

Example 3.4.11. Sketch the graph of r = 2 cos 2θ.

Solution. The rose has 2(2) = 4 petals, each of


π
radius 2. It is symmetric about the -axis and the
2
pole. First, we plot a portion of the rose using the
following points.

θ r θ r
3π √
0 2 − 2
8
π √ π
2 −2

s
8 2

ic
π

at
0
4
Now, we apply the symmetry of
em
the rose. we re-
h
at
flect the portion about the pole. Then we reflect
π
M

the resulting portion about the -axis to com-


2
of

plete the rose.


e

π 3π 5π 7π
ut

Note that the tangent lines to the rose at the pole are θ = , , , and the tips of the petals
4 4 4 4
tit

π 3π
are located at θ = 0, , π, .
s

2 2
In
UP

3.4.3 Other Polar Curves

The graph of an equation of the form r2 = k cos 2θ or r2 = k sin 2θ, where k is a nonzero real
number, is called a lemniscate.

r2 = 9 cos 2θ r2 = 9 sin 2θ
64 CHAPTER 3. POLAR COORDINATE SYSTEM

Spirals comprise of curves whose r grows indefinitely as θ varies over R and hence, they are not
periodic like the ones we have seen on the previous examples.

r=θ r = eθ/3

EXERCISES. Identify the graph of the given polar equation. Give the symmetry of the curve. If
the polar curve is a rose, give the number of petals of the rose. Sketch its graph on a polar plane.

s
1. r = 3 + 3 sin θ 6. r = 2 − 5 cos θ 11. r = −4 + cos θ

ic
at
2. r = 1 − 5 sin θ 7. r = −5 + cos θ 12. r = 4 cos 2θ

3. r = 2 + sin θ 8. r = −6 − 5 cos θ
h em 13. r = −2 cos 2θ
at
M

4. r = 4 − 3 sin θ 9. r = −2 + 4 cos θ 14. r = 3 sin 3θ


of

5. r = 3 + 3 cos θ 10. r = 4 − 3 sin θ 15. r = −3 sin 3θ


e
ut
tit

3.5 Calculus of Polar Curves


s
In

Given a polar curve r = f (θ), we can express x and y as a function of θ using the conversion
UP

formulas x = r cos θ and y = r sin θ. Thus, we can think of the polar curve as a parametric curve
with the following equations:

x = f (θ) cos θ and y = f (θ) sin θ.

3.5.1 Tangent Line to a Polar Curve


Recall that the slope of a parametric curve C : x = x(θ), y = y(θ) is given by

dy dy/dθ
= .
dx dx/dθ

Thus, if r = f (θ), where f is a differentiable function of θ,


dx dy
= f 0 (θ) cos θ − f (θ) sin θ and = f 0 (θ) sin θ + f (θ) cos θ.
dθ dθ
Hence, we have the following result.
3.5. CALCULUS OF POLAR CURVES 65

dy dx
Theorem 3.5.1 (Tangent Line to a Polar Curve). Given that and are continuous and
dθ dθ
dx
6= 0, the slope of the tangent line to the polar curve r = f (θ) at a point (r, θ) is

 
dr
(sin θ) + r cos θ

m=   .
dr
(cos θ) − r sin θ

Remark 3.5.2.

1. A polar curve may have several tangent lines at a point.


dr
2. The derivative does not give the slope of the tangent line to the curve defined by r = f (θ).

Example 3.5.3. Find the (Cartesian) equation of the tangent line to the limaçon r = 3 − 2 cos θ
π
at the point where θ = .
3

ic s
Solution. First, we look for the Cartesian coordinates of the point of tangency. Note that

at
π

em
r = 3 − 2 cos
= 2.
3 h
at
 π
Thus, the point of tangency has polar coordinates 2, . We then have
M

3
π
of

=1
x = r cos θ = 2 cos
3
e

π √
ut

y = r sin θ = 2 sin = 3
3
tit


s

and so the point of tangency in Cartesian coordinates is (1, 3).


In

Moreover,
UP

dr √
= 2 sin θ = 3.

dθ θ=π/3

θ=π/3

The slope is given by


√ !
3 √
 
1
( 3) + 2
2 2
m =  √ !

(2,π/3) 1 √ 3
( 3) − 2
2 2

5 5 3
= −√ = − .
3 3

Hence, the tangent line has equation



√ 5 3
y− 3=− (x − 1).
3
66 CHAPTER 3. POLAR COORDINATE SYSTEM

Example 3.5.4. Determine all points on the cardioid r = 1 − cos θ where the tangent lines are
horizontal; vertical.

Solution. The slope at any point (r, θ) is given by

 
dr
(sin θ) + r cos θ

m=  
dr
(cos θ) − r sin θ

sin θ sin θ + (1 − cos θ) cos θ
=
cos θ sin θ − (1 − cos θ) sin θ
sin2 θ + cos θ − cos2 θ
= .
2 cos θ sin θ − sin θ

If the numerator is zero, we have

ic s
at
sin2 θ + cos θ − cos2 θ = 0

em
1 + cos θ − 2 cos2 θ = 0h
(1 − cos θ)(1 + 2 cos θ) = 0.
at
M

2π 4π
of

This implies either cos θ = 1 or cos θ = − 12 , which gives θ = 0, , , for θ ∈ [0, 2π).
3 3
e
ut

On the other hand, if the denominator is zero, we have


s tit
In

2 cos θ sin θ − sin θ = 0


sin θ(2 cos θ − 1) = 0.
UP

π 5π
This means either sin θ = 0 or cos θ = 21 , which imples that θ = 0, π, , for θ ∈ [0, 2π).
3 3
dy dx
Recall that for horizontal tangent lines, = 0 and 6= 0. Therefore, the cardioid has horizontal
dθ dθ    
2π 4π 3 2π 3 4π
tangent lines at points where θ = , , i.e., at the points , and , . For vertical
3 3 2 3 2 3
dy dx
tangent lines, 6= 0 and = 0. Hence, the cardioid has vertical tangent lines at points where
dθ dθ    
π 5π 1 π 1 5π
θ = π, , , i.e., at the points (2, π), , and , .
3 3 2 3 2 3

Observe that both the numerator and denominator are zero when θ = 0. Geometrically, the polar
curve makes a sharp turn when θ = 0. Just as we defined in item 3 of Remark 2.2.5, we compute
the limit of m as θ → 0, if it exists.
3.5. CALCULUS OF POLAR CURVES 67

sin2 θ + cos θ − cos2 θ (1 − cos θ)(1 + 2 cos θ)


lim = lim
θ→0 2 cos θ sin θ − sin θ θ→0 sin θ(2 cos θ − 1)
(1 − cos θ)(1 + 2 cos θ) 1 + cos θ
= lim ·
θ→0 sin θ(2 cos θ − 1) 1 + cos θ
2
(1 − cos θ)(1 + 2 cos θ)
= lim
θ→0 sin θ(1 + cos θ)(2 cos θ − 1)
sin θ(1 + 2 cos θ)
= lim
θ→0 (1 + cos θ)(2 cos θ − 1)
0·3
=
2·1
= 0.

Hence, the cardioid also has horizontal tangent line at θ = 0, i.e., at the point (0, 0).

3.5.2 Arc length of a Polar Curve

ic s
Consider a parametric curve C := x(t), y = y(t), where t ∈ [a, b]. Recall from Section 2.2.10 that

at
em
if C is smooth and is traced exactly once on [a, b], then the length of C is given by
s
h
at
Z b 2  2
dx dy
L= + dt.
M

a dt dt
of

Suppose C is a smooth polar curve with equation r = f (θ) traced exactly once when θ ∈ [α, β].
e
ut

Then C is parametrized by
tit

x = f (θ) cos θ and y = f (θ) sin θ.


s
In

Differentiating each side of the equations with respect to θ, we have


UP

 
dx 0 dr
= f (θ) cos θ − f (θ) sin θ = cos θ − r sin θ,
dθ  dθ 
dy dr
= f 0 (θ) sin θ + f (θ) cos θ = sin θ + r cos θ.
dθ dθ

Taking the sum of the squares of the derivatives, we get


 2  2
dx dy
+ = [f 0 (θ)]2 cos2 θ − 2f 0 (θ)f (θ) cos θ sin θ + [f (θ)]2 sin2 θ
dθ dθ
[f 0 (θ)]2 sin2 θ + 2f 0 (θ)f (θ) cos θ sin θ + [f (θ)]2 cos2 θ

= [f 0 (θ)]2 + [f (θ)]2
 2
2 dr
=r + ,

which leads to the following formula.


68 CHAPTER 3. POLAR COORDINATE SYSTEM

Theorem 3.5.5 (Arc length of a Polar Curve). If a smooth polar curve C : r = f (θ) is traced
exactly once as θ varies from α to β, then the length of C is
s  2
Z β
dr
L= r2 + dθ.
α dθ

Example 3.5.6. Find the length of the cardioid r = 1 + cos θ.

Solution. Since the cardioid is symmetric about the polar axis, we just consider θ ∈ [0, π] and then
we multiply the result by 2. Hence,
Z πq
L=2 (f (θ))2 + (f 0 (θ))2 dθ
Z0 π q
=2 (1 + cos θ)2 + (− sin θ)2 dθ
Z0 π

=2 2 + 2 cos θ dθ
0
Z πs  
θ

s
=2 4 cos 2 dθ

ic
0 2

at
Z π  
θ

em
=4 cos dθ
0 2 h
 
θ π
at
= 8 sin
2 0

M

= 8.
of

Therefore, the arc length of the cardioid is 8 units.


e
ut

Example 3.5.7. Find the arc length of the circle r = 4 cos θ.


tit
s

Solution. Recall that this translated circle is traced exactly once on [0, π]. Therefore,
In

Z πp
UP

L= (4 cos θ)2 + (−4 sin θ)2 dθ


0
Z πp
= 16 cos2 θ + 16 sin2 θ dθ
Z0 π
= 4 dθ
0
= 4π.

Thus, the arc length of the circle is 4π units, which is indeed the circumference of a circle of radius
2 units.

Example 3.5.8. Set up the integral that yields the length of one petal of the rose r = sin 2θ.

Solution. Note that one petal of the given rose is traced from θ = 0 to θ = π2 . Using the formula,
we get
Z π/2 p
L= (sin 2θ)2 + (2 cos 2θ)2 dθ.
0
3.5. CALCULUS OF POLAR CURVES 69

Alternatively, by symmetry,
Z π/4 p
L=2 (sin 2θ)2 + (2 cos 2θ)2 dθ.
0

3.5.3 Area of a Polar Region


In this section, we shall find a formula to compute areas of regions enclosed by polar curves.
Consider the region bounded by the lines θ = α and θ = β, with α ≤ β ≤ α + 2π, and the polar
curve r = f (θ), where f is continuous on [α, β], as shown in the figure below. We wish to find the
area A of this region.

ic s
at
hem
at
M
of

Figure 3.1: Region bounded by r = f (θ), θ = α and θ = β


e
ut

To do this, we partition the region into n wedges using the rays θ = θ1 , θ = θ2 , . . ., θ = θn−1 , for
tit

some α < θ1 < θ2 < . . . < θn−1 < β. Let ∆θ1 , ∆θ2 , . . ., ∆θn be the central angles of the wedges.
s
In

Denote by A1 , A2 , . . ., An the areas of the wedges. We approximate the area Ai of the ith wedge
using a sector with central angle ∆θi , and radius f (θi∗ ), where θi∗ ∈ [θi−1 , θi ], as shown below.
UP

Figure 3.2: Sector with central angle ∆θi , and radius f (θi∗ )
70 CHAPTER 3. POLAR COORDINATE SYSTEM

Recall that the area AS of a circular sector of radius r and central angle θ (measured in radians)
is given by
1
AS = r2 θ.
2

So, Ai ≈ 1
2 (f (θi∗ ))2 ∆θi , and thus,

n n
X X 1
A= Ai ≈ (f (θi∗ ))2 ∆θi .
2
i=1 i=1

Error in the approximation vanishes if we let n → +∞. That is,

n Z β
X 1 1
A = lim [f (θi∗ )]2 ∆θi = [f (θ)]2 dθ.
n→∞ 2 α 2
i=1

Theorem 3.5.9 (Area of a Polar Region). Suppose C : r = f (θ) is continuous on [α, β], where

s
α ≤ β ≤ α + 2π. The area of the region enclosed by C and the terminal sides of θ = α and θ = β

ic
is given by

at
em
1 β 1 β 2
Z Z
2
A= [f (θ)] dθ = r dθ.
2 α 2 α h
at
M

Remark 3.5.10.
of

1. The limits of integration are determined by the radial lines, or rays, that bound the region.
e
ut
tit

2. Most of the time, it is helpful to take advantage of the symmetry of the given polar region.
s
In

Example 3.5.11. Find the area of the region in the first quadrant inside the cardioid r = 1 − cos θ.
UP

π
Solution. Here, θ must vary from θ = 0 to θ = to sweep out the region. Thus,
2
Z π/2
1
A= (1 − cos θ)2 dθ
2 0
Z π/2
1
= (1 − 2 cos θ + cos2 θ) dθ
2 0
1 π/2 3
Z  
1
= − 2 cos θ + cos 2θ dθ
2 0 2 2
  π/2
1 3 1
= θ − 2 sin θ + sin 2θ
2 2 4 0

= − 1.
8

Therefore, the area of the region is − 1 square units.
8
3.5. CALCULUS OF POLAR CURVES 71

Example 3.5.12. Find the area of the region enclosed by the petals of the rose r = 3 cos 2θ.

Solution. We first note that the graph of r = 3 cos 2θ is symmetric about the polar axis. We make
use of its symmetry and just set up the definite integral equal to the area of half a petal, then
multiply by 8. In particular, consider the upper-half of the right petal. Here, θ must vary from
π
θ = 0 to θ = . We then have
4

1 π/4
Z
A=8· (3 cos 2θ)2 dθ
2 0
Z π/4
9
=4 (1 + cos 4θ) dθ
0 2
Z π/4
= 18 (1 + cos 4θ)dθ
0
  π/4
1
= 18 θ + sin 4θ
4 0

= .

s
2

ic

at
Therefore, the area of the region is square units.
2
hem
To find the area of a polar region bounded by two polar curves, one can solve for the points of
at
intersection of these polar curves. In most cases, the points of intersection help us determine the
M

values of the limits of the area integral. Let us look at an example.


of

Example 3.5.13. Find the points of intersection of the limaçons r = 3 − 2 sin θ and r = 6 + 4 sin θ.
e
ut

Solution. We equate r from the two given equations.


tit
s
In

3 − 2 sin θ = 6 + 4 sin θ
UP

1
sin θ = −
2
7π 11π
θ= , .
6 6

7π 11π
Substituting θ = or to r = 3 − 2 sin θ,
6 6
we get r = 4. Thus, the
 points
 of intersection
  in
7π 11π
polar coordinates are 4, and 4, .
6 6

Now, let us apply this concept on the next examples.

Example 3.5.14. Find the area of the region inside the cardioid r = 4 + 4 cos θ and outside the
circle r = 6.

Solution. The two curves are both symmetric about the polar axis.
72 CHAPTER 3. POLAR COORDINATE SYSTEM

We make use of the symmetry of the curves


(which is also the symmetry of the region) by
finding the area of the portion of the region in
the first octant and multiply by 2. So, θ must
vary from θ = 0 to θ = α, as shown in the figure
on the right. Solving for α from the intersection
of the two curves,
4 + 4 cos θ = 6
1
cos θ =
2
π 5π
θ= , .
3 3
π
We take α = . Then,
3 " Z #
1 π/3 1 π/3 2
Z
2
A=2 (4 + 4 cos θ) dθ − (6) dθ
2 0 2 0

s
ic
Z π/3
= (16 + 32 cos θ + 16 cos2 θ − 36)dθ

at
0

em
Z π/3
= (16 cos2 θ + 32 cos θ − 20)dθ
h
0
at

= 18 3 − 4π.
M


Therefore, the area of the region is 18 3 − 4π square units.
of
e

Example 3.5.15. Set up the integrals that will give the area of the region inside both the circle
ut

r = 5 sin θ and the limaçon r = 2 + sin θ.


stit

Solution. Let us first solve for the intersection points between the two curves. Equating the radial
In

coordinates in the given equations, we get


UP

5 sin θ = 2 + sin θ
1
sin θ =
2
π 5π
θ= , .
6 6
π 5π 5
Substituting θ = , to r = 5 sin θ, we get r = .
6 6    2

5 π 5 5π
The points of intersection are , and , .
2 6 2 6

Observe that the region is symmetric about the π2 -axis. Using this symmetry, we may consider the
portion in the first octant and multiply its area by 2. We have
" Z #
1 π/6 1 π/2
Z
2 2
A=2 (5 sin θ) dθ + (2 + sin θ) dθ .
2 0 2 π/6
3.5. CALCULUS OF POLAR CURVES 73

Example 3.5.16. Consider the region inside the circle r = 4 cos θ but outside the rose r = 4 cos 3θ.
Set up the integrals that will give the area of the region.

Solution. From the figure above, observe that the graphs may seem to intersect at the points where
π π
θ = 0, . But, the rose and the circle have different values of r at θ = 0, . Thus, the best way to
2 2
π
find the area is to treat the regions separately. Note that the semicircle varies from θ = 0 to θ =

s
2

ic
π
while the half-petal varies from θ = 0 to θ = . Thus, with the symmetry, we have

at
6

em
" Z #
1 π/2
Z π/6
1
(4 cos θ)2 dθ − (4 cos 3θ)2 dθ .
A=2
h
at
2 0 2 0
M

Remark 3.5.17. Intersection points of polar curves are naturally obtained by equating the ex-
of

pressions for r. However, this method may not always produce all the intersection points because
e

of the many ways of representing a point in polar coordinates.


ut
tit

Consider the cardioids r = 1 − cos θ and r = 1 + cos θ.


s
In
UP

Equating the expressions for r, we get

1 − cos θ = 1 + cos θ
cos θ = 0
π
θ = (2k + 1) , k ∈ Z.
2
74 CHAPTER 3. POLAR COORDINATE SYSTEM
 
π
 3π
These values of θ give us only two points 1, and 1, . Clearly, the pole has been missed
2 2
out. This is because the cardioids pass through the pole at different values of θ. In general, it is a
good idea to graph the curves to determine exactly how many intersections points should there be.

Example 3.5.18. Find the points of intersection of C1 : r = 2 sin 2θ and C2 : r = 1.

Solution. It can be verified from their graphs that C1 and C2 intersect at eight points. If we equate
the expressions for r, we get

2 sin 2θ = 1
1
sin 2θ =
2
π 5π 13π 17π
2θ = , , ,
6 6 6 6
π 5π 13π 17π
θ= , , , .
12 12 12 12
 π
These values of θ give the points 1, ,

s
12

ic
     
5π 13π 17π

at
1, , 1, and 1, .
12 12 12

We obtained only four points. How can we get the other four?
hem
at
Note that C2 is also represented by r = −1. So,
M
of

2 sin 2θ = −1
1
e

sin 2θ = −
ut

2
tit

7π 11π 19π 23π


2θ = , , ,
s

6 6 6 6
In

7π 11π 19π 23π


θ= , , , .
UP

12 12 12 12
       
7π 11π 19π 23π
Thus, we get the other four points −1, , −1, , −1, and −1, .
12 12 12 12

EXERCISES. Do as indicated.

1. Give the slope of the tangent line to the graph of the given polar equation at the specified
angle.
π
(a) r = 4 cos θ at θ =
6
(b) r = 1 + sin θ at θ = 0
π
(c) r = 4 − 3 cos θ at θ =
2
π
(d) r2 = sin θ at θ =
3
2 π
(e) r = at θ =
1 + cos θ 2
3.5. CALCULUS OF POLAR CURVES 75

π
(f) r = 7 cos 3θ at θ =
6

2. Give a sketch of the region being described in each item. Provide an expression involving
integrals that gives the area and the perimeter of the region. In the event that the region is
bounded by some polar curves, give the intersections of the polar curves.

(a) The region inside r = 1 + cos θ.

(b) The region inside the loop of r = 2 − 4 sin θ.

(c) The region outside r = 2 and inside r = 4 sin 2θ.

(d) The region inside r = 3 sin θ and outside r = 1 + sin θ.



(e) The region inside both r = 2 cos θ and r = sin θ + cos θ.

ic s
3. Let R be the region inside the rightmost petal of the rose r = 2 cos 2θ and outside the circle

at
r = 1.
h em
at
(a) Set up the integrals that give the perimeter
M

of R.
of

(b) Set up the integrals that give the area of R.


e
ut

(c) Set up the integrals that give the area of the


tit

region outside circle but inside the rose.


s
In
UP

4. Given the cardioid r = 1 + sin θ and the circle r = 2 sin θ, consider the region R inside the
cardioid but outside the circle.

(a) Aside from the pole, determine the other


points of intersection of the cardioid and the
circle.

(b) Set up the integral that gives the area of the


region.

(c) Set up the integral that gives the perimeter


of the region.

5. Let C1 be the limaçon r = 3 − 2 cos θ and C2 be the circle r = 4 cos θ.


76 CHAPTER 3. POLAR COORDINATE SYSTEM

(a) Find the Cartesian equation of the tangent


line to C1 at the point corresponding to θ =
π
.
2
(b) Find the points of intersection of C1 and C2 .

(c) Set up the integrals that give the area of the


shaded region.

(d) Set up the integrals that give the perimeter


of the upper half of the shaded region.

6. Given the polar curves C1 : r = 2 cos 3θ and C2 : r = 2.
(a) Determine C1 ∩ C2 .

(b) Set up an integral that gives the arclength


of a petal of C1 .

s
ic
(c) Set up the integrals that give the area of the

at
shaded region.

(d) Set up the integrals that give the perimeter


h em
at
of the shaded region.
M

1 β
Z
of

7. Show that if [f (θ)]2 dθ = 0 then f = 0 on [α, β]. Is the same true for functions in
2 α
e

Cartesian form?
ut

1 β
tit

Z
8. Give an example in which [f (θ)]2 dθ is twice the actual area of the region bounded by
s

2 α
In

r = f (θ) and the lines θ = α, θ = β. Is there such an example in the Cartesian plane?
UP

3.6 Chapter Exercises


I. Find polar coordinates for each point in rectangular coordinates with a positive radial coor-
dinate; a negative radial coordinate.

√ 
1. −6, 2 3 2. (5, −5)

II. Find the rectangular coordinates of the following points in polar form.

1. −3, − 5π 2. 12, − 2π
 
4 3

√ √
III. Given the polar curve r = 2 cos θ + 2 sin θ, find the Cartesian equation of the tangent line
π
to the curve at the point corresponding to θ = .
4
3.6. CHAPTER EXERCISES 77

IV. Consider the cardioid r = 2 + 2 cos θ.

1. Find the polar coordinates where the tangent lines are horizontal and vertical.
2. Does it have a vertical tangent line at θ = π? Justify.

V. Consider the polar curves C1 : r = 3 + 2 sin θ and C2 : r = 8 sin θ.

1. Find the Cartesian equation of the tangent


line to C1 at the point where θ = 0.

2. Give polar coordinates for the points of in-


tersection of C1 and C2 .

3. Setup the integrals that will yield the area


A and perimeter P of the shaded region.

4 1

s
VI. Let R be the region inside the circle r = − sin θ and above the parabola r =

ic
.
3 1 − sin θ

at
1. Find the Cartesian equation of the tangent

em

line to the parabola at θ = .
2 h
at
2. Find the intersection points of the two
M

curves.
of

3. Set up a definite integral that will give the


e
ut

area of R.
tit

4. Set up a definite integral that will give the


s
In

perimeter of R.
UP
78 CHAPTER 3. POLAR COORDINATE SYSTEM

s
ic
at
em
h
at
M
of
e
ut
stit
In
UP
Chapter 4

Surfaces in R3

In the previous calculus course, we considered equations in the variables x and y, and we have seen
that such equation represents a curve in the two-dimensional space. In this chapter, we introduce the
three-dimensional coordinate system and we consider equations involving the variables x, y, and/or

ic s
z. The graph of an equation in the variables x, y, and/or z is a surface in the three-dimensional

at
space. We consider several types of surfaces and their equations in the three-dimensional space.
h em
at
M

4.1 The Three-Dimensional Coordinate System


of
e
ut


Recall that R2 = (x, y) x, y ∈ R . Geometrically, we represent it using the Cartesian coordinate

tit

plane such that any point in R2 corresponds to an ordered pair (x, y). To locate a point in space,
s

three numbers are required. We represent any point in space by (a, b, c), an ordered triple of real
In

numbers.
UP

Definition 4.1.1. The correspondence between points in space and ordered triples (a, b, c) is called
the three-dimensional rectangular coordinate system or Cartesian space, denoted by R3 .

To represent R3 in a geometric three-dimensional


space, first, we choose a fixed point O, called the
origin, and three mutually perpendicular lines
passing through O, called the coordinate axes.
We label these axes as the x-axis, y-axis, and
z-axis.

Figure 4.1: Coordinate axes

79
80 CHAPTER 4. SURFACES IN R3

These three coordinate axes determine three co-


ordinate planes, as illustrated in the figure on
the right. The three coordinate planes then divide
the cartesian space into eight parts, called oc-
tants. The octant that contains the points whose
coordinates are all positive is called the first oc-
tant.

Figure 4.2: Coordinate planes

The direction of the positive z-axis is determined


by the right-hand rule. That is, right thumb

s
points towards the positive z-axis when the other

ic
at
four fingers are curled from the positive x-axis to

em
the positive y-axis.
h
at
Figure 4.3: Right-hand rule
M
of

Remark 4.1.2.
e
ut
tit

1. The origin has coordinates (0, 0, 0).


s
In

2. The points on the x-axis are of the form (a, 0, 0).


UP

3. The points on the y-axis are of the form (0, b, 0).

4. The points on the z-axis are of the form (0, 0, c).

5. The points on the xy-plane are of the form (a, b, 0).

6. The points on the xz-plane are of the form (a, 0, c).

7. The points on the yz-plane are of the form (0, b, c).

To locate a point P (a, b, c), we start at the origin O, then move a units along the x-axis, b units
along the y-axis, and c units along the z-axis. The point P then determines a rectangular box as
seen in the figure.
4.1. THE THREE-DIMENSIONAL COORDINATE SYSTEM 81

Figure 4.4: Point P (a, b, c) in R3

s
Example 4.1.3. Locate the following points in space.

ic
at
em
1. P (3, −2, −6) 2. Q(−4, 3, −5) 3. R(3, 3, 2)
h
at
Solution.
M
of
e
ut
s tit
In
UP

Figure 4.5: Point plotting in R3

Recall that for points P1 (x1 , y1 ) and P2 (x2 , y2 ) in R2 , the distance between P1 and P2 is given by
p
d(P1 , P2 ) = (x1 − x2 )2 + (y1 − y2 )2 ,
82 CHAPTER 4. SURFACES IN R3

while, the midpoint M of the line segment connecting P1 to P2 is given by


 
x 1 + x 2 y1 + y2
M= , .
2 2
The extensions of these formulas to R3 are given by the succeeding theorem.

Theorem 4.1.4 (Distance and Midpoint Formula in R3 ). Let P1 (x1 , y1 , z1 ) and P2 (x2 , y2 , z2 ) be
points in space. The (undirected) distance between P1 and P2 is
p
d(P1 , P2 ) = (x1 − x2 )2 + (y1 − y2 )2 + (z1 − z2 )2 ,

and the midpoint M of the line segment connecting P1 and P2 is


 
x 1 + x 2 y1 + y2 z1 + z 2
M= , , .
2 2 2
Example 4.1.5. Find the distance between the points

1. A(1, 0, −2) and B(9, 1, 2); 2. Q(3, −5, 1) and R(2, −7, 3).

ic s
Solution.

at
em
1. The distance between A(1, 0, −2) and B(9, 1, 2) is
p
h
d(A, B) = (1 − 9)2 + (0 − 1)2 + (−2 − 2)2
at
√ √
M

= 64 + 1 + 16 = 81 = 9 units.
of

2. The distance between Q(3, −5, 1) and R(2, −7, 3) is


e
ut

p
d(Q, R) = (3 − 2)2 + (−5 − (−7))2 + (1 − 3)2
tit


= 1 + 4 + 4 = 3 units.
s
In
UP

Example 4.1.6. Find the midpoint of the line segment connecting the points

1. A(4, −5, 1) and R(2, −7, −3); 2. P (3, −2, 9) and Q(−5, 2, −7).

Solution.

1. The midpoint of the line segment connecting A(4, −5, 1) and R(2, −7, −3) is
 
4 + 2 −5 − 7 1 − 3
M= , , = (3, −6, −1) .
2 2 2

2. The midpoint of the line segment connecting P (3, −2, 9) and Q(−5, 2, −7) is
 
3 − 5 −2 + 2 9 − 7
M= , , = (−1, 0, 1) .
2 2 2

EXERCISES. Do as indicated

1. Plot the following points in R3 .


4.2. PLANES IN R3 83

(a) (−2, 0, 1) (c) (7, −1, −4) (e) (−1, −2, −6)

(b) (2, 1, 5) (d) (−4, 0, 0) (f) (1, 2, −1)

2. Find the distance and the midpoint of the line segment connecting the following points.

(a) (6, −1, 0) and (0, −3, 3) (c) (1, −7, 6) and (9, −3, 5)
(b) (2, 1, −5) and (1, 4, −3) (d) (0, 0, 8) and (−9, 12, 0)

4.2 Planes in R3
In this section, we consider the simplest surface one can think of: a plane. Recall that an equation
in the variables x and y of the form ax + by = 0, where at least one of a and b is nonzero, represents
a line in the two-dimensional space. We consider its analogue in the three-dimensional space, linear
equations in x, y, and z. However, we only look at special cases in this section and we delay the
general discussion of planes until we formally define it in Section 5.4.

ic s
at
Remark 4.2.1.

em
1. The set of points {(x, y, 0) : x, y ∈ R} is the xy-plane. Thus, the xy-plane has equation z = 0.
h
at
2. The set of points {(x, 0, z) : x, z ∈ R} is the xz-plane. Thus, the xz-plane has equation y = 0.
M
of

3. The set of points {(0, y, z) : y, z ∈ R} is the yz-plane. Thus, the yz-plane has equation x = 0.
e
ut

4. Let a be a nonzero constant. The set of points {(a, y, z) : y, z ∈ R} is a plane through (a, 0, 0)
tit

parallel to the yz-plane. The equation of this plane is x = a.


s
In

5. Let b be a nonzero constant. The set of points {(x, b, z) : x, z ∈ R} is a plane through (0, b, 0)
UP

parallel to the xz-plane. The equation of this plane is y = b.

6. Let c be a nonzero constant. The set of points {(x, y, c) : x, y ∈ R} is a plane through (0, 0, c)
parallel to the xy-plane. The equation of this plane is z = c.

x=2 y=1 z = −3
84 CHAPTER 4. SURFACES IN R3

4.3 Spheres
Definition 4.3.1. A sphere is the set of all points equidistant from a fixed point. The fixed point
is called the center of the sphere, and the fixed distance is called the radius.

Consider a sphere centered at C(h, k, l) with radius r. By


definition, a point P (x, y, z) is on the sphere if and only if
p
r = d(P, C) = (x − h)2 + (y − k)2 + (z − l)2 .

Hence, the equation of the sphere centered at C(h, k, l) with


radius r is given by

(x − h)2 + (y − k)2 + (z − l)2 = r2 .

In particular, the sphere centered at the origin with radius r


has equation

ic s
x2 + y 2 + z 2 = r 2 . Figure 4.6: Sphere centered at C with radius

at
r

em
Example 4.3.2. Find the equation of the sphere centered at (3, 1, −5) with diameter of 8 units.
h
at
Solution. The radius of the sphere is half of the diameter, which is 4. Applying the formula, we get
M

(x − 3)2 + (y − 1)2 + (z + 5)2 = 16.


of

Example 4.3.3. Find the center and radius of the sphere with equation x2 + y 2 + z 2 − 4x + 2y +
e
ut

6z − 11 = 0.
s tit

Solution. By completing the squares, we see that


In

(x2 − 4x + 4) + (y 2 + 2y + 1) + (z 2 + 6z + 9) = 11 + 4 + 1 + 9
UP

(x − 2)2 + (y + 1)2 + (z + 3)2 = 25.

Thus, we see that the sphere is centered at (2, −1, −3) and has radius 5.

Example 4.3.4. Find the equation of the sphere with a diameter having endpoints A(0, −1, 4)
and B(6, 3, −8).

Solution. The center of the sphere is the midpoint of the line segment connecting the points A and
B, i.e.,  
0 + 6 −1 + 3 4 − 8
C(h, k, l) = , , = (3, 1, −2).
2 2 2
The radius r of this sphere is half of the distance between A and B, i.e.,
1 1p
r = d(A, B) = (0 − 6)2 + (−1 − 3)2 + (4 − (−8))2
2 2
1√ 1√
= 36 + 16 + 144 = 196 = 7.
2 2
4.4. TRACES OF SURFACES 85

Hence, the sphere has equation

(x − 3)2 + (y − 1)2 + (z + 2)2 = 49.

EXERCISES. Do as indicated.

1. Find the equation of the sphere whose endpoints of a diameter are P (1, 0, −3) and Q(−3, 2, 1).

2. Sketch the following spheres.

(a) x2 + y 2 + z 2 = 16 (c) x2 + (y + 2)2 + z 2 = 25


(b) x2 + y 2 + (z − 3)2 = 9 (d) (x − 1)2 + (y − 1)2 + (z − 1)2 = 1

4.4 Traces of Surfaces

ic s
at
In two-dimensional analytic geometry, the graph of an equation involving x and y is a curve in R2 .

em
Now, in three-dimensional analytic geometry, an equation in x, y, and z represents a surface in R3 .
h
at
In order to sketch the graph of a surface, it is useful to determine the intersection of a surface with
M

planes parallel to the coordinate planes. These curves are called the traces (or cross-sections) of
of

the surface.
e
ut
tit

4.4.1 Review of Conic Sections


s
In

Starting from this section, it is essential to have a firm grip of conic sections as we will be using
UP

them to identify the traces of surfaces and to sketch the graphs of special surfaces. Recall the
following:

• Let p > 0. A parabola with vertex V (h, k) has an equation in one of the following forms:

y − k = 4p(x − h)2 y − k = −4p(x − h)2


opening upward opening downward
86 CHAPTER 4. SURFACES IN R3

x − h = 4p(y − k)2 x − h = 4p(y − k)2


opening to the right opening to the left

• Let a > b > 0. An ellipse with center C(h, k) has an equation in one of the following forms:

ic s
at
h em
at
M
of

(x − h)2 (y − k)2 (y − k)2 (x − h)2


+ =1 + =1
e

a2 b2 a2 b2
ut

ellipse with horizontal major axis ellipse with vertical major axis
stit
In
UP

• Let a, b > 0. A hyperbola with center C(h, k) has an equation in one of the following forms:

(x − h)2 (y − k)2 (y − k)2 (x − h)2


− =1 − =1
a2 b2 a2 b2
hyperbola with horizontal transverse axis hyperbola with vertical transverse axis
4.4. TRACES OF SURFACES 87

4.4.2 Traces
Definition 4.4.1. The intersection of a surface and a plane is called the trace of the surface on
that plane.
Remark 4.4.2. In most cases, it more convenient to compute for the trace of a surface on the
coordinate planes or planes parallel to the coordinate planes (those of the form x = k, y = k or
z = k).
Example 4.4.3. Determine the the traces of the surface z = 10 − 4x2 − y 2 on the planes x = 1,
y = 2, and z = 6.
Solution. to obtain the trace of the surface on the plane x = 1, we substitute x = 1 into the
equation of the surface. Thus, we have
z = 10 − 4(1)2 − y 2 which simplifies to z = 6 − y 2 .
This means that the trace of the surface on the plane x = 1 is a parabola. The graph on the left is
a graph showing the intersection of the surface and the plane given by x = 1. On the right is the

s
graph of the surface and the trace that we are after.

ic
at
h em
at
M
of
e
ut
s tit
In

For y = 2, we do the same thing that we did with the first part, i.e., substitute y = 2 in the equation
UP

of the surface and obtain


z = 10 − 4x2 − (2)2 which simplifies to z = 6 − 4x2 .
So the trace of the surface on the plane y = 2 is a parabola. The graph on the left is a graph
showing the intersection of the surface and the plane given by y = 2. On the right is the graph of
the surface and the trace that we are after.
88 CHAPTER 4. SURFACES IN R3

Finally for z = 6, we obtain

6 = 10 − 4x2 − y 2 which gives 4x2 + y 2 = 4.

Therefore, the trace of the surface on z = 6 is an ellipse. The illustrations are given below.

Example 4.4.4. Find the traces of x2 + 4y 2 = 1 on the planes z = −1.5, z = 0 and z = 1.

ic s
at
Solution. The traces of the surface x2 + 4y 2 = 1 on the given planes are shown below.

h em
at
M
of
e
ut
s tit
In
UP

Note that any plane having an equation z = k will intersect the surface at an ellipse with equation
x2 + 4y 2 = 1.

EXERCISES. Identify and sketch the traces of the following surfaces on the given planes.

1. z = 36 − x2 − 4y 2 on the planes x = 0, y = 3, and z = 32

2. x2 + 4y 2 − z 2 = 0 on the planes x = 2, y = 0, and z = −2

3. y 2 + z 2 = 1 on the planes x = 1, y = 1 and z = 1


y
4. z = on the planes x = 1, y = 2 and z = 1
x

1 − x2
5. z = on the planes z = −1, 0, 1
y2
4.5. CYLINDERS 89

4.5 Cylinders

Definition 4.5.1. A cylinder is a surface that consists of all lines (called rulings) that are parallel
to a given line `0 and pass through a given plane curve C (called the directrix).

Remark 4.5.2. A cylinder is said to be right if the ruling is perpendicular to the plane contain-
ing the directix. Throughout the discussion, we only consider this case. An example of a right
cylinder is shown below. The directrix is the highlighted curve on the xz-plane and the rulings are
perpendicular to the xz-plane.

ic s
at
h em
at
M
of
e
ut

Figure 4.7: Right Cylinder


s tit
In

Remark 4.5.3.
UP

1. An equation of a right cylinder involves at most two variables only.

2. The cylinder is formed by taking a copy of the generating curve and moving it in the direction
of the axis of the missing variable.

Variables in the equation Rulings


x and y only parallel to the z-axis
x and z only parallel to the y-axis
y and z only parallel to the x-axis
90 CHAPTER 4. SURFACES IN R3

x2 2
Example 4.5.4. Sketch the graph of the surface +y = 1.
9
x2
Solution. Note that the directrix is the ellipse + y2 = 1
9
on the xy-plane. The rulings are parallel to the z-axis. The
graph is called an elliptic cylinder.

Figure 4.8: Elliptic Cylinder

ic s
at
Example 4.5.5. Sketch the graph of the surface z = y 2 .
hem
at
M

Solution. Note that the directrix is the parabola z = y 2 on


of

the yz-plane. The rulings are parallel to the x-axis. The


graph is called a parabolic cylinder.
e
ut
s tit
In

Figure 4.9: Parabolic Cylinder


UP

Example 4.5.6. Sketch the graph of the surface x2 −z 2 = 1.

Solution. Note that the directrix is the hyperbola x2 −z 2 = 1


on the xz-plane. The rulings of the cylinder are lines parallel
to the y-axis. The graph is called a hyperbolic cylinder.

Figure 4.10: Hyperbolic Cylinder


4.6. SURFACES OF REVOLUTION 91

EXERCISES. Sketch a portion of the following cylinders. Indicate proper rulings.

1. x2 + y 2 = 16 z2
6. x2 − =1
9
2. y = 3x2
7. x = ln(z) + 1
3. x + y = 2 8. y = sin z

4. x = y 3 9. z = ey
1 x2 y 2
5. z = 10. + =1
x 16 4

4.6 Surfaces of Revolution

ic s
Definition 4.6.1. A surface of revolution is a surface generated by a plane curve C as it is

at
revolved about a fixed line ` that lies on the plane of C. The curve C is called the generating

em
curve of the surface of revolution and the line ` is called the axis of revolution.
h
at
M
of
e
ut
s tit
In
UP

Figure 4.11: Surface of Revolution generated by a


parabola revolved about the z-axis

Remark 4.6.2. We will only consider the special cases. That is, the axis of revolution is a coordi-
nate axis and the generating curve C lies on a coordinate plane containing the axis of revolution.

Consider a surface generated by y = f (x), which is revolved about the x-axis. Let P (x, y, z) be a
point on the surface. We know that Q(x, f (x), 0) is a point on the surface lying on the xy-plane.
Then |P C| = |QC|, with C(x, 0, 0).
92 CHAPTER 4. SURFACES IN R3

Figure 4.12: Construction of a surface of revolution

ic s
Thus, by the distance formula, we get

at
hem
at
|P C| = |QC|
M

p p
(x − x)2 + (y − 0)2 + (z − 0)2 = (x − x)2 + (f (x) − 0)2 + (0 − 0)2
of

y 2 + z 2 = [f (x)]2 .
e
ut
s tit
In

Theorem 4.6.3. The equation of the surface of revolution generated by a curve, revolved about
an axis, is given in the following table:
UP

Generating Curve Axis of Revolution Equation of the Surface of Revolution


y = f (x) x-axis y 2 + z 2 = [f (x)]2
z = f (x)
x = f (y) y-axis x2 + z 2 = [f (y)]2
z = f (y)
x = f (z) z-axis x2 + y 2 = [f (z)]2
y = f (z)

Example 4.6.4. Find the equation and sketch the graph of the surface generated by the parabola
y = z 2 on the yz-plane revolved about the z-axis.
4.6. SURFACES OF REVOLUTION 93

Solution. The generating curve is the parabola


y = z 2 = f (z) on the yz-plane. By Theorem
4.6.3, an equation of the surface of revolution is

x2 + y 2 = [f (z)]2 = z 4 or simply x2 + y 2 − z 4 = 0.

Example 4.6.5. Find the equation and sketch the graph of the surface generated by y = sin x,
revolved about the x-axis.

s
ic
at
Solution. The generating curve is the graph of h em
at
y = sin x = f (x) on the xy-plane. By Theorem
M

4.6.3, an equation of the surface of revolution is


of

y 2 +z 2 = [f (x)]2 = sin2 x or simply sin2 x−y 2 −z 2 = 0.


e
ut
s tit
In
UP

z2
Example 4.6.6. Find the equation and sketch the graph of the surface generated by y 2 + = 1,
4
revolved about the y-axis.

Solution: First note that the surface can also be gen-


p
erated by revolving the curve z = 4 − 4y 2 = f (y)
about the y-axis. Thus, by Theorem 4.6.3, an equa-
tion of the surface of revolution is

x2 + z 2 = [f (y)]2 = 4 − 4y 2 ,

or equivalently

x2 + 4y 2 + z 2 = 4.
94 CHAPTER 4. SURFACES IN R3

Example 4.6.7. Find the equation and sketch the graph of the surface generated by z = ex ,
revolved about the z-axis.

Solution. The generating curve on the xz-plane


has equation x = ln z = f (z). The equation of
the surface of revolution is

x2 + y 2 = [f (z)]2 = ln2 z,

which is equivalent to

x2 + y 2 − ln2 z = 0.

Example 4.6.8. Find an equation of a curve in the yz-plane that if revolved about the z-axis will

s
result to the surface 20 − 4z = 5x2 + 5y 2 .

ic
at
em
Solution. Note that we can rewrite the equation of surface as

20 − 4z
h
x2 + y 2 =
at
.
5
M

r r
20 − 4z 20 − 4z
of

Thus, we can take the curve y = or y = − on the yz-plane as generating


5 5
e

curves of the surface of revolution.


ut
tit

EXERCISES. Do as indicated.
s
In

1. Find the equations of the surfaces of revolution obtained when the given generating curves
UP

are revolved about the given axes of revolution. Sketch a portion of their graphs.

(a) y = −3x about the x-axis, y-axis


(b) x = ey about the x-axis, y-axis
(c) x2 + 4z 2 = 4 about the x-axis, z-axis
1
(d) z = about the x-axis, z-axis
x
(e) z = y 3 about the y-axis, z-axis
y2
(f) z 2 − = 1 about the y-axis, z-axis
9
2. Find a generating curve on the xy-plane for the surface of revolution given by 4x2 +4z 2 = e4y .

3. Find the equations of all possible generating curves for the surface of revolution given by
4x2 = 9 cosh2 z − 4y 2 .
4.7. QUADRIC SURFACES 95

4.7 Quadric Surfaces

Definition 4.7.1. The graph of an equation in R3 of the form

Ax2 + By 2 + Cz 2 + Gx + Hy + Iz + J = 0

where A, B, . . . , J ∈ R, A, B, C not all zero, is called a quadric surface, or simply a quadric.

In this section, it will be useful to recall the trace or cross-section of a surface on planes parallel to
the coordinate planes.

x2 y2 z2
Example 4.7.2. Sketch the graph of + + = 1 in R3 .
9 16 4

We shall make use of the traces on the coor-

ic s
dinate planes to skecth the quadric surface.

at
The traces on the coordinate planes are the
following: h em
at
x2 y2
• xy-plane : + = 1 (ellipse)
M

9 16
of

y2 z2
• yz-plane : + = 1 (ellipse)
e

16 4
ut

x2 z 2
tit

• xz-plane : + = 1 (ellipse)
9 4
s
In

The figure on the right shows the graph of


UP

the surface with the traces on the coordinate


planes.

The graph of this surface is called an ellipsoid.

Remark 4.7.3 (Ellipsoid). The graph of an equation of the form

x2 y 2 z 2
+ 2 + 2 = 1,
a2 b c

where a, b, c > 0 is called an ellipsoid. If any two of the denominators are equal then the ellipsoid
is called a spheroid. If the third axis is shorter than the two other axes, it is called an oblate
spheroid, whereas if it is longer, then the ellipsoid is called a prolate spheroid. If all are equal,
then we have a sphere.
96 CHAPTER 4. SURFACES IN R3

oblate spheroid prolate spheroid

y2 z2
Example 4.7.4. Sketch the graph of x2 + − = 1 in R3 .
4 9
We shall first consider the traces on the coordinate

ic s
planes:

at
em
y2
• xy-plane : x2 + = 1 (ellipse)
4
h
at
M

y2 z2
• yz-plane : − = 1 (hyperbola)
4 9
of
e

z2
ut

• xz-plane : x2 − = 1 (hyperbola)
tit

9
s

Since the branches of the hyperbolas on the yz-plane


In

and xz-plane extend along the +z-axis and −z-axis,


UP

we consider additional traces where we can cut the


surface. In particular, we take the traces on the planes
z = 3 and z = −3 :
x2 y 2
• z = ±3 : + = 1 (ellipse)
2 8
The figure on the right shows the graph of the sur-
face using the traces on the coordinate planes and the
traces on z = 3 and z = −3.

The graph of this surface is called a hyperboloid of one sheet.

Remark 4.7.5 (Hyperboloid of One Sheet). The graph of an equation of the form

x2 y 2 z 2 x2 y 2 z 2 x2 y 2 z 2
+ − = 1 or − + = 1 or − + 2 + 2 = 1,
a2 b2 c2 a2 b2 c2 a2 b c
4.7. QUADRIC SURFACES 97

where a, b, c > 0, is called a hyperboloid of one sheet.

x2 y 2 z 2 x2 y 2 z 2 x2 y 2 z 2
+ 2 − 2 =1 − 2 + 2 =1 − + 2 + 2 =1
a2 b c a2 b c a2 b c
x2 z2
Example 4.7.6. Sketch the graph of − − y2 + = 1 in R3 .
9 4
We consider the traces on the coordinate
planes:

ic s
at
x2
• xy-plane : − − y 2 = 1 (empty set)

em
9
h
z2
at
• yz-plane : −y 2 + = 1 (hyperbola)
M

4
of

x2 z 2
• xz-plane : − − = 1 (hyperbola)
e

9 4
ut
tit

However, the branches of the hyperbolas on


s

the yz-plane and on the xz-plane extend to-


In

wards the +z-axis and −z-axis. So, we cut the


UP

surface using planes parallel to the xy-plane.


In particular, the traces of the surface on the
planes z = 4 and z = −4 are
x2 y 2
• z = ±4 : + = 1 (ellipse)
27 3
The graph of the surface using the traces on
the coordinate planes and on the planes z = 4
and z = − is illustrated on the right.

The graph of this surface is called a hyperboloid of two sheets.

Remark 4.7.7 (Hyperboloid of Two Sheets). The graph of an equation of the form

x2 y 2 z 2 x2 y 2 z 2 x2 y 2 z 2
− − + = 1 or − − = 1 or − + 2 − 2 = 1,
a2 b2 c2 a2 b2 c2 a2 b c
98 CHAPTER 4. SURFACES IN R3

where a, b, c > 0, is called a hyperboloid of two sheets.

x2 y 2 z 2 x2 y 2 z 2 x2 y 2 z 2
− 2 + 2 =1 − 2 + 2 =1 − + 2 − 2 =1
a2 b c a2 b c a2 b c

y2 z2
Example 4.7.8. Sketch the graph of x2 + − = 0 in R3 .
4 9
The traces on the coordinate planes are the

s
following:

ic
at
y2
• xy-plane : x2 + = 0 (point)

em
4
y2 z2
h
at
• yz-plane : − = 0 (two lines)
4 9
M

z2
• xz-plane : x2 −
of

= 0 (two lines)
9
e
ut

Since the lines on the xz-plane and on the


tit

yz-plane extend towards the +z-axis and the


s

−z-axis, we cut the sruface using horizontal


In

planes. In particular, the traces on the planes


UP

z = 3 and z = −3 are
y2
• z = ±3: x2 + = 1 (ellipse)
4
The graph of the surface using the traces on
the coordinate planes and the obtained addi-
tional traces is shown on the right.

The graph of this surface is called an elliptic cone.

Remark 4.7.9 (Elliptic Cone). The graph of an equation of the form

x2 y 2 z 2 x2 y 2 z 2 x2 y 2 z 2
+ − = 0 or − + + = 0 or − 2 + 2 = 0,
a2 b2 c2 a2 b2 c2 a2 b c

where a, b, c > 0, is called an elliptic cone.


4.7. QUADRIC SURFACES 99

x2 y 2 z 2 x2 y 2 z 2 x2 y 2 z 2
+ 2 − 2 =0 − + 2 + 2 =0 − 2 + 2 =0
a2 b c a2 b c a2 b c

y2 z
Example 4.7.10. Sketch the graph of x2 + = in R3 .
4 3

s
The traces on the coordinate planes are the

ic
following:

at
em
y2
• xy-plane : x2 + = 0 (point) h
4
at
y2 z
M

• yz-plane : = (parabola)
4 3
of

z
• xz-plane : x2 = (parabola)
e

3
ut

An additional trace is needed since the


tit

parabola on the yz-plane extends towards the


s
In

+z-axis. In particular, we take the trace on


UP

the plane z = 3, i.e.,


y2
• z = 3: x2 + = 1 (ellipse)
4
The graph of the surface using these traces is
shown on the right.

The graph of this surface is called an elliptic paraboloid.

Remark 4.7.11 (Elliptic Paraboloid). The graph of an equation of the form

x2 y 2 z x2 z 2 y y2 z2 x
2
+ 2
= or 2
+ 2
= or 2
+ 2 = ,
a b c a c b b c a

is called an elliptic paraboloid.


100 CHAPTER 4. SURFACES IN R3

x2 y 2 z x2 z 2 y y2 z2 x
2
+ 2 = , c>0 2
+ 2 = , b>0 2
+ 2 = , a>0
a b c a c b b c a

ic s
at
em
h
at
x2 y 2 z x2 z 2 y y2 z2 x
M

+ 2 = , c<0 + 2 = , b<0 + 2 = , a<0


a2 b c a2 c b b2 c a
of
e
ut

Example 4.7.12. Sketch the graph of x2 − y 2 = −z in R3 .


tit

The traces on the coordinate planes are the following:


s
In

• xy-plane : x2 − y 2 = 0 (two lines)


UP

• yz-plane : −y 2 = −z (parabola)

• xz-plane : x2 = −z (parabola)

We cut the surface by horizontal planes and planes parallel to the xz-plane. In particular, we can
take the following additional traces:

• z = 4: x2 − y 2 = −4 (hyperbola)

• z = −4: x2 − y 2 = 4 (hyperbola)

• y = 4: x2 − 16 = −z (parabola)

• y = −4: x2 − 16 = −z (parabola)

The graph of the surface using all the above traces is shown in the following illustration.
4.7. QUADRIC SURFACES 101

The graph of this surface is called a hyperbolic paraboloid or a saddle.

s
Remark 4.7.13 (Hyperbolic Paraboloid). The graph of an equation of the form

ic
at
y 2 x2 z z 2 x2 y z2 y2 x

em
2
− 2
= or 2
− 2
= or 2
− 2 = ,
b a c c a b
h c b a
is called a hyperbolic paraboloid.
at
M
of
e
ut
s tit
In
UP

y 2 x2 z z 2 x2 y z2 y2 x
− 2 = − 2 = − 2 =
b2 a c c2 a b c2 b a

The figure on the right shows a hyperbolic


paraboloid with complete parabolic traces.
102 CHAPTER 4. SURFACES IN R3

Remark 4.7.14.

1. If in the equation of a quadric, x, y, and z are replaced by x − h, y − k, and z − `, respectively,


then all the points on the quadric are translated by h units in x, k units in y, and ` units in
z.

2. From our previous examples, observe that additional traces are considered. These additional
traces give an appropriate cut on the surface. In case the quadric is translated h units in x,
k units in y, and ` units in z, the traces on the planes x = h, y = k, and z = ` are helpful for
sketching.

3. In some cases, it is more difficult to sketch a quadric if we follow the usual orientation of the
coordinate axes (with the z-axis pointing up). Thus, it is sometimes convenient to rotate the
axes to make the sketching easier. However, one must make sure that the axes still follow the
right hand rule.

EXERCISES. Identify the type of the following quadric surfaces. Find the traces on the coordinate

ic s
planes. Find additional traces whenever necesaary. Sketch the graph of the quadric using the traces.

at
1. z = 4 − x2 − 4y 2
8.
x2 h em
− y2 +
z2
=0
at
4 4
M

2. x = y 2 + 9z 2 (y − 2)2 (z − 2)2
9. x2 + + =1
of

4 4
3. x2 − y 2 = z
e

p
10. z = 3x2 + 3y 2
ut

4. y 2 − 4z 2 = x
tit

p
11. z = 4 − 4x2 − y 2
s
In

x2 y 2 z2
5. + + =1 x2 y 2 (z − 3)2
4 9 16 12. + − =1
UP

4 9 9
x2 y2 z2
6. − + − z2 = 1 13. x2 − y 2 + =1
4 16 16
x2 y 2 z2 (x − 2)2 y 2 z2
7. − + + =0 14. − − =1
9 9 16 4 9 16

4.8 Chapter Exercises


I. Sketch the graphs of the following planes.

1. 3x − 4y + z = 12
2. −x + 2y − 6z = 0
3. x + 2z = 4
4. y − 4z = 0
4.8. CHAPTER EXERCISES 103

II. Consider the equation E : y = 9 − x2 .

1. Sketch the graph of E in R3 .


2. Find the equation of the surface generated by revolving about the y-axis the parabola on
the xy-plane with equation E. Sketch the surface of revolution.

III. Given the curve y = ln x.

1. Sketch the given curve as a cylinder in R3 .


2. Find the equation of the surface of revolution if the given curve is revolved about the
y-axis. Sketch the surface of revolution.
y2 z2
IV. Find the equation of the surface of revolution when the hyperbola − = 1 is revolved
9 4
about the y-axis and the z-axis. Sketch the resulting surfaces of revolution.

V. Given the surface 4z + 16 = 3x2 + 3y 2 .

ic s
1. Identify its traces at the coordinate planes and sketch its graph.

at
2. Find the equation of a curve in the xz-plane that if revolved about the z-axis will result
to the given surface.
em
h
at
z2 x2
VI. Let S be the surface with equation given by y 2 − =1+ .
M

3 12
of

1. What type of quadric is S?


e

2. Write the equation of the traces of S on the coordinate planes and on the planes y = ±2.
ut

Be sure to identify the conics they represent.


s tit

3. Sketch the surface S. Emphasize the traces on the planes as given above.
In

y 2 (z − 4)2
UP

VII. Let S be the quadric with equation x2 + − = 1.


4 2
1. Identify the type of quadric.
2. Give the standard equations of the traces of S on the coordinate planes and on the planes
z = 4 and z = 8. Identify these traces.
3. Sketch S using the traces. Label important points on the traces.
104 CHAPTER 4. SURFACES IN R3

ics
at
hem
at
M
of
e
ut
stit
In
UP
Chapter 5

Vectors, Lines, and Planes

In physics, one usually uses different types of quantities when describing the motion of an object.
These quantities can be classified in one of two categories, namely scalars and vectors.

For instance, one can look at the displacement and the distance traveled by the object from one

ic s
point to another. A single number is needed to represent the distance, which refers to the total

at
length of the path traversed from point A to point B. This is an example of a scalar quantity. On
h em
the other hand, the displacement indicates the relative position of point B from point A. This is
an example of a vector quantity. Other examples of a vector quantity are velocity and acceleration
at
M

of a moving object, and force that acts on an object.


of

In this chapter, we deal mainly with vectors in R2 and in R3 . We also look at how vectors can be
e
ut

used to find equations of lines and planes in space.


s tit
In

5.1 Vector Notation and Geometric Representation


UP

In this section, we give notations and geometric representation of vectors in the three-dimensional
space. A vector is a quantity that indicates length and direction. It is geometrically represented
by an arrow from an initial point to a terminal point.

Definition 5.1.1. A vector in the three-dimensional space is an ordered triple of real numbers
ha, b, ci. The numbers a, b, and c are called components of the vector.

Remark 5.1.2.

1. Generally, a vector may be represented by a letter with an arrow above it, say ~v . The vector
~v = ha, b, ci may be represented by any directed line segment from a point (x0 , y0 , z0 ) to the
point (x0 + a, y0 + b, z0 + c).
−−→
2. The vector from the point P (x1 , x2 , x3 ) to the point Q(y1 , y2 , y3 ) is denoted by P Q and has
−−→
components P Q = hy1 − x1 , y2 − x2 , y3 − x3 i.

105
106 CHAPTER 5. VECTORS, LINES, AND PLANES

3. The representation of a vector ~v = ha, b, ci with initial point at the origin and terminal point
at (a, b, c) is called the position representation of the vector.

Q(x2 , y2 , z2 )

P (x1 , y1 , z1 ) −
−→
PQ

Figure 5.1: Vector from P to Q Figure 5.2: position representation of ha, b, ci

ic s
4. For every point P (a, b, c), there corrresponds a vector whose position representation has

at
terminal point at P ; that is, the vector ha, b, ci. Thus, there is a one-to-one correspondence
between the set of vectors and the set of points in R3 . h em
at
5. The vector whose components are all zero is called the zero vector, and is denoted by ~0. It
M

is represented by any point in the space.


of
e

6. If one component of a vector is zero, the vector lies on a coordinate plane. In particular, if
ut

the third component is zero, the vector is on the xy-plane and we simply write ha, bi.
s tit

7. Two vectors ~v1 = ha1 , b1 , c1 i and ~v2 = ha2 , b2 , c2 i are equal if and if only their corresponding
In

components are equal, i.e., a1 = a2 , b1 = b2 and c1 = c2 . Geometrically, equal vectors are


UP

represented by arrows with the same length and direction.

Example 5.1.3.

1. The vector from the point (4, 1, −3) to the origin is given by

~v = h0 − 4, 0 − 1, 0 − (−3)i = h−4, −1, 3i .

2. The vector from the point (2, −5, 1) to the point (0, 2, −4) is given by

~v = h0 − 2, 2 − (−5), −4 − 1i = h−2, 7, −5i .

3. The representation of the vector ~v = h−3, 1, −4i with initial point at P (−1, 2, 3) has termi-
nal point Q(−4, 3, −1). The same vector can be represented by the arrow from the point
A(4, −1, −2) to the point B(1, 0, −6).
5.1. VECTOR NOTATION AND GEOMETRIC REPRESENTATION 107

Definition 5.1.4. The magnitude or norm of the vector ~v = ha, b, ci is the length of a represen-
tation of ~v , and is given by
p
k~v k = a2 + b2 + c2 .

Remark 5.1.5. The magnitude of a vector is always non-negative. The magnitude is zero if and
only if the vector is the zero vector.

Example 5.1.6. Find the magnitude of the following vectors.

1. v~1 = h2, 1, −2, i 3. v~3 = h−5, 13, 12i


* √ √ √ +

6 2 3
 10 10 2
2. v~2 = , , 4. v~4 = − , ,
7 7 7 10 5 2

Solution.

√ √ √ √ √

s
1. kv~1 k = 4+1+4= 9=3 3. kv~3 k = 25 + 169 + 144 = 338 = 13 2

ic
at
r r
36 4 9 1 2 1

em
2. kv~2 k = + + =1 4. kv~4 k = + + =1
49 49 49 h 10 5 2
at
Remark 5.1.7. A vector with magnitude 1 is called a unit vector.
M

Definition 5.1.8. The direction angles of a nonzero vector ~v are the least nonnegative angles
of

α, β, and γ that the position representation of ~v makes with the positive x-axis, y-axis, and z-axis,
e
ut

respectively. The cosines of these direction angles are called direction cosines of the vector ~v .
tit
s
In
UP

Figure 5.3: Direction Angles

Remark 5.1.9.

1. Each direction angle has value on the interval [0, π].


108 CHAPTER 5. VECTORS, LINES, AND PLANES

a b c
2. If ~v = ha, b, ci, then cos α = , cos β = and cos γ = , from which it follows that
k~v k k~v k k~v k

cos2 α + cos2 β + cos2 γ = 1.



Example 5.1.10. Find the direction angles of the vector ~v = 2, −1, 1 .

Solution. Since k~v k = 2 + 1 + 1 = 2, then

2 1 1
cos α = , cos β = − , and cos γ =
2 2 2
and so,
π 2π π
α= , β= and γ = .
4 3 3
5π π π
Example 5.1.11. Show that there is no vector with direction angles α = , β = , and γ = .
6 3 4
Solution. We prove by contradiction. Suppose that there is a vector with the given direction angles.

ic s
Then, its direction cosines are

at

3 1 1

em
cos α = − , cos β = , and cos γ = √ .
2 2 h 2
at
Therefore,
M

3 1 1 3
cos2 α + cos2 β + cos2 γ =
+ + = 6= 1.
4 4 2 2
of

Thus, the given set of direction angles violates the second statement in Remark 5.1.9, hence, no
e
ut

such vector exists.


tit

Remark 5.1.12. For two-dimensional vectors, we only consider one direction angle. In particular,
s
In

the direction angle of a vector ~v = ha, bi is the least nonnegative angle α that the position represen-
UP

tation of ~v makes with the positive x-axis. In this case, the components of the vector ~v are given
by
a = k~v k cos θ and b = k~v k sin]θ.

EXERCISES. Do as indicated.
−−→
1. Find the vector P Q.

(a) P (−3, 8, 1), Q(2, 5, 7) (c) P (2, 5, −1), Q(6, −3, −2)
(b) P (4, −6, 2), Q(−1, 9, 0) (d) P (−7, 0, 9), Q(9, 5, −4)

2. Let ~v = h3, −2, −4i. Find the terminal point of the representation of ~v having the given point
as its initial point.
5.2. VECTOR OPERATIONS 109

(a) origin (c) (3, −4, 2)


(b) (−6, 0, 1) (d) (2, −3, −1)

~ = h2, k, −ki and B


3. Find the value(s) of k so that the vectors A ~ = h2, k 3 , k 2 i are equal.

4. Determine the magnitude of the following vectors.

(a) h3, −5, −4i (c) h1, 2, −2i


(b) h−3, 6, 2i (d) h5, 0, 12i


k
5. Find all values of k for which the vector 2 , k, 1 − k is a unit vector.

6. Find the direction angles α, β and γ of h0, − 3, 1i, where α, β and γ are the angles formed
by the vector and the positive x-, y- and z-axes respectively.

s
5.2 Vector Operations

ic
at
em
In this section, we consider several operations which can be performed on vectors, give their geo-
metric meaning, and provide some geometric applications.
h
at
M

5.2.1 Scalar Multiplication and Vector Addition


of

Definition 5.2.1. Let ~v = ha1 , a2 , a3 i and let c ∈ R. Multiplying each component of ~v by the
e
ut

scalar c yields a vector, denoted by c~v . That is,


tit

c~v = hca1 , ca2 , ca3 i.


s
In

This process is called scalar multiplication.


UP

Remark 5.2.2.

1. 0 ~v = ~0.
p p
2. kc~v k = (ca1 )2 + (ca2 )2 + (ca3 )2 = |c| a21 + a22 + a23 = |c| k~v k.

3. If c > 0, then c~v points in the same direction as ~v . On the other hand, if c < 0, then c~v points
in the direction opposite of ~v .

4. Two nonzero vectors ~v1 and ~v2 are parallel if and only if ~v2 = c~v1 , for some c ∈ R \ {0}.

Definition 5.2.3. Let A ~ = ha1 , a2 , a3 i, and B


~ = hb1 , b2 , b3 i. The sum A
~+B
~ and the difference
~−B
A ~ are respectively given by

~+B
A ~ = ha1 + b1 , a2 + b2 , a3 + b3 i
~−B
A ~ = ha1 − b1 , a2 − b2 , a3 − b3 i
110 CHAPTER 5. VECTORS, LINES, AND PLANES

Remark 5.2.4.

~ and B
1. If A ~ are vectors positioned so the initial point of B
~ is at the terminal point of A,
~ then
the sum A~+B ~ is the vector from the initial point of A~ to the terminal point of B.
~

~
A

~
B
~+B
A ~

Figure 5.4: Vector Addition

2. If the vectors A~ and B~ are positioned such that they share the same initial point, the vector
A~ −B~ is the vector from the terminal point of B
~ to the terminal point of A.
~ This comes from
the fact that B~ + (A~ − B)
~ = A.~

ic s
at
~
A em
h
at
M
of

~−B
~
e

A
ut

~
B
s tit
In
UP

Figure 5.5: Vector Subtraction

~+B
3. Alternatively, the sum A ~ and the difference A~−B ~ are vectors defining the diagonals of
~ and B
the parallelogram having the position representation of A ~ as two adjacent sides.

~ = h1, 2, 3i and B
Example 5.2.5. Let A ~ = h−4, 5, −6i. Find the following:

~+B
1. A ~

~−B
2. 2A ~
1~ 3~
3. the magnitude of B+ A
2 2
Solution.

~+B
1. A ~ = h1, 2, 3i + h−4, 5, −6i = h−3, 7, −3i
5.2. VECTOR OPERATIONS 111

~−B
2. 2A ~ = h2, 4, 6i − h−4, 5, −6i = h6, −1, 12i
     
1~ 3~ 5 3 9 1 11 3
3. We have B + A = −2, , −3 + , 3, = − , , . Thus,
2 2 2 2 2 2 2 2
  r √
1 3 1 11 3 1 121 9 131
~ ~

2 + 2 A = − 2 , 2 , 2 = 4 + 4 + 4 = 2 .
B

Remark 5.2.6.

1. Let ~v be a nonzero vector. The vector ~u~v = k~v1k ~v is a unit vector in the same direction as ~v .
Indeed,
1 1
k~u~v k =
k~v k ~v = k~v k k~v k = 1,

since k~v k > 0. Moreover, ~u~v and ~v have the same direction since k~v k ~u~v = ~v and k~v k > 0.

2. It can be shown that the components of u~v are the direction cosines of ~v .

ic s
The process of obtaining a unit vector along a given nonzero vector is called normalization.

at
em
Example 5.2.7. Given the points P (3, 2, 1) and Q(0, 5, −1), find the unit vector that has the same
−−→ h
direction as the vector P Q.
at
−−→
M

Solution. We first find the vector P Q and take its norm; that is,
of

−−→ −−→ √
P Q = h−3, 6, −2i and P Q = 9 + 36 + 4 = 7.
e
ut

−−→
Then by item 1 of Remark 5.2.6, the unit vector in the direction of P Q is
tit

−−→
s

 
PQ 1 3 6 2
In

~uP~Q = −−→ = h−3, 6, −2i = − , , − .


P Q 7 7 7 7
UP

Example 5.2.8. Find a vector of length 5 that has the same direction as the vector ~v = h1, −2, 2i.

Solution. Observe that k~v k = 1 + 4 + 4 = 9. The desired vector must be 5 times the unit vector
along ~v . That is,  
5 5 5 10 10
5~u~v = ~v = h1, −2, 2i = ,− , .
k~v k 3 3 3 3
Remark 5.2.9. Observe that any vector ~v = ha, b, ci can be expressed as a sum of scalar multiples
of three unit vectors, i.e.,
~v = ah1, 0, 0i + bh0, 1, 0i + ch0, 0, 1i.

For this reason, the three unit vectors above are called standard basis vectors for R3 and are
denoted by
ı̂ := h1, 0, 0i, ̂ := h0, 1, 0i, and k̂ := h0, 0, 1i.

Hence, the vector ~v = ha, b, ci may also be written as ~v = aı̂ + b̂ + ck̂.
112 CHAPTER 5. VECTORS, LINES, AND PLANES


̂
ı̂

x y

Figure 5.6: Standard Basis Vectors

Example 5.2.10. In terms of the three standard basis vectors, we have h3, 5, −2i = 3ı̂ + 5̂ − 2k̂,
h1, 0, 3i = ı̂ + 3k̂, and h0, 7, −4i = 7̂ − 4k̂.

5.2.2 Dot Product


Definition 5.2.11. The dot product or scalar product of two vectors A ~ = ha1 , a2 , a3 i and

ic s
~ = hb1 , b2 , b3 i, denoted by A
B ~ · B,
~ is the scalar quantity given by

at
em
~·B
A ~ = a1 b1 + a2 b2 + a3 b3 .
h
~ = h2, −1, 4i, B
~ = h−3, 2, 0i and C
~ = h7, 4, 1i, then
at
Example 5.2.12. If A
M

~·B
1. A ~ = (2)(−3) + (−1)(2) + (4)(0) = −8
of

~ ·B
2. C ~ = (7)(−3) + (4)(2) + (1)(0) = −13
e
ut

 
3. A ~+B~ ·C ~ = h−1, 1, 4i · h7, 4, 1i = (−1)(7) + (1)(4) + (4)(1) = 1
stit
In

The following theorem enumerates some of the properties of the dot product.
UP

~ B,
Theorem 5.2.13. Let A, ~ and C
~ be vectors and c ∈ R.

~·B
1. A ~ =B ~ ·A
~
     
~· B
2. A ~ +C~ = A~·B
~ + A ~·C
~
     
~ · cB
3. A ~ = cA
~ ·B
~ =c A~·B
~

~ · ~0 = 0
4. A
2
~·A
5. A ~= A~

Proof. We prove items 1 and 2, and the proof of the rest of the properties is left to the reader. Let
~ = ha1 , a2 , a3 i, B
A ~ = hb1 , b2 , b3 i, and C
~ = hc1 , c2 , c3 i. Then by commutativity of multiplication of
real numbers, we have
~·B
A ~ = a1 b1 + a2 b2 + a3 b3 = b1 a1 + b2 a2 + b3 a3 = B
~ · A.
~
5.2. VECTOR OPERATIONS 113

Furthermore,
 
~· B
A ~ +C
~ = ha1 , a2 , a3 i · (hb1 , b2 , b3 i + hc1 , c2 , c3 i)

= ha1 , a2 , a3 i · hb1 + c1 , b2 + c2 , b3 + c3 i
= a1 (b1 + c1 ) + a2 (b2 + c2 ) + a3 (b3 + c3 )
= a1 b1 + a1 c1 + a2 b2 + a2 c2 + a3 b3 + a3 c3
= (a1 b1 + a2 b2 + a3 b3 ) + (a1 c1 + a2 c2 + a3 c3 )
   
= A ~·B ~ + A ~·C~ .

The dot product of two vectors is essential in finding the angle determined by the vectors. The
angle between two nonzero vectors is the least nonnegative angle between their position
representation.

ic s
z

at
h em
at
M

~
A
of

θ
y
e

~
ut

B
tit

x
s
In

Figure 5.7: Angle between two vectors


UP

~ and B.
Remark 5.2.14. Let θ be the angle between the vectors A ~

1. Then θ ∈ [0, π].

~ and B
2. The vectors A ~ are parallel if and only if θ = 0 or θ = π .

~ are perpendicular or orthogonal if and only if θ = π .


~ and B
3. The vectors A
2
~ and B
Theorem 5.2.15. Let A ~ be nonzero vectors and let θ be the angle between them. Then

~·B
A ~
cos θ = .
~ ~
A B

Proof. Consider the triangle determined by the position representations of two vectors. Note that
~ − B.Using
the third side is equivalent to A ~ the Cosine Law, we see that
114 CHAPTER 5. VECTORS, LINES, AND PLANES

2 2
~ ~ 2

~ ~ ~ ~
A + B − 2 A B cos θ = A − B
   
= A ~−B ~ · A ~−B~
   
=A~· A ~−B ~ −B ~· A ~−B
~
~ ~−B
A ~
A
=A~·A ~−A~·B~ −B~ ·A~+B ~ ·B
~
2 2 θ
~ ~·B~ + ~
= A − 2A B .
~
B
~·B
Hence, A ~ =
A~ ~
B cos θ.

~ = 2î − 2ĵ + k̂ and B


Example 5.2.16. Find the angle between A ~ = î + k̂.

ics
at
Solution. We apply Theorem 5.2.15 and obtain

h2, −2, 1i · h1, 0, 1i 2+0+1 1


h em
at
cos θ = = √ √ =√ .
kh2, −2, 1ik kh1, 0, 1ik 9 2 2
M
of

Therefore, the angle between the given vectors is


e
ut
tit

 
−1 1 π
s

θ = cos √ = .
In

2 4
UP

~ = 3î − 2ĵ + 3k̂ and B


Example 5.2.17. Show that the vectors A ~ = î − 3ĵ − 3k̂. are perpendicular.

~ and B.
Solution. Let θ be the angle between A ~ Then

h3, −2, 3i · h1, −3, −3i 3+6−9


cos θ = = √ √ = 0,
kh3, −2, 3ik kh1, −3, −3ik 22 19
π
which means θ = .
2

~ and B
Hence, A ~ are perpendicular.
5.2. VECTOR OPERATIONS 115

Example 5.2.18. In Physics, the work W done by a force F~ that acts


on an object with displacement d~ is given by W = F~ · d. ~ A wagon is
pulled a distance of 50 meters along a horizontal path by a constant force
of 60 N. The handle of the wagon is held at an angle of 45◦ above the
horizontal. Find the work done by the force.

Solution. Since the force and displacement are not explicitly given, we
use Theorem 5.2.15 to write the dot product in terms of the norms of
the vector and the angle between them. With θ = 45◦ , we have

W = F~ · d~
= kF kkdk cos θ
= (60)(50) cos (45◦ )

= 1500 2 N · m.

The work done is 1500 2 N · m.

ic s
at
Remark 5.2.19.
hem
at
M
of
e

~ and B
1. Two nonzero vectors A ~ are perpendicular or orthogonal if and only if A
~·B
~ = 0.
ut
s tit

~ are perpendicular then the angle between them θ = π . Thus,


~ and B
In

Indeed, if A
2
UP

π 
~·B
A ~ =
A~ ~
B cos = 0.
2

Conversely, if A ~ = 0, then cos θ = 0. Hence, θ = π and so, A


~·B ~ and B
~ are perpendicular.
2

2. We can extend the definition of parallel and perpendicular vectors to include the zero vector
and consider it to be parallel and at the same time perpendicular to all vectors.
116 CHAPTER 5. VECTORS, LINES, AND PLANES

A useful application of the dot product is on vector projections.

Definition 5.2.20. Let A ~ and B ~ be nonzero vectors. The vector projection of B ~ onto A,
~
~ is the perpendicular projection of the position representation of B
denoted by projA~ B, ~ onto the
~
line containing the position representation of A.

~ onto A
Figure 5.8: Projection of B ~

~ is given by kBk
As seen in Figure 5.8, the magnitude of projA~ B ~ |cos θ|. Furthermore,
 ~
~ cos θ A

~ = kBk
projA~ B

s
~
kAk

ic
at
!
~ Bk
kAkk ~ cos θ
= ~
A

em
~ 2
kAk
~·B ~
! h
A
at
= ~
A.
~ 2
kAk
M
of

Remark 5.2.21. Let θ be the angle between nonzero vectors A ~ and B.


~
π
e

1. If 0 ≤ θ < , the projection of B~ onto A ~ has the same direction as A.


~
ut

2
tit

π ~ onto A~ is the zero vector.


2. If θ = , the projection of B
s

2
In

π ~ onto A ~ points in the direction opposite of A.


~
3. If < θ ≤ π, the projection of B
UP

2
Example 5.2.22. Consider the vectors A ~ = h1, 0, −2i and B ~ = h2, 1, −1i. Determine the vector
projection of B~ onto A
~ and the vector projection of A~ onto B.
~

~ and B,
Solution. We first compute for the dot product of A ~ and the magnitudes of A
~ and B:
~
√ √
~·B
A ~ = 2 + 0 + 2 = 4,
A~ ~
= 6, B = 5.

~ onto A
The projection of B ~ is
~ · B)
(A ~ 4

4 2 2

~ ~
projA~ B = 2 A = h2, 1, −1i = , ,− .
~ 6 3 3 3
A

~ onto B
On the other hand, the projection of A ~ is
~ · B)
(A ~ 4

4 2

~ ~
projB~ A = 2 B = h1, 0, −2i = , 0, − .
~ 5 5 5
B
5.2. VECTOR OPERATIONS 117

Example 5.2.23. Express A ~ = h2, 3, 5i as a sum of a vector parallel to B


~ = h2, −1, −2i and a
~
vector perpendicular to B.

~ is parallel to B.
Solution. Note that projB~ A ~ Thus,

~ · A)
~
~ = (
projB~ A
B
2 B~ = (4 − 3 − 10) h2, −1, −2i = h−2, 1, 2i .
~ 4+1+4
B

Moreover, take
~ =A
C ~ = h2, 3, 5i − h−2, 1, 2i = h4, 2, 3i .
~ − proj ~ A
B

~ and B
Based from the construction of the vector projection, one can verify that C ~ are perpendicular.
Therefore, we may write
~ = h2, 3, 5i = h−2, 1, 2i + h4, 2, 3i .
A

ic s
at
5.2.3 Cross Product h em
at
~ = ha1 , a2 , a3 i and B
Definition 5.2.24. The cross product of two vectors A ~ = hb1 , b2 , b3 i is the
M

vector given by
of

~×B
A ~ = ha2 b3 − a3 b2 , a3 b1 − a1 b3 , a1 b2 − a2 b1 i .
e
ut
tit

Remark 5.2.25.
s
In
UP

1. The cross product is applied on vectors in R3 . In the case that a given vector in a cross
product is in R2 , say ~v = ha, bi, re-write the vector to the form ~v = ha, b, 0i.

~ = ha1 , a2 , a3 i and B
2. We can solve for the cross-product of A ~ = hb1 , b2 , b3 i in the following
manner:

ı̂ ̂ k̂
~×B

~ := a1 a2 a3 = a 2 a3
a a
1 3
a a
1 2
A ı̂ − ̂ + k̂,

b2 b3 b1 b3 b1 b2
b1 b2 b3


x x
1 2
where = x1 y2 − x2 y1 .
y1 y2

~ = h1, 3, 4i and B
Example 5.2.26. Let A ~ = h2, 7, −5i. Find A
~×B
~ and B
~ × A.
~
118 CHAPTER 5. VECTORS, LINES, AND PLANES

Solution. Using item 2 of Remark 5.2.25, we have



ı̂ ̂ k̂

~×B
A ~ = 1 3 4

2 7 −5

= (−15 − 28)ı̂ − (−5 − 8)̂ + (7 − 6)k̂


= −43ı̂ + 13̂ + k̂.

ı̂ ̂ k̂

~ ×A
B ~ = 2 7 −5

1 3 4

= (15 + 28)ı̂ − (5 + 8)̂ + (−7 + 6)k̂


= 43ı̂ − 13̂ − k̂.

The following properties of the cross-product can be verified using the definition.
~ B
Theorem 5.2.27. Let A, ~ and C
~ be vectors in R3 and let c ∈ R.

ic s
 
~×B
1. A ~ =− B ~ ×A~

at
em
     
~× B
2. A ~ ±C
~ = A ~×B ~ ± A~×C ~
h
at
   
~ × cB
3. A ~ =c A ~×B ~
M

~ × ~0 = ~0
4. A
of
e

~×A
5. A ~ = ~0
ut
tit

As seen in Theorem 5.2.15, the cosine of the angle between two vectors can be solved using the dot
s

product of the two vectors. In a similar manner, the sine of the angle between two vectors can be
In

obtained using the cross product of the vectors.


UP

~ and B.
Theorem 5.2.28. Let θ be the angle between the vectors A ~ Then,

~ ~ ~ ~
A × B = A B sin θ.

~ = ha1 , a2 , a3 i and B
Proof. Let A ~ = hb1 , b2 , b3 i.
~ ~ 2

2
A × B = kha2 b3 − a3 b2 , a3 b1 − a1 b3 , a1 b2 − a2 b1 ik
= (a2 b3 − a3 b2 )2 + (a3 b1 − a1 b3 )2 + (a1 b2 − a2 b1 )2
= (a21 + a22 + a23 )(b21 + b22 + b23 ) − (a1 b1 + a2 b2 + a3 b3 )2 (Verify!)
2 2  2
~ ~ ~·B ~
= A B − A
2 2 2 2
~ ~ ~ ~
= A B − A B cos2 θ
2 2
~ ~
= A B sin2 θ

Since θ ∈ [0, π], then sin θ ≥ 0. This proves the equality we need.
5.2. VECTOR OPERATIONS 119

Remark 5.2.29.

~×B
1. Two vectors are parallel if and only if A ~ = ~0.

~ and B
2. The area of the parallelogram having sides A ~ is
A~×B
~ . Indeed, if we let the base
~ the adjacent side be B,
of the parallelogram be A, ~ and the angle between them be θ, then

Area = base length × altitude



~ ~
= A B sin θ

~ ~
= A × B .

Example 5.2.30. Find the area of the parallelogram having the position representation of the
~ = 3ı̂ − 8̂ − 6k̂ and B
vectors A ~ = 3̂ + 2k̂ as adjacent sides.

Solution. Solving for the cross-product of the given vectors, we have

ic s
at

ı̂ ̂ k̂

em
~×B
A ~ = 3 −8 −6 = 2ı̂ − 6̂ + 9k̂.
h
0 3 2
at
M

Therefore, the area of the parallelogram is


of

√ √
Area = 2ı̂ − 6̂ + 9k̂ = 4 + 36 + 81 = 121 = 11 units2 .

e
ut
tit

Example 5.2.31. Find the area of the triangle with vertices at the points A(1, 2, 4), B(3, 1, 10)
s

and C(1, 4, 0).


In

−−→ −→
UP

Solution. Form two vectors using the given points, say AB = h2, −1, 6i and AC = h0, 2, −4i. These
vectors are adjacent sides of the parallelogram with three of its vertices at the points A, B, and C.
The area of the parallelogram is

−−→ −→ ı̂ ̂ k̂


AB × AC = 2 −1 6 = kh−8, 8, 4ik = 64 + 64 + 16 = 12.


0 2 −4

Therefore, the area of the triangle with vertices at the points A(1, 2, 4), B(3, 1, 10) and C(1, 4, 0)
is half that of the parallelogram, i.e., A4 = 6 units2 .

An important property of the cross-product of two nonzero vectors is proved in the following
theorem.

~×B
Theorem 5.2.32. The vector A ~ is orthogonal to both A
~ and B.
~
120 CHAPTER 5. VECTORS, LINES, AND PLANES

~ = ha1 , a2 , a3 i and B
Proof. Let A ~ = hb1 , b2 , b3 i, then
 
~×B
A ~ ·A ~ = ha2 b3 − a3 b2 , a3 b1 − a1 b3 , a1 b2 − a2 b1 i · ha1 , a2 , a3 i

= (a1 a2 b3 − a1 a3 b2 ) + (a2 a3 b1 − a2 a1 b3 ) + (a3 a1 b2 − a3 a2 b1 )


= a1 a2 b3 − a2 a1 b3 + a2 a3 b1 − a3 a2 b1 + a3 a1 b2 − a1 a3 b2
= 0.
 
In a similar manner, it can be shown that A ~×B
~ · B.
~
Thus, A~×B ~ is orthogonal to both A
~ and B.
~

Remark 5.2.33. The vectors A, ~ B,


~ and A~×B ~ follow the right-hand rule. That is, right thumb
~×B
must point in the direction of A ~ when the four other fingers are curled from the vector A
~
~
towards the vector B.

ic s
at
h em
at
M
of
e
ut

Figure 5.9: Right-hand rule


tit

~ = h1, 0, 1i and B
Example 5.2.34. Find two unit vectors perpendicular to A ~ = h3, −1, 1i.
s
In

Solution. Let C~ =A ~ × B,
~ which is orthogonal to both vectors A
~ and B.
~ Using the formula for the
UP

cross product, we obtain C ~ = h−1, 2, −1i. Thus, the two unit vectors perpendicular to A
~ and B~
are
* √ √ √ + *√ √ √ +
C~ h−1, 2, −1i 6 6 2 6 6 2

~ = 1 + 4 + 1 = − 6 , 3 ,− 2
~uC~ = , and − ~uC~ = ,− , .

C 6 3 2

EXERCISES. Do as indicated.

1. Given the vectors ~a = h2, −1, 3i, ~b = P~Q, where P (1, 1, 1) and Q(5, 1, −1), and ~c = ı̂ + ̂ + 3k̂.
Find:

(a) 7~b − 3~c


(b) the unit vector in the same direction as ~b.
(c) the measure of the angle between ~b and ~c.
5.3. EQUATIONS OF LINES IN R3 121

(d) the vector projection of ~a onto ~c.


(e) the volume of the parallelepiped defined by ~a, ~b and ~c.

~ and B
2. Let A ~ be nonzero vectors. Show that the vectors proj ~ A
~ and A
~ − proj ~ A
~ are perpen-
B B
dicular.

3. Express the vector ~v = h−1, 3, 2i as a sum of two vectors ~u and w


~ such that ~u is parallel to
~ ⊥ ~u.
2ı̂ + 3k̂ and w

4. Consider the points A(4, 7, 2), B(6, 3, 0) and C(7, 1, −3) and the parallelogram with AB and
AC as edges.

(a) Find the coordinate of the fourth vertex D of the parallelogram.


(b) Find the area of the parallelogram.

~ and B
5. Suppose A ~ are non-zero vectors in R3 . Show that

~ · B)
~ 2 + ||A
~ × B||
~ 2 = ||A||
~ 2 ||B||
~ 2.

s
(A

ic
at
em
5.3 Equations of Lines in R3 h
at
Recall that in R2 , a line is determined by a point on the line and the steepness of the line, called
M

slope. In R3 , the notion of slope is quite ambiguous since lines with varying directions may have
of

the same steepness. In the three-dimensional space, a unique line is determined by a point on the
e

line and a nonzero vector that is parallel to the line.


ut
s tit
In
UP

Figure 5.10: Line parallel to a vector Figure 5.11: Line Construction

Let P0 (x0 , y0 , z0 ) be a point on the line ` and let ~v = ha, b, ci be some vector that is parallel to
`. We wish to find equations satisfied by any point on `. Take an arbitrary point P (x, y, z) on `.
−−→
Then, P0 P = t~v , for some t ∈ R. That is,

hx − x0 , y − y0 , z − z0 i = tha, b, ci.
122 CHAPTER 5. VECTORS, LINES, AND PLANES

Equivalently, we have the following equations:

x = x0 + at
y = y0 + bt
z = z0 + ct.

Remark 5.3.1.

1. The line through the point P0 (x0 , y0 , z0 ) that is parallel to representations of the vector
~v = ha, b, ci can be represented by the set of equations

x = x0 + at, y = y0 + bt, z = z0 + ct,

called parametric equations of the line.

2. The vector ~v or any nonzero vector parallel to it is called a direction vector for the line.

ic s
at
3. If all components of a direction vector iare nonzero, we get the following equations by solving

em
for t in the equations in (1):
x − x0 y − y0 z − z0h
= = ,
at
a b c
M

called symmetric equations of the line.


of

Example 5.3.2. Find parametric equations of the line `2 containing the point (2, −3, 9) and is
e
ut

parallel to the line `1 with symmetric equations


tit

x−3 y+5 2−z


s

= = .
In

4 2 3
UP

Solution. A direction vector for `1 is ~v = h4, 2, −3i. Since `1 and `2 are parallel, then `2 is parallel
to ~v as well. Hence, `2 has parametric equations

x = 2 + 4t, y = −3 + 2t, z = 9 − 3t.

Example 5.3.3. Find the symmetric equations of the line that contains the points A(0, 3, −4) and
B(1, −1, 1).

~ = h1 − 0, −1 − 3, 1 − (−4)i = h1, −4, 5i is parallel to the line. Thus, the


Solution. The vector AB
line has symmetric equations
3−y z+4
x= = .
4 5
Example 5.3.4. Determine whether the line `1 : x = 2t − 1, y = t, z = 3t + 5 and the line
`2 : x = 3s − 5, y = 5 − 2s, z = s + 6 intersect. If they do, find the coordinates of the point of
intersection.
5.3. EQUATIONS OF LINES IN R3 123

Solution. Suppose that the two lines intersect. Then this point of intersection must be a solution
of the system of equations


 2t − 1 = 3s − 5

t = 5 − 2s .

3t + 5 = s + 6

Observe that there are only two unknowns in the system while there are three equations. Using
the first two equations, t = 1 and s = 2. Since these values satisfy the third, then the two lines
intersect. Plugging in t = 1 in the equations of `1 or s = 2 in the equations of `2 , the point of
intersection is (1, 1, 8).

ic s
at
Remark 5.3.5. Two non-intersecting lines in space are said to be parallel if they are parallel to

em
the same non-zero vector, otherwise they are said to be skew.
h
at
M

x−6 2−z
of

Example 5.3.6. Show that the lines `1 : =y= and `2 : x = 3s, y = 2s−7, z = 2s+5
3 5
e

are skew.
ut
s tit
In

Solution. From their given equations, we can see that `1 is parallel to ~v1 = h3, 1, −5i, while `2 is
UP

parallel to ~v2 = h3, 2, 2i. Clearly, ~v1 and ~v2 are not parallel, and so, `1 and `2 are not parallel.

Suppose that `1 and `2 intersect, then the system of equations



 3t + 6 = 3s
t = 2s − 7

−5t + 2 = 2s + 5

must have a solution. It can be checked that the first two equations is satisfied by t = 3 and s = 5.
However, these values do not satisfy the third equation. Thus, the system has no solution, so the
lines do not intersect. Hence, `1 and `2 are skew.
124 CHAPTER 5. VECTORS, LINES, AND PLANES

EXERCISE. Do as indicated.

1. Give parametric equations and symmetric equations of the line through P parallel to repre-
sentations of the vector ~v .

(a) P (5, 1, −2), ~v = h3, −7, 4i (c) P (3, −7, 8), ~v = h−5, 0, 2i
(b) P (−6, 0, 4), ~v = 8, 21 , −3


(d) P (2, −2, 0), ~v = h1, 4, 3i

2. Find parametric and symmetric equations of the line through the two given points.

(a) origin and (8, −1, −2) (c) (8, 4, 7), (5, 4, −3)
(b) (−6, 2, 0) and (4, 9, 1) (d) (−1, 2, 6), (3, −4, 2)

ic s
3. Find parametric equations of the line through the point A(4, −3, 0) that is parallel to the line

at
em
3−x z−1
=y+5= .
2 7 h
at
M

y−1 z−4 x+1 z−3


4. Let `1 : 2 − x = = and `2 : =y−2= .
2 3 2 4
of
e

(a) Show that the two lines intersect and find the coordinates of the pooint of intersection.
ut
tit

(b) Find the acute angle formed by the two lines.


s
In
UP

5.4 Equations of Planes in R3

In the three-dimensional space, a plane is uniquely determined by a point on the plane and a vector
that is perpendicular to the plane.
Suppose that a plane π contains the point P0 (x0 , y0 , z0 ).
Let ~n = ha, b, ci be any nonzero vector perpendicular to π.
We wish to find an equation satisfied by any point on the
plane π. To do this, take any point P (x, y, z) on π, and con-
−−→ −−→
struct the vector P0 P . Then, ~n and P0 P are perpendicular.
Therefore,
−−→
~n · P0 P = 0
ha, b, ci · hx − x0 , y − y0 , z − z0 i = 0
a (x − x0 ) + b (y − y0 ) + c (z − z0 ) = 0.
5.4. EQUATIONS OF PLANES IN R3 125

Remark 5.4.1.

1. The plane through the point P0 (x0 , y0 , z0 ) that is perpendicular to representations of the
vector ~n = ha, b, ci has equation

a (x − x0 ) + b (y − y0 ) + c (z − z0 ) = 0.

This is called the point-normal form of the equation of the plane.

2. The vector ~n is called a normal vector to the plane, and any nonzero vector parallel to ~n is
also a normal vector to the plane.

3. The point-normal form is equivalent to ax + by + cz + d = 0, where d = −ax0 − by0 − cz0 ,


which is called the general form of the equation of the plane.

Example 5.4.2. Find the general equation of the plane that contains the point (5, 0, −3) and is
perpendicular to the vector h4, −7, 2i.

ic s
Solution. The given vector serves as a normal vector to the plane. Thus, using the point-normal

at
form of the equation of the plane, we have
h em
4(x − 5) − 7(y − 0) + 2(z + 3) = 0.
at
M

The general equation of the plane is


of

4x − 7y + 2z − 14 = 0.
e
ut
tit

Example 5.4.3. Find the equation of the plane through the origin that is perpendicular to the
s

line with symmetric equations


In

x−2 y
= = 1 − z.
5 2
UP

Solution. Since the plane is perpendicular to the line, then a direction vector for the line is a normal
vector to the plane. So, we may take ~n = h5, 2, −1i. The point-normal equation of the plane is
5x + 2y − z = 0.

Remark 5.4.4. A unique plane is determined by any of the following:

• three distinct non-collinear points

• a line and a point not on the line

• two distinct intersecting lines

• two distinct parallel lines

Example 5.4.5. Find the equation of the plane containing the points A(1, 0, 1), B(2, 3, 4) and
C(0, 2, −1).
126 CHAPTER 5. VECTORS, LINES, AND PLANES

−−→ −→
Solution. Consider AB = h1, 3, 3i and AC = h−1, 2, −2i. Note that the cross product of two
−−→ −→ −−→ −→
vectors is perpendicular to both vectors. Since AB and AC are on the plane, AB × AC is a normal
vector to the plane. We have
−−→ −→
AB × AC = h−12, −1, 5i .

Taking A as a point on the plane, the plane has point-normal equation

−12(x − 1) − y + 5(z − 1) = 0.

Example 5.4.6. Find the equation of the plane containing the point P (1, 4, 3) and the line ` with
parametric equations x = 2 − 3t, y = 5 + 2t, z = 4 + t.
−−→
Solution. Take a point on `, say Q(2, 5, 4), and form the vector P Q = h2 − 1, 5 − 4, 4 − 3i = h1, 1, 1i.
−−→
Meanwhile, the line ` has direction vector ~v = h−3, 2, 1i. Both the vectors P Q and ~v lie on the
required plane, and hence, we may take the cross product of these vectors as a normal vector to
the plane. We have
−−→
~n = ~v × P Q = h1, 4, −5i.

ic s
Thus, the point-normal equation of the plane is

at
(x − 1) + 4(y − 4) − 5(z − 3) = 0.
h em
at
Example 5.4.7. Find the equation of the plane containing the lines `1 : x = t, y = −2t, z = 3t
M

and `2 : x = 3t, y = −t, z = 0 .


of

Solution. First observe that the two lines intersect at the origin, and hence, there is a unique plane
e
ut

containing both lines. Let ~v1 and ~v2 be direction vectors for `1 and `2 , respectively. We may take
tit

~v1 = h1, −2, 3i and ~v2 = h3, −1, 0i. A normal vector ~n to the required plane is perpendicular to
s

~×B ~ = h3, 9, 5i. The point-normal equation of


In

both ~v1 and ~v2 , and thus, we may consider ~n = A


the plane is 3x + 9y + 5z = 0.
UP

Remark 5.4.8.

1. Two distinct planes are parallel if any of their normal vectors are parallel.

2. Two distinct planes are perpendicular if any of their normal vectors are perpendicular.

Example 5.4.9. Show that the planes π1 : 4x + 3x − 7z = 5 and π2 : 5x − 2y + 2z = 3 are


perpendicular.

Solution. Let ~n1 and ~n2 be normal vectors to π1 and π2 , respectively. We may take ~n1 = h4, 3, −7i
and ~n2 = h5, −2, 2i. Then,

~n1 · ~n2 = (4)(5) + (3)(−2) + (−7)(2) = 0,

which implies that n~1 and ~n2 are perpendicular. Hence, π1 and π2 are perpendicular.
5.4. EQUATIONS OF PLANES IN R3 127

Remark 5.4.10. If in the equation ax + by + cz + d = 0, the coefficients a, b, c, d are all nonzero,


one can sketch a triangular portion of the plane using the x-intercept, y-intercept and z-intercept.

ic s
at
em
2x + 3y + z = 6 x + 4y − z = 4
h
at
M

Remark 5.4.11. If in the equation ax + by + cz + d = 0, the coefficients a, b, c are nonzero and


of

d = 0, the plane contains the origin, and one can sketch a portion of the plane by taking another
e
ut

two points on the plane (preferably points on two different coordinate planes).
stit
In
UP

x+y+z =0 x + y − 2z = 0

Remark 5.4.12. If in the equation ax + by + cz + d = 0, exactly one of a, b and c is zero and d 6= 0,


the plane is parallel to the axis of the missing variable.
128 CHAPTER 5. VECTORS, LINES, AND PLANES

2x + 3y = 6 y + 4z = 2

Remark 5.4.13. If in the equation ax + by + cz + d = 0, exactly one of a, b and c is zero and d = 0,


the plane contains (or is hinged at) the axis of the missing variable.

ic s
at
h em
at
M
of
e
ut
s tit
In
UP

y − 3x = 0 2x − 3z = 0

The next examples illustrate the techniques of finding equations of the line of intersection of two
planes, and the coordinates of the point of intersection of a line and a plane.

Example 5.4.14. Find the intersection of the planes π1 : 2x + y − z = 1 and π2 : 3x − 2y + z = 0.

Solution. Based on the given equations, we may take the normal vectors ~n1 = h2, 1, −1i perpen-
dicular to π1 and ~n2 = h3, −2, 1i perpendicular to π2 , which are not parallel. Hence the planes
intersect. Let (x, y, z) be any point on the line of intersection. Then, these coordinates must satisfy
the system
(
2x + y − z = 1
.
3x − 2y + z = 0
5.4. EQUATIONS OF PLANES IN R3 129

Generally, we can let any of the coordinates x, y, and z be the parameter and solve for the other
two using the equations in the system. For instance, if we take x = t, then
(
y − z = 1 − 2t
.
−2y + z = −3t
which has solutions y = 5t − 1 and z = 7t − 2. Thus, the line of intersection is defined by the
parametric equations

x=t
y = 5t − 1
z = 7t − 2.

Alternatively, one can take a point common to both planes; that is, a point that satisfies the
equations of both planes, and a vector that is perpendicular to ~n1 and ~n2 , for instance ~n1 × ~n2 .

Example 5.4.15. Find the point of intersection of the line ` : x = 7 + 3t, y = −2t, z = 5 + t and
the plane π : 4x − 5y + 6z = 2.

ic s
Solution. Let P (x, y, z) be the point of intersection. Replacing x, y, and z on π1 with their values

at
on `, we have

em
4(7 + 3t) − 5(−2t) + 6(5 + t) = 2,
h
at
whose solution is t = −2. Substituting this value in the parametric equations of `, we obtain the
M

point of intersection to be P (1, 4, 3).


of
e
ut

EXERCISES. Do as indicated.
tit

1. Find the equation of the plane containing the points P (3, 1, 5), Q(−1, −1, −1), and R(−2, 2, 4).
s
In

2. Find the equation of the plane through P (2, −1, 1) that is parallel to Π : 2x + 3y + z − 12 = 0.
UP

z+4
3. Find an equation of the plane that contains the point (2, 0, 3) and the line x + 1 = y =
.
2
y−1 z−4
4. Given the plane π : 2x + y − 3z − 2 = 0, and the lines `1 : 2 − x = = and
2 3
x+1 z−3
`2 : =y−2= .
2 4
(a) Find a set of parametric equations of the line containing the point P (1, 0, −2) and which
is perpendicular to π.
(b) Find the equation of the plane containing `1 and `2 .

5. Consider the planes π1 : 4x − y − z + 3 = 0 and π2 : 2x + y − 2z − 1 = 0

(a) Determine the acute angle between the planes π1 and π2 .


(b) Find parametric equations for the line of intersection of π1 and π2 .

6. Sketch a portion of the following planes.


130 CHAPTER 5. VECTORS, LINES, AND PLANES

(a) 3x + 4y + 24z = 12 (e) 2x + y + 4z = 0 (i) 2y + z = 6


(b) x + y − z = 1 (f) 3x − 2y + 6z = 0 (j) 3x − 2y = 0
(c) −2x + y − 5z = 10 (g) 4x − 3y = 6 (k) 5z − x = 0
(d) x − 3y − z = 0 (h) x + z = −2 (l) y + z = 0

5.5 Chapter Exercises


~ = h−1, −4, 1i and B
I. Consider the vectors A ~ = h−1, 2, −2i.

~
1. Find the vector of length 9 in the direction opposite of B.
2. Determine the angle between A and B.
~ onto A.
3. Find the vector projection of B ~

~ = h2, −3, 6i and B


II. Let A ~ = h1, 0, −1i.

s
~
1. Find the unit vector in the direction opposite A.

ic
at
~
2. Determine the direction angles of B.

em
~ and B.
3. Find the angle between A ~ h
~ onto A.
~
at
4. Determine the vector projection of B
M

~ and B
5. Find the area of the triangle with A ~ as adjacent sides.
of

III. Consider the point P (1, −2, 5), the plane π : 3x + 2y − z = 8, and the line ` : x = 5 + 8t, y =
e
ut

4, z = 2 − 2t.
tit

1. Find the equation of the sphere centered at P that is tangent to the plane π.
s
In

2. Find the point of intersection of ` and π.


UP

3. Give symmetric equations of the line through P perpendicular to the plane π.


4. Find the general equation of the plane containing the point P and the line `.
x+3 10 − y
IV. Given point P (−2, 9, 3), line l : = = z − 1 and plane Π : 6x − 2y + 3z − 7 = 0.
2 3
1. Give two unit vectors orthogonal to Π.
~ = 3î + k̂ onto a normal vector to Π.
2. Determine the vector projection of A
3. Find a set of parametric equations of the line through P perpendicular to Π.
4. Determine the coordinates of the point of intersection l and Π.
5. Find the general equation of the plane containing P and l.
Chapter 6

Vector-Valued Functions

6.1 Introduction
6.1.1 Parametric Curves in R3

s
In Chapter 3, we have seen that a pair of parametric equations x = x(t) and y = y(t) defines a

ic
at
plane curve containing points (x, y) by assigning values for the parameter t. In this section, we

em
extend this idea to space curves.
h
at
A collection of points (x, y, z) satisfying the equations
M
of

x = x(t)
e

y = y(t)
ut

z = z(t)
s tit
In

is called a parametric curve in R3 or simply space curve.


UP

Remark 6.1.1.

1. A space curve is traced in the direction of incresing values of the parameter.

2. Without an indicated restriction, parameter values are taken on the set domx ∪ domy ∪ domz.

3. If t ∈ [a, b], the point (x(a), y(a), z(a)) is called initial point while the point (x(b), y(b), z(b))
is called terminal point of the space curve.

4. Space curves with one zero coordinate function are plane curves. In particular, a parametric
curve on the xy-plane has third coordinate z = 0.

Example 6.1.2. Consider the parametric equations x = 1 + 3t, y = −2 + 4t, z = 3 − t.

• If t ∈ R, the space curve is the line through the point (1, −2, 3) and is directed along the
vector ~v = h3, 4, −1i.

131
132 CHAPTER 6. VECTOR-VALUED FUNCTIONS

• If t ∈ [0, 2], the space curve is the line segment from the point P (1, −2, 3), corresponding to
t = 0, to the point Q(7, 6, 1), corresponding to t = 2.

x2 y 2
Example 6.1.3. Find a set of parametric equations for the intersection of the cylinder + =1
9 4
and the plane y + z = 3.

Solution. The intersection is the collection of points in the plane y + z = 3, whose x and y
x2 y2
coordinates satisfy + = 1. See Figure 6.1. Recall that the generating curve of the cylinder
9 4
can be parametrized by x = 3 cos t, y = 2 sin t. Since the curve is on the plane y + z = 3, its
z−coordinate must be given by z = 3 − y = 3 − 2 sin t. Therefore, a parametrization of the curve
of intersection is
x = 3 cos t, y = 2 sin t, z = 3 − 2 sin t.

ic s
at
h em
at
M
of
e
ut
s tit
In
UP

Figure 6.1: Intersection of a cylinder and a plane

6.1.2 Vector-Valued Functions of One Parameter

Suppose that a space curve is parametrized by x = x(t), y = y(t), z = z(t). Due to the one-to-one
correspondence between the set of points and the set of vectors in R3 , we can look at the space
curve as the curve traced out by the tips (terminal points) of the position representation of the
vectors hx(t), y(t), z(t)i as t varies on domx ∪ domy ∪ domz. Thus, we can also use a vector-valued
function to represent space curves.

Definition 6.1.4. A vector-valued function R ~ of one parameter, or simply a vector function,


is a function whose domain is a set of real numbers and whose range is a set of vectors. We write,

~
R(t) = hx(t), y(t), z(t)i,
6.1. INTRODUCTION 133

where x, y, and z are real-valued functions of t. The (natural) domain of the vector-valued function
~ denoted by domR,
R, ~ is given by

~ = domx ∩ domy ∩ domz.


domR

Remark 6.1.5. One may restrict the domain of a vector function to a subset of its natural domain.
Example 6.1.6. Find the domain of the vector function.

1 1 √
 
~ (t) = ln(t − 1)ı̂ +
1. R ̂ ~ (t) =
2. S , 4 − t, et
2
t −t−6 t+5

Solution.
1
1. Let x(t) = ln(t − 1) and y(t) = . Then
t2 − t − 6
dom x = (1, +∞) and dom y = R\{−2, 3}.

Hence,

s
~ = (1, +∞) ∩ R\{−2, 3} = (1, +∞)\{3}
dom R

ic
at
1 √
2. Let x(t) = , y(t) = 4 − t, and z(t) = et . Then

em
t+5
h
dom x = R\{−5} , dom y = (−∞, 4] and dom z = R.
at
M

Hence,
~ = R\{−5} ∩ (−∞, 4] ∩ R = (−∞, 4]\{−5}.
dom S
of
e
ut

6.1.3 Graphs of Vector Functions


tit

Definition 6.1.7. The graph of R ~ (t) = hx(t), y(t), z(t)i is the space curve traced out by the
s
In

~
terminal points of the position representation of the vector R(t), ~
for all t ∈ dom R.
UP

Remark 6.1.8. The graph of the vector function R ~ (t) = hx(t), y(t), z(t)i is the space curve with
parametric equations x = x(t), y = y(t), z = z(t), as shown in Figure 6.2. Furthermore, the
graph has an orientation indicated by the increasing values of the parameter t, represented by an
arrowhead.

~
Figure 6.2: Graph of R(t)
134 CHAPTER 6. VECTOR-VALUED FUNCTIONS

Example 6.1.9. Sketch the curve with the given vector equation.

~ (t) = t2 ı̂ + (t + 1) ̂
1. R ~ (t) = hcos t, sin t, ti
2. S

Solution.

1. Note that the vector function has two components only, which means that its graph is a
plane curve. The corresponding parametric equations are x = t2 and y = t + 1, which can be
transformed to the Cartesian equation x = (y − 1)2 . Thus, the graph of R ~ is a parabola on
the xy-plane that opens to the positive x-axis with vertex at (0, 1), as shown in Figure 6.3.
It is traced upward, since as the parameter value increases, y increases as well.

ic s
at
~
Figure 6.3: Graph of R(t)
h em
= t2 ı̂ + (t + 1) ̂
at
M

2. Since x2 +y 2 = cos2 t+sin2 t = 1, the curve must lie on the circular cylinder x2 +y 2 = 1. Since
of

z = t, the point (x, y, z) lies directly above (x, y, 0) and spirals upward in a counterclockwise
e

~ (t) = hcos t, sin t, ti is a


ut

direction as t increases. Thus, the curve with vector equation S


tit

circular helix. Th graph is shown in Figure 6.4.


s
In
UP

~ = hcos t, sin t, ti
Figure 6.4: Graph of S(t)

Remark 6.1.10. The line through the point P0 (x0 , y0 , z0 ) directed along the vector ~v = ha, b, ci
~
has vector equation R(t) = hx0 + at, y0 + bt, z0 + cti.
6.1. INTRODUCTION 135

Example 6.1.11. Find a vector equation of the line segment from the point P (1, −2, 3) to the
point Q(4, 2, 2).

Solution. The direction of the line segment is the vector


−−→
P Q = h4 − 1, 2 − (−2), 2 − 3i = h3, 4, −1i. Using the initial
point P , a vector equation of the line containing the line
~
segment is R(t) = h1 + 3t, −2 + 4t, 3 − ti. We must restrict
the parameter values to an interval to get the desired line
segment. The initial point P corresponds to t = 0, while
the terminal point Q corresponds to t = 1. Therefore, the
line segment from P to Q has vector equation

~
R(t) = h1 + 3t, −2 + 4t, 3 − ti, 0 ≤ t ≤ 1.

Its graph is shown on the left.

ic s
Remark 6.1.12. If one component function of a vector-valued function is the zero function, then

at
the vector-valued function is two-dimensional. In this case, the graph is a plane curve. In particular,
~
the graph of the vector-valued function R(t)
em
= hx(t), y(t)i is the parametric curve with equations
h
at
x = x(t), y = y(t), whose orientation is indicated by the increasing values of the parameter t.
M
of

6.1.4 Operations on Vector Functions


e
ut

Operations and properties of vectors in R3 are also enjoyed by vector functions.


tit

Definition 6.1.13. Let F~ and G


~ be vector functions and f be a real-valued function. We define
s
In

the following operations:


UP

1. (F~ ± G)(t)
~ := F~ (t) ± G(t)
~

2. (F~ · G)(t)
~ := F~ (t) · G(t)
~

3. (F~ × G)(t)
~ := F~ (t) × G(t)
~

4. (f F~ )(t) := f (t)F~ (t)

5. (F~ ◦ f )(t) := F~ (f (t))

Example 6.1.14. Let F~ (t) = t + 1, t2 − 1, t − 1 , G(t)


~ = ht − 1, 1, t + 1i and f (t) = et − 1.

We have:

1. (F~ − G)(t)
~ = t + 1, t2 − 1, t − 1 − ht − 1, 1, t + 1i

= 2, t2 − 2, −2


136 CHAPTER 6. VECTOR-VALUED FUNCTIONS

2. (F~ × G)(t)
~ = ((t + 1)ı̂ + (t2 − 1)̂ + (t − 1)k̂) × ((t − 1)ı̂ + ̂ + (t + 1)k̂)
 
t2 − 1 (t + 1) − (t − 1) ı̂ − (t + 1)2 − (t − 1)2 ̂
 
=
+ (t + 1) − (t − 1) t2 − 1 k̂


= t3 + t2 − 2t î − (4t) ĵ + −t3 + t2 + 2t k̂
 

3. (F~ ◦ f )(t) = F
~ et − 1

D 2 E
et − 1 + 1, et − 1 − 1, et − 1 − 1
 
=
= et , e2t − 2et , et − 2

EXERCISES. Do as indicated.

1. Find parametric equations for the curve of intersection of the circular cylinder 9x2 + 4y 2 = 36
and the plane 2x − 3y + z = 1.
y2
2. Find parametric equations for the curve of intersection of the paraboloids 8 − z = x2 +
9
y2
and z = x2 + .

s
9

ic
at
3. Give a vector equation of the curve of intersection of the parabolic cylinder z = 4x2 + 2 and

em
the paraboloid 4y = 2x2 + z 2 .
h
at
4. Find the domain of the following vector-valued functions.
M

~ ln(1 − t) t2 − 4
(a) R(t) = 2 ı̂ + ̂
of

t +1 2t + 4

 
1
e

~
(b) R(t) = t2 − 4, sin t,
ut

t
tit

2
 
~ 2 t −4 t
(c) R(t) = ln(t − 1), ,e
s

t−2
In

5. Let P (0, 2, −1) and Q(2, −3, −4).


UP

(a) Find a vector equation for the the line through P and Q.
(b) Find a vector equation for the line segment from P to Q.

6. Let F~ (t) = et−1 − t2 , 3t2 + 1, 4 − 5 ln t , G(1)


~


= h−6, 1, 0i, and f (t) = cos t. Evaluate:
 
(a) F~ · G ~ (1)
 
(b) F~ × G ~ (1)
 
(c) F~ ◦ f (π)

6.2 Calculus of Vector Functions


In this section, we shall extend the notion of limits, derivatives, and integrals to vector-valued
functions. We shall also see geometric interpretation of these quantities.
6.2. CALCULUS OF VECTOR FUNCTIONS 137

6.2.1 Limits and Continuity of Vector Functions


~ (t) = hx (t) , y (t) , z (t)i.
Definition 6.2.1. Let R

~ (t) as t approaches a is defined as


1. The limit of R
D E
~ (t) = lim x (t) , lim y (t) , lim z (t) ,
lim R
t→a t→a t→a t→a

provided the limit of each component exists. Otherwise, the limit does not exist.

~ is said to be continuous at t = a if it satisfies the following:


2. The function R

• R
~ (a) exists;

• lim R
~ (t) exists;
t→a

• R
~ (a) = lim R
~ (t).
t→a

~ is said to be discontinuous at t = a.
If any of the conditions fails to hold, the function R

ic s
at
Example 6.2.2. Evaluate the following:
h em
at
t2 − 4 sin (2t − 4)
 
|t − 1| 1 + cos πt tan πt
 
1. lim t + 1, , 2. lim , ,
M

t→2 t−2 t−2 x→1− t−1 t2 − 1 t−1


of

Solution. As per item 1 of Definition 6.2.1, we have the following:


e
ut

t2 − 4 sin (2t − 4) t2 − 4
   
sin (2t − 4)
1. limt→2 t + 1, , = lim (t + 1) , lim , lim
tit

t−2 t−2 t→2 t→2 t − 2 t→2 t−2


s

 
2 cos (2t − 4)
In

= 3, lim (t + 2) , lim
t→2 t→2 1
UP

= h3, 4, 2i
   
|t − 1| 1 + cos πt tan πt |t − 1| 1 + cos πt tan πt
2. limx→1− , , = lim , lim , lim
t−1 t2 − 1 t−1 x→1− t − 1 x→1− t2 − 1 x→1− t − 1
π sec2 πt
 
−π sin πt
= −1, lim , lim
x→1− 2t x→1− 1
= h−1, 0, πi

Example 6.2.3. Determine whether the function


 
 sin(t) , t − 1, et , t 6= 0

~ (t) =
R t
ı̂ − 2̂ + k̂,

t=0

is continuous at t = 0.

Solution. We check if the three conditions for continuity are satisfied:


138 CHAPTER 6. VECTOR-VALUED FUNCTIONS

• R
~ (0) = ı̂ − 2̂ + k̂ = h1, −2, 1i,i.e., R(0)
~ exists
   
sin(t)
• limR
~ (t) = lim , lim(t − 1), lim et
= h1, −1, 1i, so the limit exists
t→0 t→0 t t→0 t→0

• R
~ (0) 6= lim R
~ (t).
t→0

~ is discontinuous at t = 0.
The third condition is not satisfied. Hence, R

6.2.2 Derivatives of Vector Functions


~ (t) = hx (t) , y (t) , z (t)i. The derivative of R
Definition 6.2.4. Let R ~ is defined by

~ ~
~ 0 (t) = lim R (t + ∆t) − R (t) ,
R
∆t→0 ∆t

if this limit exists.

ic s
at
h em
at
~ + ∆t) − R(t)
R(t ~
Since ∆t is a scalar, the vectors and
M

∆t
~ + ∆t) − R(t)
R(t ~ are parallel. Moreover, as shown in
of

~ + ∆t) − R(t)
R(t ~
Figure 6.5, the vector approaches a
e

∆t
ut

vector tangent to the graph of R ~ (t) as ∆t → 0.


s tit
In

~ 0 (t)
Figure 6.5: The tangent vector R
UP

Remark 6.2.5.

~ 0 (t) exists at t = a, we say that R


1. If R ~ is differentiable at t = a. If R
~ 0 (t) exists for all t on
an interval I, we say that R ~ is differentiable on I.

2. As illustrated in Figure 6.5, the representation of the vector R~ 0 (t) with initial point at the
~
tip of the position representation of R(t) is tangent to the graph of R ~ in the direction of the
curve.

The following theorem shows that to find the derivative of a vector-valued function, one can simply
differentiate each component function.

Theorem 6.2.6. Let R ~ (t) = hx (t) , y (t) , z (t)i. Then R


~ 0 (t) = hx 0 (t) , y 0 (t) , z 0 (t)i, provided
x 0 (t) , y 0 (t) and z 0 (t) exist.
6.2. CALCULUS OF VECTOR FUNCTIONS 139

~
Proof. Let R(t) = hx(t), y(t), z(t)i. By definition, we have

~ ~
~ 0 (t) = lim R (t + ∆t) − R (t)
R
∆t→0 ∆t
hx (t + ∆t) , y (t + ∆t) , z (t + ∆t)i − hx (t) , y (t) , z (t)i
= lim
∆t→0
 ∆t 
x (t + ∆t) − x (t) y (t + ∆t) − y (t) z (t + ∆t) − z (t)
= lim , lim , lim
∆t→0 ∆t ∆t→0 ∆t ∆t→0 ∆t
= x (t) , y 0 (t) , z 0 (t) .

0

~ 0 (t) and R
~ 00 (t) if R
~ (t) = ln t, tanh t, − sin−1 t .


Example 6.2.7. Find R

Solution. By differentiating the components of R ~ to get R ~ 0 and differentiating the components of


~ 0 to get R
R ~ 00 , we have
 
~ 0 (t) = 1 , sech2 t, √ −1

s
R

ic
t 1 − t2

at
* +
−t −2 1 (−2t)

em
~ 00 (t) =
R , −2sech2 t tanh t, · p .
2 h 2 (1 − t2 )3
at
Example 6.2.8. Determine a vector equation of the line tangent to the graph of the vector function
M

~ (t) = 2 + ln t, t sin(1 − t), t3 at the point where t = 1.




R
of

Solution. Recall that the vector equation of the line through a point (x0 , y0 , z0 ) with direction
e
ut

~
vector ~v = ha, b, ci is of the form L(t) = hx0 + at, y0 + bt, z0 + cti. The point of tangency, which
tit

~
lies on the tangent line, is the tip of the position representation of the vector R(1) = h2, 0, 1i, i.e.,
s
In

~ 0
the point (2, 0, 1). Moreover, the vector R (1) indicates the direction of the tangent line. We have
UP

 
~ 0 1 2
R (t) = , sin(1 − t) − t cos(1 − t), 3t ,
t

~ 0 (1) = h1, −1, 3i. Therefore, a vector equation of the tangent line is
which gives R

~
L(t) = h2 + t, −t, 1 + 3ti .

Some properties of the derivative of vector-valued functions are given in the following theorem.

Theorem 6.2.9. Let F~ and G ~ be vector functions and f be a real-valued function. If F~ , G,


~ and
f are differentiable, then the following hold:

1. (F~ ± G)
~ 0 (t) = F~ 0 (t) ± G
~ 0 (t)

2. (f F~ )0 (t) = f (t)F~ 0 (t) + F~ (t)f 0 (t)

3. (F~ · G)
~ 0 (t) = F~ (t) · G
~ 0 (t) + G(t)
~ · F~ 0 (t)
140 CHAPTER 6. VECTOR-VALUED FUNCTIONS

4. (F~ × G)
~ 0 (t) = F~ (t) × G
~ 0 (t) + F~ 0 (t) × G(t)
~

5. (F~ ◦ f )0 (t) = F~ 0 (f (t))f 0 (t)

Example 6.2.10. Let F~ (t) = t − 1, t3 , cos t , G


~ (t) = hln t, sinh t, −4i and f (t) = e−t . Evaluate

(F~ · G)
~ 0 (t) and (G
~ ◦ f )0 (t).

Solution.
0  
 1
• ~ ~ 3 , cosh t, 0 + hln t, sinh t, −4i · 1, 3t2 , − sin t



F · G (t) = t − 1, t , cos t ·
t
t−1
= + ln t + t cosh t + 3t2 sinh t + 4 sin t
3
t
 
1
• (G ~ ◦ f )0 (t) = −t , 0 −t

, cosh e −e
e−t
= −1, −e−t cosh e−t , 0

6.2.3 Integrals of Vector Functions

ic s
~ (t) = hx (t) , y (t) , z (t)i. The indefinite integral of R
~ (t) is given by

at
Definition 6.2.11. Let R

em
Z Z Z Z 
~
R (t) dt = x (t) dt, y (t) dt, z (t) dt h
at
M

~ (t) is given by
and the definite integral from t = a to t = b of a continuous vector function R
of

Z b Z b Z b Z b 
~ (t) dt =
e

R x (t) dt, y (t) dt, z (t) dt .


ut

a a a a
tit

R1
~ dt, where R(t)
~ = 1 − cos t, 2t + 3, 4t3 − 2 .


Example 6.2.12. Evaluate R(t)
s

0
In

Solution. Applying Definition 6.2.11, we have


UP

Z 1 Z 1 Z 1 Z 1 
~ 3

R(t) dt = (1 − cos t) dt, (2t + 3) dt, 4t − 2 dt
0
 0 1
0 0

 1  1
= (t − sin t) , t2 + 3t , t4 − 2t

0 0 0

= h1 − sin 1, 4, −1i.

~ ~ 0 (t) = 3t2 − 1, 6 sin 2t, 2e2t and R(0)


~


Example 6.2.13. Find R(t) given that R = h1, 0, −3i.

~ 0:
Solution. We first find an antiderivative for R
Z
~
R(t) = R ~ 0 (t) dt
Z Z Z 
2 2t

= 3t − 1 dt, 6 sin 2tdt, 2e dt

= t3 − t + C1 , −3 cos 2t + C2 , e2t + C3 , C1 , C2 , C3 are constants.




6.2. CALCULUS OF VECTOR FUNCTIONS 141

~
Now, we determine the values of C1 , C2 , and C3 using R(0) = h1, 0, −3i. We have
~
R(0) = hC1 , −3 + C2 , 1 + C3 i = h1, 0, −3i,

and we obtain
C1 = 1, C2 = 3, and C3 = −4.
~ = t3 − t + 1, −3 cos 2t + 3, e2t − 4 .


Therefore, R(t)

EXERCISES. Do as indicated.
~ ln(2t + 1) sin t cosh t ~
1. Let R(t) = ı̂ + ̂ + 2 k̂. Evaluate lim R(t).
t t t −4 t→0
 3
4t + 2t2 − 7

~ ln t
2. Let F (t) = −1
, tan t, . Evaluate lim F~ (t).
t3 − 1 t t→+∞

~ be the vector-valued function defined piecewise by


3. Let R
 


sin 3t
4 − t, , ln(t + 1) , if t 6= 0

s


t

ic
~
R(t) = .

at

 h2, 3, 0i

, if t = 0
~
(a) Give domR. h em
at
~ is continous at t = 0.
(b) Determine whether R
M

~ be a vector-valued function defined by


4. Let R
of


h2, 1, −8i
e

, if t = 1
ut




~
R(t) = .
tit

ln t t2 + 6t − 7
 
 4
 √
 , , 6 1
, if t =
s

t + 3 t − 1 t2 − 3t + 2
In

~
(a) Find domR.
UP

(b) Determine whether R(t) ~ is continuous at t = 1 or not.


~ 0 (0) and R
~ 00 (0) if R(t)
~ = t − sin 3t, 5 − cosh(t2 ), ln 1 − t .


5. Evaluate R

6. Let F~ and G
~ be vector-valued functions such that

F~ (t) = hcos(πt), e2t−1 , t2 − 1i, G(1)


~ ~ 0 (1) = h2, 3, 2i, G
= h1, 1, −1i, G ~ 00 (1) = h0, 1, 0i.

(a) Find a vector equation of the tangent line to the graph of F~ at (−1, e, 0).
 0
(b) Evaluate F~ · G
~ (1).
 0
(c) Evaluate G ~ ×G~ 0 (1).
Z 2
~
7. Let R(t) = he2t , 2 − 3t2 , t ln ti. Evaluate ~ dt.
R(t)
1

~
8. Determine R(t) ~ 0 (t) = 2e2t−4 ı̂ + 6 ̂ + 1 k̂ and R(2)
if R ~ = h3, −5, 0i.
t2 t−1
142 CHAPTER 6. VECTOR-VALUED FUNCTIONS

6.3 The Moving Trihedral


Consider a particle that moves in space along a smooth curved path C defined by some vector
~
function R(t). We characterize its path using three unit vectors, namely unit tangent, unit normal
and unit binormal vectors.

6.3.1 Unit Tangent Vector


~
Definition 6.3.1. Let C be a smooth curve defined by a vector-valued function R(t). The unit
tangent vector to the curve C is defined by
~ 0 (t)
R
T~ (t) = .
~ 0 (t)k
kR
~ 0 (t).
Remark 6.3.2. The unit tangent vector points in the same direction as R

Example 6.3.3. A particle moves on a plane such that its path is given by R(t) ~ = ht2 − 1, ti.
Determine the unit tangent vector at t = −2, t = − 21 , t = 1, and t = 2.

ic s

~ 0 (t) = h2t, 1i and kR
Solution. Since R ~ 0 (t)k = 4t2 + 1, we have

at
em
 
~ 1 2t 1
T (t) = √ h2t, 1i = √ , √ h .
4t2 + 1 4t2 + 1 4t2 + 1
at
M

We obtain the following:


of

 
~ −4 1
T (−2) = √ , √
e
ut

17 17
   
1 −1 1
tit

~
T − = √ ,√
2 2 2 Fig-
s
In

 
~ 2 1
T (1) = √ , √
UP

5 5
 
~ 4 1
T (2) = √ , √ Figure 6.6: Particle’s path and some
17 17
unit tangent vectors

ure 6.6 shows the graph of the particles’s path and the unit tangent vectors at the indicated values
of t.
~
Example 6.3.4. Let R(t) = hln t, ti. Find T~ (2).
 
~ = (0, +∞). Differentiating R,
~ we get R
~ 0 (t) = 1
Solution. Observe that domR , 1 . Moreover,
t
r
~ 0 (t)k = 1 1p 2 =
1p
kR + 1 = 1 + t 1 + t2 , since t > 0.
t2 |t| t
Therefore,    
1 t 1 2
T~ (t) = √ ,√ and T~ (2) = √ ,√ .
1+t2 1 + t2 5 5
6.3. THE MOVING TRIHEDRAL 143

~
In general, R(t) and R~ 0 (t) are not perpendicular, as illustrated in Figure 6.7. But in the case that
~
kR(t)k ~
is constant, R(t) ~ 0 (t). We prove this property in the succeeding
is always perpendicular to R
theorem.

~
Figure 6.7: Non-perpendicular R(t) ~ 0 (t)
and R

~ 0 (t)k is constant for all t, then R(t)


Theorem 6.3.5. If kR ~ ·R ~ 0 (t) = 0.

s
~
Proof. Suppose that kR(t)k ~
is constant, i.e., for all t, kR(t)k = k, where k is some nonnegative

ic
at
~ ~ ~ 2 ~ ~ 2
constant. Since R(t) · R(t) = kR(t)k , we have R(t) · R(t) = k . Differentiating both sides of this

em
equation with respect to t, we get
h
at
d h~ ~
i
R(t) · R(t) =0
M

dt
~ 0 (t) · R(t)
R ~ + R(t)
~ ·R ~ 0 (t) = 0
of

~ 0 (t) · R(t)
2R ~ =0
e
ut

~ 0 (t) · R(t)
R ~ = 0.
s tit
In
UP

~
Example 6.3.6. Consider the unit circle R(t) ~
= hcos t, sin ti. Since kR(t)k = 1 for all t, i.e.,
~
kR(t)k ~
= 1 is constant, then R(t) ~ (t) are always perpendicular.
and R 0

~
Figure 6.8: R(t) = hcos t, sin ti and T~ (t) for t = 7π 23π
15 , π, 15

Figure 6.8 shows some tangent vectors R ~ 0 (t) to the unit circle. It can be seen that these vectors
~
are perpendicular to the position representation of R(t).
144 CHAPTER 6. VECTOR-VALUED FUNCTIONS

6.3.2 Unit Normal Vector


Consider the vector T~ 0 (t). By Theorem 6.3.5, the vectors T~ (t) and T~ 0 (t) are perpendicular since
kT~ (t)k = 1 for all t. Note, however, that T~ 0 (t) is not always a unit vector.

~
Definition 6.3.7. Let C be a smooth curve defined by the vector function R(t) with unit tangent
~
vector T (t). The vector defined by
~0
~ (t) = T (t) ,
N
kT~ 0 (t)k
provided T~ 0 (t) 6= ~0, is called the unit normal vector to C.

~
Example 6.3.8. Consider the unit circle defined by R(t) = hcos t, sin ti. Then

~ 0 (t) = h− sin t, cos ti and kR


~ 0 (t)k =
p
R (− sin t)2 + (cos t)2 = 1.

So the unit tangent vector is T~ (t) = h− sin t, cos ti. Differentiating this, we get

s
T~ 0 (t) = h− cos t, − sin ti,

ic
at
which is not the zero vector for any t. Moreover, kT~ 0 (t)k =
p
(− cos t)2 + (− sin t)2 = 1. Hence,
the unit normal vector for any t is h em
at
~ (t) = h− cos t, − sin ti.
M

N
of

~
Example 6.3.9. Consider R(t) ~ (2).
= hln t, ti. Find N
e
ut
tit

Solution. From Example 6.3.4, we have


s
In

 
~ 1 t
T (t) = √ , √ .
1 + t2 1 + t2
UP

Differentiating T~ and evaluating the derivative of T~ at t = 2,


we get
 
0 t 1
T~ (t) = − ,
(1 + t2 )3/2 (1 + t2 )3/2
 
~ 0 2 1
T (2) = − √ ,√ .
125 125
Hence,

* √ √ + Figure 6.9: T~ (2) and N


~ (2) for R(t)
~
 
2 1 = hln t, ti
~ 0 −√ ,√
~ (2) = T (2) = 
N
125 125 
= −
2 5 5
, .
~ 0
kT (2)k
−√ 2 1 5 5
,√
125 125

~ at t = 2.
Figure 6.9. shows the unit tangent and unit normal vectors to the graph of R
6.3. THE MOVING TRIHEDRAL 145

Remark 6.3.10.

1. At every point on a smooth space curve, the unit tangent and unit normal vectors are per-
pendicular.

2. The unit normal vector points towards the concave side of the curve.

6.3.3 Unit Binormal Vector


Consider a smooth space curve C defined by a vector-valued function R(t) ~ with unit tangent and
~ ~ ~ ~ ~ ~ ~
unit normal vectors T (t) and N (t). Let B(t) = T (t) × N (t). Since T (t) and N (t) are perpendicular
unit vectors, then
~
kB(t)k = kT~ (t) × N ~ (t)k sin π = 1,
~ (t)k = kT~ (t)kkN
2
~
which implies that B(t) is also a unit vector.

~
Definition 6.3.11. The vector B(t) = T~ (t) × N
~ (t) is called the unit binormal vector to C.

ic s
~
Example 6.3.12. Consider R(t) ~
= hln t, ti. Find B(2).

at
em
Solution. From Example 6.3.4 and Example 6.3.9, we have
 
h  
at
~ 1 2 ~ 2 1
T (2) = √ , √ and N (2) = − √ , √ .
M

5 5 5 5
of

Hence,
~ = T~ (2) × N
~ (2) = h0, 0, 1i .
e

B(2)
ut
s tit
In

~
Remark 6.3.13. Let C be a smooth curve define by a vector-valued function R(t). The unit
UP

binormal vector can also be computed using the formula

~ 0 (t) × R
R ~ 00 (t)
~
B(t) = .
~ 0 (t) × R
kR ~ 00 (t)k

Indeed, recall that

~
B(t) = T~ (t) × N
~ (t)
~ 0 (t)
R T~ 0 (t)
= ×
~ 0 (t)k kT~ 0 (t)k
kR
~ 0 (t) × T~ 0 (t)
R
= ~ 0 (t) ⊥ T~ 0 (t).
, since R
~ 0 ~ 0
kR (t) × T (t)k

Meanwhile,
~ 0 (t)
R
T~ (t) = .
~ 0 (t)k
kR
146 CHAPTER 6. VECTOR-VALUED FUNCTIONS

Differentiating both sides with respect to t, we have


h i
~ 0 (t) d ~ 0
~ 00 (t)
R R dt kR (t)k
T~ 0 (t) = − .
~ 0 (t)k
kR ~ 0 (t)k2
kR
Thus,
~0 ~ 00
~ 0 (t) × T~ 0 (t) = R (t) × R (t)
R
~ 0 (t)k
kR
~0 ~ 00
~ 0 (t) × T~ 0 (t)k = kR (t) × R (t)k .
kR
kR~ 0 (t)k
Hence,
~ 0 (t) × R
R ~ 00 (t)
~
B(t) = .
~ 0 (t) × R
kR ~ 00 (t)k

6.3.4 Frenet Frame of Space Curves


Collectively, the three unit vectors T~ (t), N
~ (t), and B(t)
~ are called the moving trihedral of a

ic s
space curve.

at
em
Remark 6.3.14. The vectors T~ (t), N ~ (t), and B(t)
~ are mutually perpendicular, and follow the
right-hand rule. It can be shown that h
at
~
B(t) = T~ (t) × N
~ (t)
M

~ (t) = B(t)
N ~ × T~ (t)
of

T~ (t) = N
~ (t) × B(t).
~
e
ut

 
1 2 1 3
tit

~
Example 6.3.15. Let R(t) = t, t , t . Compute the vectors T~ , N ~ , and B
~ at t = 1.
2 3
s
In

~ 0 (t). We have R ~ 0 (t) = 1, t, t2 , which gives




Solution. We first compute for R
UP


~ 0 (1) = h1, 1, 1i and kR
R ~ 0 (1)k = 3.

~ 00 (t) = h0, 1, 2ti, so


Furthermore, R

~ 00 (1) = h0, 1, 2i and kR
R ~ 00 (1)k = 5.

Therefore,
 
1 1 1 1
T~ (1) = √ h1, 1, 1i = √ ,√ ,√ .
3 3 3 3
By Remark 6.3.13,
R~ 0 (1) × R
~ 00 (1) h1, 1, 1i × h0, 1, 2i
~
B(1) = =
~ 0 ~ 00
kR (1) × R (1)k k h1, 1, 1i × h0, 1, 2i k
*√ √ √ +
h1, −2, 1i 6 6 6
= = ,− , .
k h1, −2, 1i k 6 3 6
6.3. THE MOVING TRIHEDRAL 147

Finally, using Remark 6.3.14,


* √ √ +
~ (1) = B(1)
~ 2 2
N × T~ (1) = − , 0, .
2 2

Remark 6.3.16.

1. The three mutually perpendicular vectors T~ , N


~ and B
~ determine a coordinate system in the
three-dimensional space, which is called Frenet frame or simply TNB frame.

2. At a point P on a space curve C, the plane formed by the unit tangent and the unit normal
vectors is called the osculating plane of C at P . The plane formed by the unit tangent and
the unit binormal vectors is called the rectifying plane of C at P . Finally, the plane formed
by the unit normal and the unit binormal vectors is called the normal plane of C at P .

ic s
at
h em
at
M
of
e
ut
tit

Figure 6.10: Frenet Frame


s
In

~ (t) = h2 cos t, 2 sin t, ti.


Example 6.3.17. Let C be the space curve defined by R
UP


1. Find the moving trihedral of C at t = 4 .


2. Give equations of the osculating, rectifying, and normal planes of C at the point where t = 4 .

Solution.

~
1. Given R(t) ~ 0 (t) = h−2 sin t, 2 cos t, 1i and kR
= h2 cos t, 2 sin t, ti, we have R ~ 0 (t)k = 5. Thus,
 
~ 2 2 1
T (t) = − √ sin t, √ cos t, √ .
5 5 5
So we have   * √ √ √ +
3π 10 10 5
T~ = − ,− , .
4 5 5 5
Moreover,  
2 2 2
T~ 0 (t) = − √ cos t, − √ sin t, 0 and kT~ 0 (t)k = √ .
5 5 5
148 CHAPTER 6. VECTOR-VALUED FUNCTIONS

Hence,
~ (t) = h− cos t, − sin t, 0i ,
N

so   *√ √ +
~ 3π 2 2
N = ,− ,0 .
4 2 2
Finally, *√
      √ √ +
~ 3π 3π 3π 10 10 2 5
B = T~ ~
×N = , , .
4 4 4 10 10 5
√ √ 3π 
2. Note that the point on C where t = 3π 4 is P − 2, 2, 4 . Thus,
√ √ √ 
√ √

10 10 2 5 3π
osculating plane : (x + 2) + (y − 2) + z− =0
10 10 5 4
√ √
2 √ 2 √
rectifying plane : (x + 2) − (y − 2) = 0
√2 2
√ √ 
√ √

10 10 5 3π
normal plane : − (x + 2) − (y − 2) + z− = 0.

s
5 5 5 4

ic
at
~ and its moving trihedral at t =
Figure 6.11 shows the graph of R 3π
4 .
hem
at
M
of
e
ut
s tit
In
UP

~
Figure 6.11: The TNB frame of R(t) = h2 cos t, 2 sin t, ti at t = 3π
4

EXERCISES. Do as indicated.
*√ √ +
~ be a vector function such that T~ (t) = 3 1 3 ~
1. Let R cos 2t, , sin 2t and R(0) = h1, 0, −1i.
2 2 2
Find:

~ at t = 0.
(a) the unit tangent, unit normal, and unit binormal vectors to the graph of R
(b) an equation of the osculating, rectifying, and normal planes to the graph of R~ at t = 0.
* √ √ √ +
~ 6 6 6
2. Given R(π) = h1, 4, 3i, T~ 0 (π) = h−1, −1, 0i and B(π)
~ = − , , . Find:
6 6 3
6.4. ARC LENGTH AND PARAMETRIZATION USING ARC LENGTH 149

(a) the unit tangent vector T~ at t = π.


(b) the equation of the rectifying plane at t = π.

3. Let the smooth curve C be the graph of the vector function R ~ such that R(0)
~ = h3, −1, 2i,
~ 0 ~ 00
R (0) = h2, 2, 1i, and R (0) = h1, 1, −1i. At the point on C corresponding to t = 0, give the
following:

(a) equation of normal plane


(b) unit binormal vector
~
4. Find the point at which the unit tangent vector of the space curve defined by R(t) = ht2 +
2, t2 − t, ti is parallel to the plane 2x − y + 3z = 5.

5. Prove the identities in Remark 6.3.14.

6.4 Arc length and Parametrization using Arc length

ic s
6.4.1 Arc length of Space Curves

at
em
Recall that for a given smooth plane curve with parametric equations x = x(t), y = y(t), a 6 t 6 b,
the length of the curve is given by h
at
Z bq
(x0 (t))2 + (y 0 (t))2 dt.
M

L=
a
of
e
ut

The length of a smooth space curve is defined in


~ (t) = hx (t) , y (t) , z (t)i, whose
tit

the same way. Let R


s

graph is smooth and is traced exactly once from t = a


In

to t = b. By choosing values of t between a and b,


UP

we can partition the curve into subarcs. The length


of each subarc can be approximated by the length of
the line segment joining the endpoints of the subarc,
as shown in Figure 6.12. If L denotes the length of
the curve from t = a to t = b, we have
s
n
∆xi 2 ∆yi 2 ∆zi 2
X     
L≈ + + ∆ti .
∆ti ∆ti ∆ti
i=1 Figure 6.12: The length of a space curve as
limit of the sum of lengths of line segments

To lessen the error of this approximation, we let n → +∞ in such a way that ∆ti → 0 for
i = 1, 2, . . . , n. Thus, the length L of the curve on [a, b] is
s
n
∆xi 2 ∆yi 2 ∆zi 2
X      Z bq
L = lim + + ∆ti = (x0 (t))2 + (y 0 (t))2 + (z 0 (t))2 dt.
n→+∞ ∆ti ∆ti ∆ti a
i=1
150 CHAPTER 6. VECTOR-VALUED FUNCTIONS

Theorem 6.4.1. Let R(t) ~ ~ is traced exactly once on


= hx (t) , y (t) , z (t)i such that the graph of R
~ (t) 6= ~0, for all t ∈ [a, b]. The length of the the graph of R
[a, b] and R 0 ~ on [a, b] is given by
Z bq Z b
2 2 2
0
L= 0 0 0
(x (t)) + (y (t)) + (z (t)) dt = ~ (t) dt.
R
a a

~ (t) = 4t3/2 , −3 sin t, 3 cos t , t ∈ [0, 2].




Example 6.4.2. Find the length of the curve defined by R
~ 0 (t) = 6t1/2 , −3 cos t, −3 sin t . We then have


Solution. Note that R
Z 2q
2
L= 6t1/2 + (−3 cos t)2 + (−3 sin t)2 dt
0

Z 2
= 36t + 9dt
0
1 t=2
= (4t + 1) /2
3

2 t=0
1
= (27 − 1) = 13.
2

s
Therefore, the length of the curve is 13 units.

ic
at
em
6.4.2 Parametrizing Vector Functions Using Arclength
~
Given a vector-valued function R(t),
h
the parameter t is an arbitrary parameter and may possess no
at
geometric significance. In most cases, specially in physics, it is convenient to give a representation
M

~
of a space curve in terms of the arc length. We now illustrate how a given vector function R(t) can
of

be reparametrized, i.e., expressed in terms of another parameter, in particular, using the arc length
e
ut

as parameter.
tit

~
s

Recall that the arc length L of a smooth curve R(t) = hx(t), y(t), z(t)i , a ≤ t ≤ b is given by
In

Z b
UP

L= kR~ 0 (t)k dt.


a

~
Given a smooth curve R(t) = hx(t), y(t), z(t)i, we choose a parameter value t0 and consider
Z t
s= ~ 0 (u)k du.
kR
t0

~
This represents the directed arc length of the portion of the graph of R(t) from the point at t0 to
the point at t, and by directed we mean that s > 0. Note that s is a differentiable function of t with
ds ~ 0 (u)k > 0. Thus, s is an increasing function of t, and hence, s is a one-to-one function of
= kR
dt
t. This implies that we may express t as a differentiable function of s, say t = ϕ (s). With this, we
~
may express R(t) in terms of s via
~
R(t) ~
= R(ϕ(s)) ~ ∗ (s).
=R
~ ∗ (s) the arc length parametrization of R.
We call R ~
6.4. ARC LENGTH AND PARAMETRIZATION USING ARC LENGTH 151

~ (t) = ht, 2t, −2ti using the arc length as


Example 6.4.3. Reparametrize the line given by R
parameter from the point P (0, 0, 0).

~ (t0 ). It is clear that t0 = 0. We have


Solution. First, we find t0 such that P (0, 0, 0) = R
Z t
s= ~ 0 (u) kdu, where R
kR ~ (u) = hu, 2u, −2ui
0
Z t Z t
= kh1, 2, −2ik du = 3 du = 3t.
0 0
 
s ~ (t), we get R
~ ∗ (s) = s 2s −2s
Hence, t = . Substituting in R , , .
3 3 3 3
Remark 6.4.4. Geometrically, when reparametrizing using the arc length parameter, i.e., express-
~ ~ ∗ (s) using s = t kR~ 0 (u)k du, we are wrapping the real number line along the given
R
ing R(t) as R t0
curve in such a way that

• the origin of the real number line coincides with the point on the curve corresponding to t0 ,

ic s
and

at
• the real number line is oriented in the direction of the increasing parameter t.
h em
Hence, the arc length parameter measures distance along the curve starting at the given reference
at
point t0 .
M
of

Example 6.4.5. Reparametrize R(t) ~ = hcos t, sin t, ti using the arc length as parameter from the
e

point P (1, 0, 0). Use this reparametrization to find the point Q on the graph of R if the length of
ut


the arc from P to Q (in the direction of increasing parameter) is 2 π units.
s tit

Solution. It can be easily checked that the point P (1, 0, 0) corresponds to t0 = 0. We then have
In
UP

Z t Z tp √
s= ~ 0 (u)k du =
kR (− sin u)2 + (cos u)2 + 12 du = 2 t.
t0 0

s
Hence, t = √ . Thus,
2       
~ ∗ (s) = s s s
R cos √ , sin √ , √ .
2 2 2
To find the coordinates of Q, we compute

~ ∗ ( 2 π) = hcos π, sin π, πi = h−1, 0, πi.
R

Hence, Q has coordinates (−1, 0, π).

~ (s) = 1.
∗0
Remark 6.4.6. The arc length parametrization of a space curve is unit-speed, i.e., R
Thus, T~ (s) = R
~ ∗ 0 (s).

EXERCISES. Do as indicated.
152 CHAPTER 6. VECTOR-VALUED FUNCTIONS

1. Find the length of the graph of the following:

~
(a) R(t) = h3 sin t, 4t, 3 cos ti, t ∈ [−1, 2]
~ = 2t, t2 , 32 t3 , t ∈ [0, 1]


(b) R(t)

~
2. Reparametrize R(t) = hcos t, sin t, cosh ti with respect to the arclength measured from t = 0
in the direction of increasing t.

~
3. Let C be the curve defined by R(t) = h3 cos t, 4t, 3 sin ti.

(a) Reparametrize the curve C in terms of the arclength s measured from the point where
t = 0 in the direction of increasing values of t.
(b) Find the coordinates of point P on R ~ if P is 10 units away from the point where t = 0
measured in the direction of increasing values of t.

ic s
6.5 Curvature

at
em
For a smooth space curve, one can look at how bent a curve is at any point on the curve. This
h
scalar quantity is known as curvature. We do this by determining how the unit tangent vector T~
at
changes with respect to arc length. Consider two circles with different radii as shown in Figure
M

6.13. Over the same length of arc, we observe that the angle ∆θ between the tangent vectors is
of

larger on the smaller circle than the larger circle. So the unit tangent vector of the smaller circle
e

changes its direction faster than the unit tangent vector of the larger circle. We can say that the
ut
tit

smaller circle has greater curvature than the larger circle.


s
In
UP

Figure 6.13: Two circles of different radii

Definition 6.5.1. The curvature κ of a space curve is the magnitude of the rate of change of the
unit tangent vector with respect to the arc length. That is,

dT~
κ = ,

ds

where T~ is the unit tangent vector to the curve and s is the arclength parameter.
6.5. CURVATURE 153

Example 6.5.2. Let us show that a line has zero curvature. Suppose that the line passes through
6 ~0. Then line has vector equation
the point (x0 , y0 , z0 ) and has direction vector ha, b, ci =

~
R(t) = hx + 0 + at, y0 + bt, z0 + cti,

~ 0 (t) = ha, b, ci. Thus,


so R
Z t
s= ~ 0 (u)k du
kR
t0
Z tp
= a2 + b2 + c2 du
t0
p
= a2 + b2 + c2 (t − t0 ) .

This implies
s
t = t0 + √ .
a2 + b2 + c2
The arc length parametrization of the line is

ic s
      
~ ∗ s s s
R (s) = x0 + a t0 + √ , y0 + b t0 + √ , z0 + c t0 + √

at
.
a2 + b2 + c2 a2 + b2 + c2 a2 + b2 + c2
By Remark 6.4.6, h em
at
 
~ ∗ 0 (s) = a b c
T~ (s) = R
M

√ ,√ ,√ ,
2 2
a +b +c2 2 2
a +b +c2 a + b2 + c2
2
of

dT~ ~ dT~
e

which is a constant vector function. Hence, = 0 and κ = = 0.



ut

ds ds
tit
s

To find the curvature of a smooth space curve defined by a vector-valued function R ~ (t), we first
In

parametrize the vector-valued function using the arc length parameter s from an initial point P ,
UP

which is not often straightforward. Thus, it will be more convenient if we can find a formula for
the curvature in terms of the original parameter t.

By chain rule, we have


dT~ dT~ ds
= ·
dt ds dt
thus getting
dT~ dT~ /dt
κ= =

ds ds/dt

ds ~ 0 (t)k, we obtain
but since = kR
dt
kT~ 0 (t)k
κ(t) = .
kR~ 0 (t)k
154 CHAPTER 6. VECTOR-VALUED FUNCTIONS

Example 6.5.3. Show that the curvature of a circle is constant and is equal to the reciprocal of
its radius.

Solution. Consider a circle of radius a > 0. For simplicity, we consider the circle centered at the
origin, i.e., the graph of the vector-valued function R ~ (t) = ha cos t, a sin t, 0i. The unit tangent
vector is
~ 0 (t)
R h−a sin t, a cos t, 0i
T~ (t) = =q = h− sin t, cos t, 0i .
~ 0
kR (t) k 2 2 2
(−a sin t) + (a cos t) + 0
Differentiating with respect to t, we get

T~ 0 (t) = h− cos t, − sin t, 0i .

Therefore, q
~ 0
kT (t) k (− cos t)2 + (− sin t)2 + 02 1
κ (t) = = = .
~
q
0
kR (t) k a
(−a sin t)2 + (a cos t)2 + 02
This result shows that smaller circles have greater curvature than larger circles.

ic s
at
~
We now give an alternative formula for computing the curvature using only the derivatives of R.
h em
Theorem 6.5.4. The curvature of a smooth space curve defined by the vector valued function
at
~
R(t) is given by
M

~ 0 (t) × R
kR ~ 00 (t) k
κ(t) = .
of

kR~ 0 (t) k3
e

~ 0 kT~ = ds T~ . Applying product rule, we get R


ut

Proof. Note that R~ 0 = kR ~ 00 = ds T~ 0 + T~ d22s .


dt dt dt
tit

Hence,
s
In

ds ~ 0 ~ d2 s ds ~ 0 ds ~ ~ d2 s
 
0 00
ds ds
~ ~
kR × R k = T × ~ ~
T +T 2 = T × T + T ×T 2

UP

dt dt dt dt dt dt dt
 
ds 2 
ds
 
d2 s ~ ~

= T~ × T~ 0 + T × T

dt dt dt2
   
ds 2 ds 2 ~ ~ 0
= T~ × T~ 0 = kT × T k

dt dt
 2  2
ds 0 ds
= ~ ~
kT kkT k sin θ = kT~ 0 k
dt dt

since kT~ k = 1 and T~ 0 ⊥ T~ . Therefore, we have


ds 2 ~ 0 ds 2 ~ 0
kT~ 0 (t)k ~ 0 (t) × R ~ 00 (t) k
 
dt kT (t)k dt kT (t)k kR
κ(t) = = 3 = = .
kR~ 0 (t)k ds

kT~ (t)k3 k ds ~ 3 kR~ 0 (t) k3
dt dt T (t)k

~ = t, t2 , t3 .


Example 6.5.5. Find the curvature of the twisted cubic R(t)
6.5. CURVATURE 155

Solution. From the given, we have



~ 0 (t) = 1, 2t, 3t2 , kR ~ 00 (t) = h0, 2, 6ti.
~ 0 (t)k = 1 + 4t2 + 9t4 , and R


R

Thus, p
~ 0 (t) × R
~ 00 (t) = 6t2 , −6t, 2 and kR
~ 0 (t) × R
~ 00 (t)k = 36t4 + 36t2 + 4.


R

Hence, √
~ 0 (t) × R
kR ~ 00 (t) k 36t4 + 36t2 + 4
κ(t) = = .
kR~ 0 (t) k3 (1 + 4t2 + 9t4 )3/2

1
In a previous example, we have seen that a circle with radius a > 0 has constant curvature .
a
1
Equivalently, a circle with curvature κ > 0 has radius . Suppose that a smooth space curve C
κ
defined by R~ (t) has curvature κ at the point P corresponding to t = t0 . We can construct a circle
of radius ρ = κ1 through P that is

• contained in the osculating plane of C at P ,

ic s
at
• tangent to R
~ 0 (t0 ) at P , and

h em
• centered at the tip of the position representation R(t
~ 0 ) + ρN
~ (t0 ).
at
M

This is called the osculating circle of the curve C at P and its radius ρ is called the radius of
curvature.
of
e
ut
s tit
In
UP

Figure 6.14: A curve with its osculating circle at P

As evident in Figure 6.5, the osculating circle to C at P almost coincides with C at points near
P . This means that the osculating circle ”approximates” the points on C near P . Moreover, the
osculating circle lies on the osculating plane of C at P .

~
Example 6.5.6. Find the radius of curvature of R(t) = ht, ln t, 2i at the point (1, 0, 2).
156 CHAPTER 6. VECTOR-VALUED FUNCTIONS

Solution. The point (1, 0, 2) corresponds to t = 1. Therefore,

 
1
~ 0 (t) 1, , 0  
~ R t t 1
T (t) = = 
 = √ ,√ ,0
~ 0 (t)k
kR 1, 1 , 0
1 + t2 1 + t2
t
 
1 −1


√ , √ ,0
~ 0
kT (1)k 2 2 2 2 2
κ(1) = = = .
~ 0
kR (1)k kh1, 1, 0ik 4

1 4 √
Hence, ρ(1) = = √ = 2 2.
κ(1) 2

ic s
at
h em
at
M
of
e
ut
s tit
In

~
Figure 6.15: Osculating circle of R(t) = ht, ln t, 2i at the point (1, 0, 2)
UP

~
Example 6.5.7. Find the radius of curvature of R(t) = h2 sin t, 2 cos t, ti at any point.

Solution.

R~ 0 (t) h2 cos t, −2 sin t, 0i


T~ (t) = = = hcos t, − sin t, 0i
kR~ 0 (t)k kh2 cos t, −2 sin t, 0ik
kT~ 0 (t)k kh− sin t, − cos t, 0ik 1
κ(t) = = =
kR~ 0 (t)k kh2 cos t, −2 sin t, 0ik 2
1
∴ ρ(t) = =2
κ(t)
6.6. MOTION IN SPACE 157

~
Figure 6.16: An osculating circle and osculating plane of R(t) = h2 sin t, 2 cos t, ti

ic s
EXERCISES. Do as indicated.

at
~ be a vector function such that

em
1. Let R

~
R(e) = h2, −1, 4i, R
h
~ 0 (e) = h1, −1, 1i, R
~ 00 (e) = h0, 1, −1i.
at
M

Find the curvature of the graph of R ~ at t = e.


of

~ 0 (0) = h4, 0, 3i and T~ (t) = 4 cos 5t, − 4 sin 5t, 3 . Find the curvature of the graph of


2. Given R 5 5 5
e

~ at t = 0.
ut

R
tit

~ π
3. Find radius of curvature of the graph of R(t) = (sin t + cos t)ı̂ + t̂ + (sin t − cos t)k̂, at t = .
s

2
In
UP

6.6 Motion in Space


Suppose an object (viewed as a point) moves in space. Its path of motion can be characterized by
some vector-valued function of the time parameter t. If at any given time t, the object is at the
~
the tip of the position representation of R(t), ~ the position function of the object.
we call R

6.6.1 Basic Definitions


As the object moves from t = a to t = b on its path,
Rb
• the distance it has traveled is s = a kR~ 0 (t)kdt, and

• its displacement is the vector R(b)


~ ~
− R(a).

For an arbitrary time t, consider a time interval I = [t, t + ∆t]. Then


s(t + ∆t) − s(t)
• the average speed on the interval I is ,
∆t
158 CHAPTER 6. VECTOR-VALUED FUNCTIONS

~ + ∆t) − R(t)
R(t ~
• while the average velocity on the interval I is .
∆t
If we let ∆t → 0, we get

• the instantaneous speed


s(t + ∆t) − s(t) ds ~ 0 (t)k,
v(t) = lim = = kR
∆t→0 ∆t dt

• and the instantaneous velocity


~ ~
~ (t) = lim R(t + ∆t) − R(t) = R
V ~ 0 (t).
∆t→0 ∆t

Moreover, the rate of change of the velocity at a given time is called the instantaneous acceler-
ation of the object. That is,
~ ~
~ = lim V (t + ∆t) − V (t) = V
A(t) ~ 0 (t) = R
~ 00 (t).
∆t→0 ∆t

ics
at
Example 6.6.1. An object moves in space with position function R h em
~ (t) = hcos t, sin t, 0i. Determine
at
the velocity and acceleration of the object at time t = π and the distance it has traveled from t = π
M

to t = 3π.
of

Solution. Differentiating the position function with respect to t, we get


e
ut

~ 0 (t) = h− sin t, cos t, 0i .


~ (t) = R
V
s tit

Differentiating further, we have


In

~ 0 (t) = h− cos t, − sin t, 0i .


~ (t) = V
UP

~ (π) = h0, −1, 0i and A


Thus, V ~ (π) = h1, 0, 0i .
Furthermore, the distance traveled from t = π to t = 3π is
Z 3π 3π
s= kh− sin t, cos t, 0ik dt = t = 2π.

π π

Therefore, the object has travelled a distance of 2π units.

~ (t) = h4 cos t, 4 sin t, 3ti, where


Example 6.6.2. An object moves in space with position function R
t is in seconds. Find:

1. its speed at any time,

2. the distance traveled from t = 0 to t = π, and

3. the displacement from t = 0 to t = π.


6.6. MOTION IN SPACE 159

Solution.

1. The velocity vector at any time t is V~ (t) = R


~ 0 (t) = h−4 sin t, 4 cos t, 3i. So the speed at any
ds ~ 0 (t)k = 5 units/sec.
time t is = kR
dt
Z π Z π
~ 0
2. The distance traveled from t = 0 to t = π is s = kR (t)kdt = 5 dt = 5π units.
0 0

~
3. The displacement from t = 0 to t = π is = R(π) ~
− R(0) = h−4, 0, 3πi − h4, 0, 0i = h−8, 0, 3πi.

Example 6.6.3. The velocity of a moving object at any time t is given by

~ (t) = 1, 2t, 3t2 .




V

Find the position of the object at t = 3 if it is at the point (−1, 2, 3) when t = 1.

~
Solution. First, we recover R(t) ~ (t) = R
using the fact that V ~ 0 (t). So,

ic s
at
Z Z
~ ~ 1, 2t, 3t2 dt


R (t) = V (t) dt =

h em
= t + C1 , t2 + C2 , t3 + C3 .

at
M

~
It is given that R(1) = h−1, 2, 3i. Thus,
of

~ (1) = h1 + C1 , 1 + C2 , 1 + C3 i = h−1, 2, 3i .
R
e
ut
tit

~ (t) = t − 2, t2 + 1, t3 + 2 .


This implies C1 = −2, C2 = 1 and C3 = 2, thereby giving R
s

~ (3) = h1, 10, 29i, and so the object is at the point (1, 10, 29) when t = 3.
Hence, R
In
UP

6.6.2 Tangential and Normal Components of Acceleration

Consider a passenger on a moving car.

• If the car speeds up rapidly on a straight path, the passenger’s body is thrown back against
the car seat’s backrest.

• As the car turns on a curved path, the passenger’s body is thrown toward the outside of the
curve of the road. If the curvature is greater, the more that this effect is experienced.

These situations can be understood more clearly by considering the components of the velocity and
acceleration along the unit tangent and unit normal vectors.

First observe that  


~ 0 (t) = kR
~ (t) = R ~ 0 (t)kT~ (t) = ds
V T~ (t).
dt
160 CHAPTER 6. VECTOR-VALUED FUNCTIONS

So,
2s
  
0 d ds
~ (t) = V
A ~ (t) = T~ (t) + T~ 0 (t)
dt2 dt
 2   
d s ds
= ~
T (t) + kT~ 0 (t) k N
~ (t)
dt2 dt
 2 
d s ~ 0 (t) kkT~ 0 (t) k N
= T~ (t) + kR ~ (t)
dt2
 2  ~0
=
d s ~ 0 (t) k2 kT (t) k N
T~ (t) + kR ~ (t)
dt2 ~ 0 (t) k
kR
 2   2
d s ds
= T~ (t) + κ(t)N~ (t) .
dt2 dt

d2 s ds 2

If we let AT (t) = and AN (t) = dt κ(t), then
dt2
~ = AT (t) T~ (t) + AN (t) N
A(t) ~ (t).

ic s
at
We call the quantitites AT and AN , respectively, the scalar tangential component and scalar

em
normal component of acceleration.
h
Remark 6.6.4.
at
M

1. The scalar tangential component of acceleration gives the rate at which speed is changing
of

with respect to t.
e
ut

2. The scalar normal component of acceleration gives the rate at which velocity’s direction is
tit

changing with respect to t.


s
In

~
Let R(t) ~ (t) = R
be the position function of an object moving in space. Recall that the velocity V ~ 0 (t)
UP

points in the same as direction as the unit tangent vector T~ (t). If θ is the angle between the
~ and acceleration vector A,
velocity vector V ~ then the scalar tangential and normal components of
acceleration may also be computed as follows:

~ ~ ~ ~
~ cos θ = kV kkAk cos θ = V · A
AT = kAk
~k
kV kV~k
~ ~ ~ ~
~ sin θ = kV kkAk sin θ = kV × Ak
AN = kAk
kV~k ~k
kV

~ (t) = t, t2 , t3 . Find the




Example 6.6.5. An object moves in space with position function R
scalar tangential and normal components of the acceleration at t = 1.

Solution. We compute the velocity and acceleration at t = 1. The velocity vector at any t is
~ (t) = R
~ 0 (t) = 1, 2t, 3t2 . At t = 1, we have V
~ (1) = h1, 2, 3i. The acceleration vector at any t is


V
6.7. CHAPTER EXERCISES 161

~ (t) = V
A ~ 0 (t) = h0, 2, 6ti, and at t = 1, we have A(1)~ = h0, 2, 6i. Therefore, the scalar tangential
and normal components of the acceleration are

h1, 2, 3i · h0, 2, 6i 22 11 14
AT (1) = =√ =
kh1, 2, 3ik 14 7
√ √
kh1, 2, 3i × h0, 2, 6ik kh6, −6, 2ik 76 266
AN (1) = = √ =√ = .
kh1, 2, 3ik 14 14 7

EXERCISES. Do as indicated.
~
1. Suppose that the path of a moving object is define by the vector function R(t) = hsin 2t, t, − cos 2ti.

(a) Determine the velocity, speed, and acceleration at any time t.


(b) Find the displacement and the distance traveled by the object from the point (0, 0, 1) to
the point (0, π, −1).
(c) Find the scalar tangential and normal components of the acceleration at t = π4 .

ic s
~ (t) = he2t , 2et , 2i. The insect is at h−1, 1, 0i when
2. An insect moves in space with velocity V

at
em
t = 0. Find:
~
(a) the position vector R(t)
h
of the particle at any time t,
at
M

(b) the acceleration at any time t,


of

(c) the distance travelled by the insect from t = 0 to t = 1,


e

(d) the scalar tangential and normal components of the insect’s acceleration at t = 0.
ut
tit

~
3. A particle, initially at the point (1, 4, 3), moves in the space with acceleration A(t) =
s

h2, 2t, −2i. Its velocity at t = 1 is h2, −1, −2i.


In

~ ~
UP

(a) Find the position function R(t) and the acceleration vector A(t).
(b) At t = 3, find the scalar tangential and normal components of the acceleration.
(c) On the interval t ∈ [0, 3], determine the distance it has traveled and its displacement.

6.7 Chapter Exercises


I. Find parametric equations for the curve of intersection of the elliptic paraboloid z = 4x2 + y 2
and the parabolic cylinder z = 1 − 3y 2 .

II. Determine whether the vector-valued function


 2
t −9 √

3 t−3
, 3t − 1, , if t < 4, t 6= 3,


t−3 ln(4 − t)

F~ (t) =

h6, 2, 1i

, if t = 3

is continuous at t = 3.
162 CHAPTER 6. VECTOR-VALUED FUNCTIONS

III. Let C1 be a curve defined by some vector function R ~ with unit binormal vector B ~ such that
 
~ ~ 0 ~ 3 3 4
R(0) = h−2, 5, 4i , R (π) = h4, 0, 3i , and B (t) = cos t, sin t, .
5 5 5

1. Find the equation of the osculating plane to C1 at t = 0.


2. Find the unit normal vector to C1 at t = π.
 0
3. Evaluate R ~ ·B
~ (0).

~
IV. Let C2 be the curve defined by R(t) = h2e2t − 3, 1 − e2t , 2e2t i.

1. Find a vector equation of the tangent line to C2 at the point where t = 0.


~ ∗ (s) of C2 using the point P (−1, 0, 2) as reference
2. Find the arclength parametrization R
point.
3. If the arc on C2 from the point P to the point Q in the direction of increasing parameter
has length 3, determine the coordinates of Q.

ic s
V. An object, initially at the point A(1, 4, 3), moves in a way that its velocity any time t ≥ 0 is

at
em
 
~ 2 4
V (t) = 3t − 2, , 2t .
1 + t2
h
at
1. Find the coordinates of the position of the object at t = 1.
M
of

2. Determine the velocity, speed, and acceleration at t = 1.


e

3. Find the scalar tangential and normal components of the acceleration at t = 1.


ut
tit

4. Determine the curvature of the path of motion at t = 1.


s
In
UP
Chapter 7

Sequences and Series

In this chapter, we find out what it means by the sum of an infinite collection of numbers. We
explore conditions under which an infinite sum

a1 + a2 + a3 + · · · + an + · · ·

ic s
knwon as an infinite series is meaningful. We discuss methods for computing the sum of an infinite

at
series by applying algebra and calculus on the seires. Infinite series are important in science and

em
mathematics because many functions either arise most naturally in the form of an infinite series
h
or have infinite series representation that are useful for numercial computation. Throughout the
at
discussion, N = {0, 1, 2, 3, 4, . . .}.
M
of

7.1 Sequences
e
ut
tit

Definition 7.1.1. A sequence function is a function from N to R.


s

An (infinite) sequence is the ordered list of elements of the range of a sequence function.
In

The elements of an infinite sequence are called its terms.


UP

Notation. If f : N → R is a sequence function and an = f (n), then the resulting sequence is


+∞
{an }n=0 = {a0 , a1 , a2 , . . . , an , . . .} or simply (if there is no ambiguity) {an }.
Variation: For any k ∈ N, {an }+∞
n=k = {ak , ak+1 , ak+2 , . . . , an , . . .}.

Remark 7.1.2. Below are some special sequences.

1. Arithmetic sequence: {tn } where tn = a + dn for constants a and d, d 6= 0, e.g.,

{−2 + 3n}+∞
n=0 = {−2, 1, 4, 7, 10, 13, . . .}

{3 − 2n}+∞
n=0 = {3, 1, −1, −3, −5, . . .} .

2. Geometric sequence: {tn } where tn = arn for nonzero constants a and r, e.g.,

{3 · 2n }+∞
n=0 = {3, 6, 12, 24, . . .}
  n +∞
1
24 = {24, 12, 6, 3, . . .} .
2 n=0

163
164 CHAPTER 7. SEQUENCES AND SERIES

3. Fibonacci sequence: {tn }+∞


n=1 where t1 = 1, t2 = 1, and tn = tn−1 + tn−2 for all n ≥ 3, i.e.,

{1, 1, 2, 3, 5, 8, 13, 21, . . .} .

Our main concern now is to study the behavior of a given sequence {an } as n increases without
bound. Let us consider the following illustration.
 +∞
Illustration 7.1.3. Consider the sequence {an } = n12 n=1 . Observe from the table below that
the values of an get closer and closer to 0 as the values of n increase without bound. Intuitively,
we say that an converges to 0 and that the sequence is convergent.

n an n an
1 1 10000 0.00000001
10 0.01 100000 0.0000000001
100 0.0001 1000000 0.000000000001
1000 0.000001 10000000 0.00000000000001

ic s
at
Definition 7.1.4. Let {an } be a sequence and let L ∈ R. The limit of {an } is L if and only if for

em
any ε > 0 there exists an N > 0 such that |an − L| < ε for all n ∈ N with n > N . In this case, we
say that the sequence {an } converges to L or is convergent, and write lim an = L.
h
at
n→+∞
M

If no number L exists which satisfies the above, then we say that {an } diverges or is divergent.
of

 +∞
1
Example 7.1.5. Prove using the definition that the sequence converges to 0.
e

n2 n=1
ut
tit

1 1 1
Proof. For any ε > 0 let N = √ . If n ∈ N with n > N then n2 > , so that 2 < ε. Thus,
ε ε n
s
In

whenever n ∈ N with n > N ,


1
− 0 = 1 < ε.
UP


n2 n2
 +∞
1 1
It follows from Definition 7.1.4 that is convergent with lim 2 = 0.
n2 n=1 n→+∞ n

Remark 7.1.6. Let m, k ∈ N. The sequence {an }+∞


n=k converges to L if and only if the sequence
+∞
{an }n=m converges to L.

If a sequence {an }+∞


n=k is obtained by restricting the domain of a function f defined on [k, +∞), then
there is a relationship between the limit of f at infinity and the limit of {an }. This is presented in
the next theorem.

Theorem 7.1.7. Let f be a function defined on [k, +∞) for some k ∈ N and let an = f (n) for all
n ∈ N with n ≥ k.

1. If lim f (x) exists, then lim an = lim f (x).


x→+∞ n→+∞ x→+∞
7.1. SEQUENCES 165

2. If lim f (x) = +∞ (respectively, −∞), then lim an = +∞ (respectively, −∞).


x→+∞ n→+∞
 +∞
1
Example 7.1.8. Prove using Theorem 7.1.7 that the sequence converges to 0.
n2 n=1

1
Proof. The function f given by f (x) = 2 is defined on [1, +∞) and lim f (x) = 0. It follows
x x→+∞
from Theorem 7.1.7 that
1
lim 2 = 0.
n→+∞ n
 +∞
1
Therefore converges to 0.
n2 n=1
(√ )+∞
n2 − 5
Example 7.1.9. Determine whether the sequence is convergent or divergent.
3n + 1
n=3

x2 − 5
Solution. Let f (x) = , which is defined on [3, +∞). By Theorem 7.1.7,
3x + 1

ic s
√ √ √
q
1 − x52

at
2 2 2 1
n −5 x −5 x −5 x 1
lim = lim = lim · 1 = lim = .

em
n→+∞ 3n + 1 x→+∞ 3x + 1 x→+∞ 3x + 1 x→+∞ 3 + 1 3
x x
(√ )+∞
h
at
n2 − 5 1
So is convergent with limit .
M

3n + 1 3
n=3
of

+∞
en

e

Example 7.1.10. Determine whether the sequence is convergent or divergent.


ut

n n=1
tit

ex
Solution. Let f (x) = , which is continuous on [1, +∞). Since f (x) is continuous on [1, +∞) and
s
In

x

lim f (x) is indeterminate of the form , L’Hopital’s Rule can be applied. By Theorem 7.1.7 we
UP

x→+∞ ∞
have
en ex L’HR Dx e x ex
lim = lim = lim = lim = +∞.
n→+∞ n x→+∞ x x→+∞ Dx x x→+∞ 1
 n +∞
e
So the sequence is divergent.
n n=1

The next theorem provides another method of finding the limit of a sequence. It is analogous to
the Squeeze Theorem for functions introduced in Math 53.

Theorem 7.1.11 (Squeeze Theorem for Sequences). Let {an }, {bn }, and {cn } be sequences. If
an ≤ bn ≤ cn for all n ≥ k for some k ∈ N, and lim an = lim cn = L for some L ∈ R, then
n→+∞ n→+∞
lim bn = L.
n→+∞
 +∞
n!
Example 7.1.12. Determine whether the sequence is convergent or divergent.
nn n=1
166 CHAPTER 7. SEQUENCES AND SERIES

x!
Solution. Note that the expression x is not defined for non-integer values of x, so Theorem 7.1.7
x
cannot be used to find the limit of the sequence. We proceed by using Squeeze Theorem. Let
n!
an = n . For each n ≥ 1 the term an is positive, and
n
n factors
z }| {
1 · 2 · 3···n 1 2 3 n 1 1
an = = · · ··· ≤ · 1 · 1···1 = .
n
| · n ·
{zn · · · n
} n n n n n n
n factors

1 1
So 0 < an ≤ for all n. Since lim 0 = 0 and lim
n = 0, it follows from the Squeeze Theorem
n→+∞   n→+∞ n
n!
that lim an = 0. Therefore the sequence is convergent with limit 0.
n→+∞ nn

(−1)n +∞
 
Example 7.1.13. Determine whether the sequence converges or diverges.
n n=1

Solution. For each n we have


(−1)n

s
−1 1

ic
≤ ≤ .
n n n

at
−1 1 (−1)n

em
Since lim = 0 = lim , it follows from the Squeeze Theorem that lim = 0.
n→+∞ n n→+∞ n h n→+∞ n
(−1)n +∞
 
at
Thus, converges to 0.
n
M

n=1
of

Definition 7.1.14. A sequence {an } is said to be


e
ut

1. constant if an = an+1 for all n;


tit

2. increasing if an ≤ an+1 for all n;


s
In

3. decreasing if an ≥ an+1 for all n;


UP

4. monotone if it is either increasing or decreasing.

An increasing (respectively, decreasing) sequence in which any two consecutive terms are distinct
is said to be strictly increasing (respectively, strictly decreasing).
+∞
A sequence {an }n=k is ultimately increasing (respectively, ultimately decreasing) if {an }+∞
n=m is
increasing (respectively, decreasing) for some m ≥ k.

Remark 7.1.15. Any of the following conditions implies that a sequence {an }+∞
n=k is increasing:

1. an+1 − an ≥ 0 for all n;


an+1
2. all terms are positive and ≥ 1 for all n;
an
3. an = f (n) where f is differentiable on [k, +∞) for all n ∈ N and f 0 (x) ≥ 0 for all x ≥ k.

Analogous conditions can be obtained which imply that a sequence is strictly increasing, decreasing,
or strictly decreasing.
7.1. SEQUENCES 167

+∞
2n

Example 7.1.16. Determine whether the sequence is increasing or decreasing.
n! n=2

2n
Solution. Let an = . Then
n!
2n+1 2n 2n+1 − 2n (n + 1) 2n (1 − n)
an+1 − an = − = = .
(n + 1)! n! (n + 1)! (n + 1)!
Now 1 − n < 0 since n ≥ 2, so an+1 − an < 0 for all n. Therefore, the given sequence is decreasing.

Another Solution. Notice that an > 0 for all n. We have


2n+1
an+1 (n + 1)! 2n+1 n! 2
= n = · = .
an 2 (n + 1)! 2n n+1
n!
2
Since < 1 for any n ≥ 2, the sequence is strictly decreasing.

s
n+1

ic
at
 n +∞
e
Example 7.1.17. Determine whether the sequence is increasing or decreasing.

em
n n=1
ex
h
at
Solution. Let f (x) = , which is continuous on [1, +∞). Then
x
M

xex − ex ex (x − 1)
f 0 (x) =
of

= ≥ 0 ∀ x ≥ 1.
x2 x2
e
ut

Thus f is increasing on [1, +∞), and so the sequence is increasing.


tit

Definition 7.1.18. A sequence {an } is said to be


s
In

1. bounded below if there exists m ∈ R such that an ≥ m for all n, and m is called a lower
UP

bound for the sequence;


2. bounded above if there exists M ∈ R such that an ≤ M for all n; and M is called an upper
bound for the sequence;
3. bounded if it is both bounded below and bounded above.
(−1)n +∞
 
Example 7.1.19. Show that the sequence is bounded.
n n=1

Proof. For each n ≥ 1,


(−1)n
−1 ≤ ≤ 1.
n
Therefore the sequence is bounded below by −1 and bounded above by 1. Hence, the sequence is
bounded.

Remark 7.1.20. An increasing sequence is bounded below by its first term, while a decreasing
sequence is bounded above by its first term.
168 CHAPTER 7. SEQUENCES AND SERIES

The following result gives a sufficient condition for convergence of sequences which does not
require computing the limit.
Theorem 7.1.21 (Monotone Convergence Theorem). If a sequence is bounded and monotone,
then it is convergent. Equivalently, a monotone sequence is convergent if and only if it is bounded.
 n +∞
2
Example 7.1.22. Show that is convergent.
n! n=2
Proof. By Example 7.1.16 the sequence is decreasing, and hence is monotone and is bounded above
by its first term. Since all terms are positive, the sequence is bounded below by 0. Thus the
sequence is bounded. By the Monotone Convergence Theorem, the sequence is convergent.

EXERCISES. Do as indicated.
1. Determine whether the sequence

0.9, 0.99, 0.999, . . . , 0. 999 ·
| {z }· · 9, . . . ,
n digits

s
in which the nth term consists of n 9’s to the right of the decimal point, is convergent or

ic
at
divergent. If it converges, find its limit.

em
2. Prove: If {|an |} converges to 0, then {an } converges to 0. (Hint: Use the Squeeze Theorem.)
h
3. Prove:
at
M

a. An increasing sequence is bounded below by its first term.


of

b. A decreasing sequence is bounded above by its first term.


e

c. A decreasing sequence of positive terms is bounded.


ut
tit

d. An increasing sequence of negative terms is bounded.


s
In

4. Determine whether the given sequence is ultimately increasing, ultimately decreasing, or neither.
UP

  +∞   +∞  +∞


1 n n!
a. cos b. ln c.
n n=1 n+1 n=2 1 · 3 · · · (2n + 1) n=0

5. Consider the geometric sequence {arn }+∞


n=0 , where a 6= 0 and r 6= 0.

a. Show by directly computing the limit that the sequence converges to 0 when −1 < r ≤ 1 and
diverges otherwise.
b. Use the Monotone Convergence Theorem to show that the sequence converges when 0 < r ≤ 1.

6. Give an example of a sequence which is

a. convergent but not monotonic;


b. bounded but divergent;
c. monotonic but divergent.

7. Is it possible for a sequence to be convergent but not bounded? Explain.


7.2. SERIES OF CONSTANT TERMS 169

7.2 Series of Constant Terms


Definition 7.2.1. An infinite series is an expression of the form

+∞
X
an = a1 + a2 + · · · + an + · · · .
n=1

The numbers a1 , a2 , a3 , . . . are called terms of the series.

+∞
P P P
Notation. If there is no ambiguity, we may write an as an or simply an .
n=1 n
+∞
P
A partial sum of an infinite series an is a finite sum Sk given by
n=1

k
X
Sk = an = a1 + a2 + · · · + ak .
n=1

ic s
at
+∞
X
Definition 7.2.2. Let Sk denote the kth partial sum of a series an . The series is said to be
h em n=1
at
1. convergent with sum S if and only if the sequence {Sk }+∞
k=1 of partial sums converges to S.
M

+∞
P
In this case, we write an = S;
of

n=1
e

2. divergent if and only if the sequence of partial sums diverges.


ut
tit

+∞
P
Example 7.2.3. Let c ∈ R and consider the constant series c. Then for each k ∈ N
s
In

n=1
UP

k
X
Sk := c = ck.
n=1

If c = 0 then Sk = 0 and lim Sk = 0.


k→+∞ 
+∞ if c > 0,
If c 6= 0 then lim Sk = lim ck =
k→+∞ k→+∞ −∞ if c < 0.

Thus, the constant series converges to 0 if c = 0, and diverges otherwise.


+∞
X 1
Example 7.2.4. Determine whether the series is convergent or divergent.
n2 + n
n=1

Solution. Observe that for all n ∈ N,

1 1 1
= − .
n2 +n n n+1
170 CHAPTER 7. SEQUENCES AND SERIES

Thus
1
S1 = 1 −
 2   
1 1 1 1
S2 = 1 − + − =1−
2 2 3 3
     
1 1 1 1 1 1
S3 = 1 − + − + − =1−
2 2 3 3 4 4

It can be shown that for any k ∈ N,


     
1 1 1 1 1 1
Sk = 1 − + − + ... + − =1− .
2 2 3 k k+1 k+1

Hence,  
1
lim Sk = lim 1− = 1.
k→+∞ k→+∞ k+1
Therefore, the series converges to the sum 1.

ic s
Example 7.2.5 (Behavior of geometric series). Let a, r ∈ R \ {0}. Find all values of the common

at
+∞

em
P n
ratio r for which the geometric series ar converges.
n=0 h
at
+∞
P
Solution. If r = 1, then the series is a constant series a. Since a 6= 0, the series diverges by
M

n=0
Example 7.2.3.
of

a 1 − rk+1

k
arn .
e

P
Assume that r 6= 1. Let Sk = Observe that for any k ∈ N, Sk = .
ut

n=0 1−r
We consider two cases.
tit

Case 1. Assume that |r| < 1, i.e., −1 < r < 1. Then lim rk+1 = 0, so that
s
In

k→+∞
UP

a 1 − rk+1

a(1 − 0) a
lim Sk = lim = = .
k→+∞ k→+∞ 1−r 1−r 1−r

So the series converges.


Case 2. Assume that r ≤ −1 or r > 1. Then lim rk+1 does not exist. It follows that neither
k→+∞
does lim Sk . Hence the series diverges.
k→+∞
+∞ a
arn converges with sum
P
Hence, the geometric series if |r| < 1, and diverges otherwise.
n=0 1−r
+∞
X (−4)n
Example 7.2.6. Determine if converges or diverges. If it converges, find its sum.
6n−1
n=0

Solution. Observe that


+∞ +∞ +∞  +∞ 
(−4)n (−4)n −4 n X −2 n
X X X  
= = 6 = 6 .
6n−1 6−1 6n 6 3
n=0 n=0 n=0 n=0
7.2. SERIES OF CONSTANT TERMS 171

−2 −2
So the series is geometric with first term 6 and common ratio . Since < 1, the geometric
3 3
6 18
series is convergent with sum  = .
−2 5
1−
3

Example 7.2.7 (Behavior of harmonic series). Show that the harmonic series
+∞
X 1 1 1 1
= 1 + + + + ···
n 2 3 4
n=1

is divergent.
k
X 1 1
Solution. Let Sk = and consider the graph of y = .
n x
n=1
By comparing areas we have
Z n+1
1 1
dx < · 1

s
x n

ic
n

at
for each n ∈ N\{0}. It follows that
1
Sk = 1 + + . . . +
1 h em
at
2 k
M

Z 2 Z 3 Z k+1
1 1 1
> dx + dx + . . . + dx
1 x 2 x x
of

k
Z k+1
1
e

= dx
ut

1 x
tit

= ln(k + 1).
s
In

Since lim ln(k + 1) = +∞, we also have lim Sk = +∞. So the harmonic series diverges.
UP

k→+∞ k→+∞

Theorem 7.2.8.
+∞
P +∞
P +∞
P +∞
P
1. Let c ∈ R \ {0}. If an converges then can converges with sum c an , and if an
n=1 n=1 n=1 n=1
+∞
P
diverges then can diverges.
n=1
+∞
P +∞
P +∞
P +∞
P +∞
P
2. If an and bn both converge, then (an + bn ) converges with sum an + bn .
n=1 n=1 n=1 n=1 n=1
+∞
P +∞
P +∞
P
3. If an converges and bn diverges, then (an + bn ) diverges.
n=1 n=1 n=1
+∞
P
4. Suppose that an = bn for all but a finite number of values of n. If an converges then
n=1
+∞
P +∞
P +∞
P
bn converges, and if an diverges then bn diverges.
n=1 n=1 n=1
172 CHAPTER 7. SEQUENCES AND SERIES

+∞
X 2
Example 7.2.9. Determine whether the series converges or diverges.
n+8
n=1

Solution. We can write


+∞ +∞ +∞  
X 2 X2 X 1
= = 2 .
n+8 n n
n=1 n=9 n=9
+∞
X 1
The series differs from the harmonic series by exactly eight terms, and so is divergent by
n
n=9
+∞
X 2
statement 4 of Theorem 7.2.8. Since 2 6= 0, statement 1 of the theorem implies that is
n+8
n=1
also divergent.
+∞
X n − 2n
Example 7.2.10. Determine whether the series converges or diverges.
n · 2n
n=1

Solution. Notice that


+∞ +∞ 
n − 2n

X X 1 1

s
= − .

ic
n · 2n 2n n
n=1 n=1

at
+∞

em
X 1 1
The series n
is geometric with common ratio , which has absolute value less than 1, and
2 2 h
n=1
at
+∞
X 1
M

hence is convergent. On the other hand, is the harmonic series, which is divergent. By
n
n=1
of

+∞
X n − 2n
statement 3 of Theorem 7.2.8 the series diverges.
e

n · 2n
ut

n=1
tit

+∞
P
Theorem 7.2.11 (Divergence Test). If the series an is convergent then lim an = 0. Equiva-
s

n→∞
In

n=1
lently,
UP

+∞
P
if lim an 6= 0 then an is divergent.
n→∞ n=1
n
P
Proof. Let Sn = ak . For n ≥ 2,
k=1
an = Sn − Sn−1
+∞
P
If ak is convergent then there is a number S such that lim Sn = S. Therefore
n=1 n→+∞

lim an = lim (Sn − Sn−1 )


n→+∞ n→+∞

= lim Sn − lim Sn−1


n→+∞ n→+∞

=S−S
= 0.
7.2. SERIES OF CONSTANT TERMS 173

+∞ 2
X n +n−1
Example 7.2.12. Determine whether the series is convergent or divergent.
3n2 − 2n
n=1

Solution. Since
1 1 1
n2 + n − 1 n2 + n − 1 n2
1+ n − n2 1
lim = lim · 1 = lim 2 = 6= 0,
n→+∞ 3n2 − 2n n→+∞ 3n2 − 2n n→+∞ 3− 3
n2 n

the series diverges by the Divergence Test.


+∞
X n
Example 7.2.13. Determine if the series converges or diverges.
1 + ln(n)
n=1

Solution. Using L’Hopital’s Rule, we get

n x 1
lim = lim = lim 1 = lim x = +∞.
n→+∞ 1 + ln(n) x→+∞ 1 + ln(x) x→+∞ x→+∞
x

ic s
So the series diverges by the Divergence Test.

at
em
Warning. If lim an = 0, the Divergence Test is inconclusive.
n→+∞ h
at
+∞  n +∞
1 1
M

X X
Example 7.2.14. Consider the geometric series and the harmonic series . No-
2 n
of

 n n=0 n=1
1 1
tice that lim = 0 and lim = 0. However, the geometric series converges while the
e

n→+∞ 2 n→+∞ n
ut

harmonic series diverges.


s tit
In

EXERCISES. Do as indicated.
UP

1. Determine whether the series converges or diverges. If the series converges, find its sum.

+∞   +∞   +∞  n
X e n X 1 2 X 1
a. c. + e. 1+
π 2n 3n n
n=0 n=0 n=1
+∞  n +∞ +∞
9 − 4n 2n − 1 − 3n

X X X 1
b. d. f.
6n n5 + 4 n2 −1
n=0 n=1 n=2

P P
2. Give examples of series an and bn such that:
n n
P P P
a. an and bn are divergent and (an + bn ) is convergent;
n n n
P P P
b. an and bn are divergent and (an + bn ) is divergent.
n n n
174 CHAPTER 7. SEQUENCES AND SERIES

7.3 Convergence Tests for Series of Nonnegative Terms


P
Throughout this section we consider only series an where an ≥ 0 for all n.
n
+∞
P +∞
P
Theorem 7.3.1 (Bounded-Sum Test). Let an be a series of positive terms. Then an
n=1 n=1
converges if and only if its sequence of partial sums is bounded above.
Proof. Let Sn = a1 + . . . + an . Then for all n we have

0 < an+1 = Sn+1 − Sn

and so Sn+1 > Sn . Therefore {Sn }+∞


n=1 is increasing, and hence is monotonic. Also Sn > 0 for all
n, so {Sn }n=1 is bounded below. Thus {Sn }+∞
+∞
n=1 is bounded if and only if it is bounded above. By
+∞
the Monotone Convergence Theorem, {Sn }n=1 is convergent if and only if it is bounded. Putting
+∞
P
everything together, we get that an converges if and only if its sequence of partial sums is
n=1
bounded above.
+∞ 

s

1 1

ic
X
Example 7.3.2. Consider the telescoping sum − . Observe that for all n ∈ N,

at
n n+1
n=1

em
1 1
an = − > 0. Moreover, for all k ∈ N,
n n+1 h
at
k  
X 1 1 1
M

Sk = − =1− .
n n+1 k+1
n=1
of

1
Since 0 < < 1 for all k ∈ N, Sk < 1, i.e., Sk is bounded above. By the Bounded-Sum Test,
e

k+1
ut

the series converges.


tit

The Bounded-Sum Test can be used to prove the following test for convergence.
s
In

Theorem 7.3.3 (Integral Test). Let f be a function that is continuous, positive-valued, and
UP

decreasing on [1, +∞), and suppose that f (n) = an for all n ∈ N∗ .


R +∞ +∞
P
1. If 1 f (x) dx converges, then the series an converges.
n=1
R +∞ +∞
P
2. If 1 f (x) dx = +∞, then the series an diverges.
n=1
Proof. Assume that f satisfies the hypotheses. Then

Z n+1
an+1 = f (n + 1) · 1 < f (x) dx < f (n) · 1 = an
n

m
P
for each n. Let Sm = an . Thus
n=1
Z m
Sm − a1 < f (x) dx < Sm − am .
1
7.3. CONVERGENCE TESTS FOR SERIES OF NONNEGATIVE TERMS 175
R +∞
Case 1. Suppose that 1 f (x) dx converges, say, to the number L. By the above
Z m
Sm < a1 + f (x) dx.
1

Since f is nonnegative and decreasing, comparing areas yields


Z m  Z m  Z +∞
a1 + f (x) dx < lim a1 + f (x) dx = a1 + f (x) dx = a1 + L.
1 m→+∞ 1 1

So Sm < a1 + L for all m, and thus the sequence of partial sums is bounded above. By the
+∞
P
Bounded-Sum Test, an converges.
n=1
R +∞
Case 2. Suppose that 1 f (x) dx = +∞. By the above
Z m
Sm > am + f (x) dx.
1

It follows from the hypothesis that

ic s
 Z m 

at
lim am + f (x) dx = +∞,
m→+∞

em
1

which implies that h


at
lim Sm = +∞.
M

m→+∞

+∞
of

P
Hence an diverges.
n=1
e
ut

+∞
2n
tit

X
Example 7.3.4. Determine whether the series converges or diverges.
1 + n2
s

n=1
In

2x
UP

Solution. Let f (x) = . Clearly, f is positive-valued and continuous on [1, +∞). Moreover,
1 + x2

2(1 − x2 )
f 0 (x) = ≤0 for all x ≥ 1,
(1 + x2 )2

so f is decreasing on [1, +∞). The Integral Test can then be applied. Since
Z +∞ Z t
2x 2x
dx = lim dx
1 1 + x2 t→+∞ 1 1+x
2
 t
= lim ln 1 + x2 1
t→+∞

= lim ln 1 + t2 − ln(2)
 
t→+∞

= +∞

+∞
X 2n
the series diverges by the Integral Test.
1 + n2
n=1
176 CHAPTER 7. SEQUENCES AND SERIES

+∞
X 1
Example 7.3.5. Determine whether the series converges or diverges.
n=2
n ln2 n

1
Solution. Let f (x) = . Clearly, f is positive-valued and continuous on [2, +∞). Since x ln2 x
x ln2 x
increases on [2, +∞), it follows that f decreases on [2, +∞). Hence the Integral Test can be applied.
We have
Z +∞ Z t
1 1
dx = lim 2 dx
2 x ln2 x t→+∞ 2 x ln x
Z ln t
du
= lim
t→+∞ ln 2 u2

1 ln t

= lim −
t→+∞ u
 ln 2 
1 1
= lim − +
t→+∞ ln t ln 2

s
1

ic
= .
ln 2

at
em
+∞
X 1
Therefore converges by the Integral Test. h
n ln2 n
at
n=2
M
of

Example 7.3.6 (Behavior of p-series). Let p ∈ R+ . Find all values of p for which the p-series
+∞
e

X 1
ut

is convergent.
np
tit

n=1
s
In

Solution. If p = 1, the series is the harmonic series, which is divergent by Example 7.2.7.
1
Suppose that p 6= 1. Let f (x) = p . Then f is positive-valued and continuous on [1, +∞). Also
UP

x
f 0 (x) = −px−p−1 < 0 for all x ≥ 1, so f is decreasing on [1, +∞). The Integral Test can be applied.
We have
Z +∞ Z t
1
dx = lim x−p dx
1 xp t→+∞ 1
t
x−p+1
= lim
t→+∞ −p + 1

 −p+1 1 
t 1
= lim −
t→+∞ −p + 1 −p + 1

+∞
 if 0 < p < 1;
= 1

 if p > 1.
p−1

By the Integral Test, a p-series is convergent if p > 1 and divergent if 0 < p ≤ 1.


7.3. CONVERGENCE TESTS FOR SERIES OF NONNEGATIVE TERMS 177

Example 7.3.7. Determine if the series converges or diverges.

+∞ +∞
X 1 X 1
1. √ 2. √
n=1
n n=1
n n

Solution.
+∞
X 1 1
1. The series √ is a p-series with p = < 1, and so is divergent.
n 2
n=1

+∞
X 1 3
2. The series √ is a p-series with p = > 1, and so is convergent.
n n 2
n=1

P P
Theorem 7.3.8 (Comparison Test). Let an and bn be series of nonnegative terms.
n n
P P
1. If an ≤ bn for all n ≥ k for some k ∈ N, and bn is convergent, then an is convergent.

s
n n

ic
P P
2. If an ≥ bn for all n ≥ k for some k ∈ N, and bn is divergent, then an is divergent.

at
n n

em
P
Proof. Assume first that an ≤ bn for all n and that h bn is convergent. For n ≥ k let Sn =
n
at
ak + . . . + an and Tn = bk + . . . + bn . Then Sn ≤ Tn for all n ≥ k, and by the Bounded-Sum
M

Test {Tn }+∞ +∞


n=k is bounded above. Let M be an upper bound of {Tn }n=k . Then Tn ≤ M for all n,
so it follows also that Sn ≤ M for all n. Therefore {Sn }+∞n=k is bounded above. Therefore by the
of

+∞
e

P P
Bounded-Sum Test the series an converges, and by item 4 of Theorem 7.2.8, so does an .
ut

n=k n
tit

P P
Now suppose that an ≥ bn for all n ≥ k and that bn is divergent. If an is convergent then by
s

n n
In

P P
(1.) the series bn is convergent, which is false. Therefore an is divergent.
n n
UP

+∞
X 1
Example 7.3.9. Determine whether the series √ is convergent or divergent.
n=2
3
n−1
+∞
1 1 X 1 1
Solution. For n ≥ 2, √ > √ . The series √ is a p-series with p = < 1 and so is
3
n−1 3
n n=1
3
n 3
+∞ +∞
X 1 X 1
divergent. Hence √ is divergent. Therefore √ is divergent by the Comparison Test.
n=2
3
n n=2
3
n−1
+∞ n
X 2 sin2 (n)
Example 7.3.10. Determine whether the series is convergent or divergent.
3n + 1
n=1

2n sin2 n
Solution. Observe that > 0 for all n ≥ 1. Since 0 < sin2 n < 1 for all n, we have
3n + 1
 n
2n sin2 n 2n 2n 2
< < = .
3n + 1 3n + 1 3n 3
178 CHAPTER 7. SEQUENCES AND SERIES

+∞  n
X 2 2
The series is a geometric series with common ratio ∈ (−1, 1), and so is convergent.
3 3
n=1
+∞ n
X 2 sin2 n
By the Comparison Test, the given series is also convergent.
3n + 1
n=1
+∞
P +∞
P
Theorem 7.3.11 (Limit Comparison Test). Let an and bn be series of positive terms.
n=1 n=1
an
1. If lim = L ∈ R+ , then either both series converge or both series diverge.
n→+∞bn
an P P
2. If lim = 0 and bn converges, then an converges.
n→+∞ bn
an P P
3. If lim = +∞ and bn diverges, then an diverges.
n→+∞ bn
+∞
X 1
Example 7.3.12. Determine whether the series √
3
is convergent or divergent.
n=2
n+1
Warning. One can attempt to use the Comparison Test in this example. For each n ≥ 2 we have

s
+∞

ic
1 1 X 1 1
√ < √ . However, the series √ is a p-series with p = , and so is divergent. Thus the

at
3 3 3
n+1 n n 3
n=2

em
Comparison Test is inconclusive in this case.
1 1
h
Solution. Let an = √ and bn = √ . For each n ≥ 2
at
3 3
n+1 n
M

√3
an 1 n 1
lim = lim √ · = lim 1 = 1.
of

n→+∞ bn 3
n→+∞ n+1 1 n→+∞ 1 + √ 3n
e
ut

+∞ +∞
X 1 1 X 1
Since √ is a divergent p-series with p = < 1, then the series √ diverges by the
tit

3 3
n 3 n + 1
n=2 n=2
s

Limit Comparison Test.


In

+∞
4n2 + 3n − 1
UP

X
Example 7.3.13. Determine whether the series is convergent or divergent.
3n5 + n
n=1

4n2 + 3n − 1 1 +∞
P 1
Solution. Let an = 5
and bn = 3 . The series 3
is a p-series with p = 3 > 1 and
3n + n n n=1 n
so is convergent. Since for each n ≥ 1,
an 4n2 + 3n − 1 n3
lim = lim ·
n→+∞ bn n→+∞ 3n5 + n 1
4n5 + 3n4 − n3
= lim
n→+∞ 3n5 + n
4n5 + 3n4 − n3 n15
= lim · 1
n→+∞ 3n5 + n n5
3 1
4+ n − n2
= lim 1
n→+∞ 3+ n4
4
= ,
3
7.3. CONVERGENCE TESTS FOR SERIES OF NONNEGATIVE TERMS 179

+∞
X 4n2 + 3n − 1
it follows from the Limit Comparison Test that is also convergent.
3n5 + n
n=1

+∞
X 1
Example 7.3.14. Determine whether the series is convergent or divergent.
n=2
ln2 n

1 1 +∞
P 1
Solution. Let an = 2 and bn = . Since diverges and
ln n n n=1 n

an n x L’HR 1 x L’HR 1
lim = lim 2 = lim 2 = lim 1 = lim = lim 2 = +∞,
n→+∞ bn n→+∞ ln n x→+∞ ln x x→+∞ 2 ln x · x
x→+∞ 2 ln x x→+∞
x

+∞
P 1
the series 2 diverges by the Limit Comparison Test.
n=2 ln n

EXERCISES. Do as indicated.

ic s
1. Determine whether the given series is convergent or divergent using (i) the Integral Test, (ii) the

at
Comparison Test, and (iii) the Limit Comparison Test.
h em
at
+∞ +∞   +∞
X ln n X 1 1 X n
M

a. b. sin c. n2
n n2 n e
n=3 n=1 n=1
of

+∞
e

X ln n
ut

2. Consider the series .


n2
tit

n=3
s

a. Explain why the Comparison Test and Limit Comparison Test fail when applied to the given
In

series and the harmonic series.


UP

b. Explain why the Comparison Test and Limit Comparison Test fail when applied to the given
series and the p-series with p = 2.
c. Determine whether the series is convergent or divergent.

3. Determine whether the series converges or diverges.

+∞ +∞ +∞
n3 + 11
 
X X 1 X 1 + cos n
a. c. sin e.
4n5 − 7n3 + 3 n n2 − cos n
n=2 n=1 n=1
+∞ +∞ √ +∞  
X 1 X X 1 1
b. d. e− n
f. coth
ln n n2 n
n=2 n=1 n=1

+∞
X 1
4. Use the Integral Test to find all values of p such that converges.
n lnp n
n=3
180 CHAPTER 7. SEQUENCES AND SERIES

+∞
X ln n
5. Given the series .
np
n=3

a. Use the Comparison Test to show that the series diverges when 0 < p ≤ 1 and converges
when p > 2.
b. Use the Limit Comparison Test to show that the series converges when 1 < p ≤ 2.
P P
6. Let an and bn be series of positive terms. Use the Limit Comparison Test to prove that if
P n P n P
an and bn are both convergent, then n an bn is convergent.
n n
P P
7. Give examples of series an and bn of positive terms such that
n n
P P P
a. an is convergent, an is divergent, and an bn is divergent;
n n n
P P P
b. an is convergent, an is divergent, and an bn is convergent;
n n n
P P P
c. an and an are both divergent, and an bn is divergent;

s
n n n

ic
P P P
d. an and an are both divergent, and an bn is convergent.

at
n n n

7.4 Alternating Series Test


h em
at
M

(−1)n bn or (−1)n−1 bn , where


P P
Definition 7.4.1. An alternating series is a series of the form
n n
of

bn > 0 for each n. That is, an alternating series is a series of nonzero terms which alternate in sign.
e
ut

(−1)n bn be an alternating series with bn > 0 for


P
Theorem 7.4.2 (Alternating Series Test). Let
tit

n
all n. If
s
In

(i) the sequence {bn } is ultimately decreasing, and


UP

(ii) lim bn = 0
n→+∞

then the series is convergent.


+∞
(−1)n−1 bn , where bn > 0 for all n, and assume that this series satisfies
P
Proof. Consider the series
n=1
the conditions (i) and (ii) above. Let Sk = b1 − b2 + . . . + (−1)k bk . If k is even, say k = 2n for
some n, then it follows from condition (ii) that

Sk = S2n = b1 − (b2 − b3 ) − . . . − (b2n−2 − b2n−1 ) − b2n ≤ b1 .

Also S2n+2 = S2n + (b2n+1 − b2n+2 ) ≥ S2n . So the sequence {S2n }+∞n=1 of partial sums Sk with k
even is increasing and bounded above, and hence is monotonic and bounded. Therefore by the
Bounded-Sum Test {S2n }+∞n=1 is convergent. If k is odd, say k = 2n − 1, then

Sk = S2n−1 = (b1 − b2 ) + (b3 − b4 ) + . . . + (b2n−3 − b2n−2 ) + b2n−1 ≥ b1 .


7.4. ALTERNATING SERIES TEST 181

Also S2n+1 = S2n−1 − (b2n − b2n+1 ) ≤ S2n−1 . So the sequence {S2n−1 }+∞ n=1 of partial sums Sk
with k odd is decreasing and bounded below, and hence is monotonic and bounded. Therefore by
the Bounded-Sum Test {S2n−1 }+∞n=1 is convergent. Finally we show that n→+∞
lim S2n = lim S2n−1 .
n→+∞
Indeed,
lim S2n − lim S2n−1 = lim (S2n − S2n−1 ) = lim a2n = 0,
n→+∞ n→+∞ n→+∞ n→+∞

which implies that lim S2n = lim S2n−1 . Therefore the sequence {Sk }+∞
k=1 converges, and hence
n→+∞ n→+∞
+∞
(−1)n−1 bn converges.
P
n=1

Warning. If either lim bn 6= 0 or {bn } is not ultimately decreasing, the Alternating Series Test
n→+∞
is inconclusive. Other tests must be used to determine convergence or divergence.
+∞
X (−1)n−1
Example 7.4.3. Determine whether the series converges or diverges.
n
n=1
1
Solution. Let bn = , which is positive for each n. Then
n

ic s
1 1

at
bn+1 = < = bn
n+1 n
and h
lim bn = limem1
= 0.
at
n→+∞ n→+∞ n
M

+∞
X (−1)n−1
Therefore, by the Alternating Series Test, the series converges.
of

n
n=1
e
ut

+∞
X n(−1)n
Example 7.4.4. Determine whether the series converges or diverges.
tit

2n
n=1
s
In

n
Solution. Let bn = , which is positive for each n. Then
2n
UP

bn+1 n + 1 2n n+1 n+1


= n+1 · = = ≤ 1,
bn 2 n 2n n+n
for all n ∈ N. Moreover,
x 1
lim bn = lim = lim = 0.
n→+∞ x→+∞ 2x x→+∞ 2x ln 2
+∞
X n(−1)n
Hence, the series converges by the Alternating Series Test.
2n
n=1
+∞
(−1)n e1/n is convergent or divergent.
P
Example 7.4.5. Determine whether the series
n=1

Solution. Although the given is an alternating series, the Alternating Series Test fails to give a
conclusion since lim e1/n = 1 = 6 0. Though it follows that lim (−1)n e1/n does not exist. In
n→+∞ n→+∞
+∞
n 1/n
(−1)n e1/n diverges.
P
particular, lim (−1) e 6= 0. By the nth Term Divergence Test the series
n→+∞ n=1
182 CHAPTER 7. SEQUENCES AND SERIES

EXERCISES. Determine whether the series converges or diverges.

+∞ +∞ +∞
(−1)n (−1)n
 
X X cos(nπ) X 1
1. 2. 3. csc
n2 + n ln n n n
n=1 n=2 n=1

7.5 Tests for Absolute Convergence


P P
Theorem 7.5.1. If |an | is convergent then an is convergent.
n n
P
Proof. Suppose that |an | is convergent. Note that
n

−|an | ≤ an ≤ |an |

for all n, which implies that


0 ≤ an + |an | ≤ 2|an |.

ic s
P P P
Since |an | is convergent so is 2|an |. By the Comparison Test, the series (an + |an |) is also

at
n n n

em
convergent. Now
X X 
an = (an + |an |) − |an | .
h
at
n n
M

P P P
Since both (an + |an |) and |an | are convergent, it follows that an is convergent.
n n n
of

+∞
e

P
Definition 7.5.2. A series an is said to be
ut

n=1
tit

+∞
s

P
1. absolutely convergent if |an | is convergent;
In

n=1
UP

+∞
P +∞
P
2. conditionally convergent if an is convergent but |an | is divergent.
n=1 n=1

Remark 7.5.3.

1. By Theorem 7.5.1, an absolutely convergent series is convergent.


P P
2. If an is divergent then |an | is divergent.
n n

+∞
X (−1)n−1
Example 7.5.4. Determine whether the series is absolutely convergent, conditionally
n
n=1
convergent, or divergent.

Solution. The given series is the alternating harmonic series, which is convergent by Example 7.4.3.
The series of absolute values is the harmonic series, which is divergent. Therefore the alternating
harmonic series is conditionally convergent.
7.5. TESTS FOR ABSOLUTE CONVERGENCE 183

+∞
X 2 cos n
Example 7.5.5. Determine whether the series √ is absolutely convergent, condition-
n=1
3n n+n
ally convergent, or divergent.

Warning. The given series contains positive and negative terms but is not an alternating series.
Thus the Alternating Series Test cannot be used.
+∞
X 2 cos n
Solution. Consider the series of absolute values 3n√n + n . Observe that for all n,

n=1

2 cos n 2| cos n| 2 2 2

3n n + n = 3n√n + n ≤ 3n√n + n < 3n√n = 3n3/2 .

+∞ +∞
X 1 3 X 2
The series √ is a p-series with p =
> 1, and so is convergent. Thus √ is also
n n 2 3n n
n=1 n=1
+∞ +∞
X 2 cos n X 2 cos n
convergent. By the Comparison Test, √ is convergent. Therefore √ is
3n n + n 3n n + n

ic s
n=1 n=1
absolutely convergent.

at
em
+∞
P
Theorem 7.5.6 (Ratio Test). Let an be a series of nonzero terms.
h
n=1
at

M

an+1
1. If lim = L < 1 for some L ∈ R, then the series is absolutely convergent.
n→+∞ an
of


an+1
= L > 1 for some L ∈ R or lim an+1 = +∞, then the series is divergent.
e


2. If lim
ut

n→+∞ an n→+∞ an
tit


an+1
s

Warning. If lim = 1 then the Ratio Test is inconclusive. (See Example 7.5.10.)
In


n→+∞ an
UP

+∞
X 1 · 3 · 5 · · · (2n − 1)
Example 7.5.7. Determine whether the series is convergent or divergent.
n!
n=1

1 · 3 · 5 · · · (2n − 1)
Solution. Let an = . Then
n!

= lim 1 · 3 · 5 · · · (2n − 1)(2(n + 1) − 1) ·
an+1 n!
lim
n→+∞ an n→+∞ (n + 1)! 1 · 3 · 5 · · · (2n − 1)
2n + 1
= lim
n→+∞ n + 1

= 2.

Since 2 > 1, the given series diverges by the Ratio Test.


+∞
X n!
Example 7.5.8. Determine whether the series is convergent or divergent.
nn
n=1
184 CHAPTER 7. SEQUENCES AND SERIES

n!
Solution. Let an = . Then
nn
n
nn

an+1 (n + 1)! n
lim = lim · = lim ,
n→+∞ an n→+∞ (n + 1)n+1 n! n→+∞ n + 1

which is indeterminate of the form 1∞ . Now


 
x

x
 ln x+1
lim x ln = lim 1
x→+∞ x+1 x→+∞
x
x+1 1
x ·(x+1)2
= lim −1 (by L’Hopital’s Rule)
x→+∞
x2
−x
= lim
x→+∞ x+1
= −1

so that   
an+1 n
lim = lim exp n ln = exp(−1) = e−1 .

ic
n→+∞ an n→+∞ n+1

at
Since e−1 < 1, the given series is absolutely convergent by the Ratio Test.

em
+∞ 2
X (−4)n h
Example 7.5.9. Determine whether the series is convergent or divergent.
at
(2n + 1)!
n=1
M

(−4) n2
of

Solution. Let an = . Then


(2n + 1)!
e
ut



an+1 (−4)(n+1)2 (2n + 1)! 42n+1
tit

lim = lim · = lim



2
an n→+∞ (2(n + 1) + 1)! (−4)n n→+∞ (2n + 3)(2n + 2)

n→+∞
s
In

where
UP

42x+1 L’HR 42x+1 · 2 ln 4 L’HR 42x+1 · 4 ln2 4


lim = lim = lim = +∞.
x→+∞ (2x + 3)(2x + 2) x→+∞ 4x + 10 x→+∞ 4

So lim an+1
an = +∞. By the Ratio Test, the series is divergent.

n→+∞

P an+1
Example 7.5.10. Give examples of series an such that lim = 1 and
n n→+∞ an
P
1. an is absolutely convergent;
n
P
2. an is conditionally convergent;
n
P
3. an is divergent.
n

Solution. For any p > 0 we have


p
 p  p
1 n n 1
= (1 + 0)p = 1.

lim · = lim = lim 1 −
n→+∞ (n + 1)p 1 n→+∞ n + 1 n→+∞ n+1
7.5. TESTS FOR ABSOLUTE CONVERGENCE 185

+∞
P 1 an+1 +∞
P
1. Consider an where an = 2 . Then lim = 1. The series |an | is convergent since
n=1 n n→+∞ an
n=1
+∞
P
it is a p-series with p = 2 > 1. So an is absolutely convergent.
n=1

(−1)n

+∞
P an+1 +∞
P
2. Consider an where an = . Then lim = 1. The series an is conditionally
n=1 n n→+∞ an
n=1
convergent by Example 7.4.3.

+∞
P 1 an+1 +∞
3. Consider an where an = . Then lim = 1. The series P an is divergent by
n=1 n n→+∞ an n=1
Example 7.2.7.
+∞
P
Theorem 7.5.11 (Root Test). Given a series an .
n=1
1
1. If lim |an | n = L < 1 for some L ∈ R, then the series is absolutely convergent.
n→+∞
1 1
2. If lim |an | n = L > 1 for some L ∈ R or lim |an | n = +∞, then the series is divergent.
n→+∞ n→+∞

ic s
1
Warning. If lim |an | = 1, then the Root Test is inconclusive. (See Example 7.5.14.)

at
n
n→+∞

em
+∞
hX 2n
Example 7.5.12. Determine whether the series n converges or diverges.
(tan−1 (n))
at
n=1
M

2n
Solution. Let an = n. Then
of

(tan−1 (n))
e
ut

1 2 2 4
lim |an | n = lim = 2 · = > 1.
tan−1 (n)
tit

n→+∞ n→+∞ π π
s

By the Root Test, the given series is divergent.


In

+∞
UP

X ln2n n
Example 7.5.13. Determine whether the series is convergent or divergent.
nn
n=1

ln2n n
Solution. Let an = . Then
nn
1 ln2 n
lim |an | n = lim
n→+∞ n→+∞ n

where
ln2 x L’HR 2 ln x L’HR 2
lim = lim = lim = 0.
x→+∞ x x→+∞ x x→+∞ x
1
Hence lim |an | n = 0 < 1, so the series converges absolutely by the Root Test.
n→+∞
P 1
Example 7.5.14. Give examples of series an such that lim |an | n = 1 and
n n→+∞
P
1. an is absolutely convergent;
n
186 CHAPTER 7. SEQUENCES AND SERIES
P
2. an is conditionally convergent;
n
P
3. an is divergent.
n

Solution. We have
 1     
1 n 1 1 − ln n
lim = lim exp ln = lim exp
n→+∞ n n→+∞ n n n→+∞ n
where
− ln x L’HR −1
lim = lim = 0.
x→+∞ x x→+∞ x
So  1  1  
1 n 1 x − ln x
lim = lim = lim exp = exp(0) = 1.
n→+∞ n x→+∞ x x→+∞ x
It follows that for any p > 0
 1   1 !p
1 n 1 n
lim = lim = 1p = 1.
n→+∞ np n→+∞ n

ic s
+∞ 1 1 +∞

at
P P
1. Consider an where an = 2
. Then lim |an | n = 1 and an is absolutely convergent.
n n→+∞

em
n=1 n=1

+∞
P (−1)n 1 +∞
P h
2. Consider an where an = . Then lim |an | n = 1 and an is conditionally conver-
at
n=1 n n→+∞ n=1
M

gent.
+∞ +∞
of

P 1 1 P
3. Consider an where an = . Then lim |an | n = 1 and an is divergent.
n n→+∞
e

n=1 n=1
ut
s tit

EXERCISES. Do as indicated.
In
UP

1. Determine whether the series is absolutely convergent, conditionally convergent, or divergent.

+∞ +∞ +∞
X (−1)n X (2n)! X n
a. d. g. n2 cot−1 n
n2 + n (n!)2
n=1 n=0 n=0
+∞   +∞ +∞
X
n 1 X (2n)! X
b. (−1) sin e. h. (−1)n sechn
n nn
n=1 n=0 n=0
+∞ +∞  n2 +∞
X n2 X n X (−1)n
c. f. i. √
2n n+1 n ln n
n=0 n=0 n=2

+∞
X (−1)n
2. Determine the values of p for which the alternating p-series is absolutely convergent,
np
n=1
conditionally convergent, and divergent.
P P P
3. Give examples of convergent alternating series an and bn such that an bn is divergent.
n n n
7.6. POWER SERIES 187

7.6 Power Series


Definition 7.6.1. Let a ∈ R. A power series about a (or centered at a) is an expression of the
form
+∞
X
cn (x − a)n = c0 + c1 (x − a) + c2 (x − a)2 + c3 (x − a)3 + . . . + cn (x − a)n + . . .
n=0

where x is a variable and cn is a constant for all n.

Remark 7.6.2.

1. Convention: (x − a)0 := 1 for all x ∈ R


2. If there is a largest number d such that cd 6= 0, then the power series is a polynomial with
degree d. Not all power series are polynomials.

ic s
at
Example 7.6.3.

1.
+∞
P h em
xn is a power series centered at 0, with cn = 1 for all n.
at
n=0
M

+∞
X (x − 1)n 1
2. is a power series centered at 1, with cn = for all n.
of

n! n!
n=0
e
ut

+∞
X (x + 1)2n +∞
ck (x + 1)k where c0 = 0,
P
3. is a power series centered at −1. It can be written as
tit

n2 k=0
n=1
s

4
In

ck = 0 if k is odd, and ck = if k is even.


k2
UP

+∞ +∞
(3 − 5x)n (−5)n 3 n
 
X X 3
4. n
can be written as n
x− , and so is a power series centered at ,
2 2 5 5
n=0  n n=0
−5
with cn = for all n.
2
+∞ +∞ +∞
X 1 X 1 X
5. The following are NOT power series: , (x − a) n , cos(nπx).
xn
n=0 n=0 n=1

Definition 7.6.4. The interval of convergence of a power series in the variable x is the set of
all values of x for which the resulting series converges.
+∞
X
Remark 7.6.5. Every power series cn (x − a)n is convergent when x = a. Indeed,
n=0

+∞
X +∞
X +∞
X
cn (a − a)n = c0 + c1 (0)n = c0 + 0 = c0 .
n=0 n=1 n=1
188 CHAPTER 7. SEQUENCES AND SERIES

+∞
X
Theorem 7.6.6. For a given power series cn (x − a)n , exactly one of the following is true:
n=0

1. The series converges only at x = a.


2. The series converges absolutely for all x ∈ R.
3. There is a positive number R such that the series converges absolutely if |x − a| < R and
diverges if |x − a| > R.

Remark 7.6.7.

1. By Theorem 7.6.6, the interval of convergence of a power series centered at a is of one of the
following forms:

• {a};
• (−∞, +∞);

s
• an interval with endpoints a − R and a + R for some real number R > 0.

ic
at
+∞
cn (x − a)n has radius of convergence
em
P
2. We say that
n=0 h
• 0 if its interval of convergence is {a};
at
M

• ∞ if its interval of convergence is (−∞, +∞);


• R if its interval of convergence is an interval with endpoints a − R and a + R for some real
of
e

number R > 0.
ut
tit

3. In many cases, the radius and interval of convergence can be obtained using the conditions of
s

the Ratio Test.


In


UP

X
Example 7.6.8. Find the radius and interval of convergence of the power series xn .
n=0

Solution. Observe that the given power series is geometric with common ratio x. By Example
7.2.5, the power series converges absolutely if |x| < 1 and diverges if |x| ≥ 1. Hence, the interval of
convergence is (−1, 1) and the radius of convergence is 1.

Another Solution. Let an = xn . For x 6= 0,


n+1
an+1 x
lim = lim n = lim |x| = |x|.
n→+∞ an n→+∞ x n→+∞

By the Ratio Test the series is absolutely convergent when |x| < 1, and divergent when |x| > 1. It
is inconclusive when |x| = 1, i.e., x = 1 or x = −1.
+∞
• If x = 1 then the series is a constant series
P
1. Since 1 6= 0, this is divergent by Example
n=0
7.2.3.
7.6. POWER SERIES 189

+∞
• If x = −1 then the series is (−1)n . Since lim (−1)n 6= 0, the series is divergent by the
P
n=0 n→+∞
Divergence Test.

Therefore the interval of convergence is (−1, 1) and the radius of convergence is 1.


+∞ n
X x
Example 7.6.9. Find the radius and interval of convergence of the power series .
n!
n=0

xn
Solution. Let an = . For x 6= 0
n!
n+1
an+1 x n! |x| 1
lim = lim · = lim = |x| · lim = 0.
n→+∞ an n→+∞ (n + 1)! |xn | n→+∞ n + 1 n→+∞ n + 1

Since 0 < 1, it follows from the Ratio Test that the series converges for any x. Therefore the
interval of convergence is (−∞, +∞) and the radius of convergence is ∞.
+∞
X (x − 1)n

s
Example 7.6.10. Find the radius and interval of convergence of .

ic
2n n2
n=1

at
em
(x − 1)n
Solution. Let an = . Then
2n n2 h
at


an+1 (x − 1)n+1 2n n2
M

lim = lim ·
n→+∞ an n→+∞ 2n+1 (n + 1)2 |(x − 1)n |
of

|x − 1| n2
= lim
e

2 n→+∞ (n + 1)2
ut

1
|x − 1| n2
tit

n2
= lim 2 · 1
2 n→+∞ n + 2n + 1
s

n2
In

|x − 1| 1
= lim
2 n→+∞ 1 + n2 + 1
UP

n2
|x − 1|
= .
2
By the Ratio Test, the power series converges absolutely if |x − 1| < 2, which is to say −1 < x < 3,
and diverges if |x − 1| > 2, that is, if x < −1 or x > 3. Note that the Ratio Test fails if |x − 1| = 2,
which corresponds to the cases x = −1 and x = 3. We consider separately the series resulting from
these values of x.
+∞
X 1
• If x = 3, the series becomes . This is convergent since it is a p-series with p = 2 > 1.
n2
n=1

+∞
X (−1)n
• If x = −1, the series becomes . This is absolutely convergent by the above, and
n2
n=1
hence is convergent.

Therefore, the interval of convergence is [−1, 3] and the radius of convergence is 2.


190 CHAPTER 7. SEQUENCES AND SERIES

+∞
X (3x − 2)n
Example 7.6.11. Find the radius and interval of convergence of .
ln n
n=2

(3x − 2)n
Solution. Let an = . Then
ln n
(3x − 2)n+1

an+1 ln n
lim = lim · n

n→+∞ an n→+∞ ln(n + 1) (3x − 2)

ln n
= |3x − 2| lim
n→+∞ ln(n + 1)

1
n
= |3x − 2| lim 1 (by L’Hopital’s Rule)
n→+∞
n+1


n + 1
= |3x − 2| lim
n→+∞ n
= |3x − 2| · 1

By the Ratio Test, the power series converges absolutely if |3x − 2| < 1 and diverges if |3x − 2| > 1.

ic s
1 1
That is, the power series converges absolutely if < x < 1 and diverges if x < or x > 1. The

at
3 3

em
1
test fails at x = and x = 1.
3 h
at
+∞
X 1 1 1
• If x = 1, we get the series
M

. For all n ≥ 2 it is known that ln n < n, so > .


ln n ln n n
n=2
of

+∞
X 1
Since is the harmonic series (minus the first term) and hence is divergent, the series
e

n
ut

n=2
+∞
tit

X 1
is divergent by the Comparison Test.
ln n
s
In

n=2

+∞
UP

1 X (−1)n 1
• If x = , the series becomes . Since lim = 0, and {ln(n)}+∞
n=2 is a decreasing
3 ln n n→+∞ ln n
n=2
+∞
X (−1)n
sequence (by the fact that ln is an increasing function), the series converges by the
ln n
n=2
Alternating Series Test.
1  1
Hence the interval of convergence is 3, 1 and the radius of convergence is .
3
EXERCISES. Find the radius and interval of convergence of the given power series.

+∞ +∞ +∞
X (x − a)n X (x − 2)n X
1. , for any a ∈ R 3. 5. (−5x)n
n! 2n − 1
n=0 n=0 n=0

+∞ +∞ +∞
X (3x + 4)n X (1 − 2x)n X
2. 4. √ 6. n!(x − 2)n
n2 n2 − 9
n=1 n=4 n=0
7.7. FUNCTIONS AS POWER SERIES 191

7.7 Functions as Power Series


+∞
X
The sum of a power series cn (x − a)n is a function f given by
n=0

+∞
X
f (x) = cn (x − a)n
n=0

for all x in the interval of convergence of the series. In this section and in the next two, we are
concerned with the following problems:

1. Given a power series, find a non-series expression for its sum f (x), together with
the set of all numbers x for which this is valid.
2. Given a non-series expression g(x), find a power series whose sum is g(x), together
with the set of all numbers x for which this is valid.
+∞
xn .
P
Example 7.7.1. Find a non-series expression for the sum of the power series

s
n=0

ic
at
Solution. Observe that the given series is geometric with first term 1 and common ratio x. By
1

em
Example 7.2.5 this series converges to the sum when |x| < 1 and diverges otherwise. Thus
1−x h
we write
at
+∞
X 1
xn = for all x ∈ (−1, 1).
M

1−x
n=0
of

Example 7.7.2. Express each of the following as a power series and indicate the interval in which
e
ut

this representation is valid.


stit

1 2x 4
In

1. − 2. 3.
x 1 + x2 2−x
UP

Solution.
−1 1
1. Observe that = . By Example 7.7.1
x 1 − (x + 1)
+∞
1 X
= (x + 1)n
x
n=0

for all x satisfying −1 < x + 1 < 1, that is, for all x ∈ (−2, 0).
1 1
2. Observe that = . It follows from Example 7.7.1 that
1 + x2 1 − (−x2 )
+∞ +∞
2x X
2 n
X
= 2x (−x ) = (−1)n 2x2n+1
1 + x2
n=0 n=0

for all x satisfying x2 < 1, that is, for all x ∈ (−1, 1).
192 CHAPTER 7. SEQUENCES AND SERIES
 
4 1
3. Observe that =2 x . It follows from Example 7.7.1 that
2−x 1− 2

+∞
X  x n X x n +∞
4
=2 =
2−x 2 2n−1
n=0 n=0

for all x satisfying x2 < 1, that is, for all x ∈ (−2, 2).

+∞
X
Theorem 7.7.3. Assume that the power series cn (x − a)n has radius of convergence R, where
n=0
R ∈ R+ or R = +∞. Then the function f defined by
+∞
X
f (x) = cn (x − a)n
n=0

is differentiable (and therefore continuous) on the interval I, where I = (a − R, a + R) if R ∈ R+


and I = (−∞, +∞) if R = +∞. Furthermore,

ic s
+∞

at
X
f 0 (x) = ncn (x − a)n−1

em
n=1
h
and
at
+∞
(x − a)n+1
Z
M

X
f (x) dx = C + cn
n+1
of

n=0

where C is an arbitrary constant. The series representations for f 0 (x) and


R
f (x) dx both have
e
ut

radius of convergence R.
tit

+∞
s

X Rx
(−1)n x2n . Find f 0 (x) and
In

Example 7.7.4. Let f (x) = 0 f (t) dt, and the intervals of conver-
R x n=0
UP

gence of f (x), f 0 (x), and 0 f (t) dt.

Solution.
+∞
X
1. The series (−1)n x2n is geometric with common ratio −x2 . By Example 7.2.5 it converges
n=0
exactly when | − x2 | < 1, which is to say that the interval of convergence is (−1, 1).

2. By Theorem 7.7.3,
+∞
X
f 0 (x) = (−1)n 2n x2n−1 .
n=1
+∞
and f 0 converges for all x ∈ (−1, 1). If x = 1 the series becomes (−1)n 2n, and if x = −1
P
n=1
+∞
(−1)n−1 2n. Since
P
it becomes lim 2n 6= 0, both of these series diverge by the nth Term
n=1 n→+∞
Divergence Test. Hence the interval of convergence of f 0 (x) is (−1, 1).
7.7. FUNCTIONS AS POWER SERIES 193

3. By Theorem 7.7.3 and the Second Fundamental Theorem of the Calculus,

x +∞
(−1)n x2n+1
Z X
f (t) dt =
0 2n + 1
n=0

+∞
X (−1)n
which converges also for all x ∈ (−1, 1). If x = 1 the series becomes ; since
2n + 1
n=0
 +∞
1 1
is decreasing and lim = 0, the series converges by the Alternating
2n + 1 n=0
n→+∞ 2n + 1
+∞
X (−1)n+1
Series Test. If x = −1 the series becomes , which also converges by the Alternating
2n + 1
n=0 Rx
Series Test. Hence the interval of convergence of 0 f (t) dt is [−1, 1].

Example 7.7.5.
1
1. Represent by a power series and indicate the interval in which this representation is

s
(1 − x)2

ic
valid.

at
em
+∞
X n
2. Find the sum of .
2n
h
n=1
at
M

 2 +∞
1 1 1 P n 1
Warning. Since 2
= and = x for all x ∈ (−1, 1), we have =
(1 − x) 1−x 1 − x n=0 (1 − x)2
of

 +∞ 2  +∞ 2
e

P n P n
x for all x ∈ (−1, 1). However, the expression x is not a power series.
ut

n=0 n=0
tit

Solution.
s
In

 
1 1
1. Observe that Dx = . Using Example 7.7.1 and Theorem 7.7.3 we get
UP

1−x (1 − x)2
+∞ +∞
1 X
n
X
= Dx x = nxn−1
(1 − x)2
n=0 n=1

for all x ∈ (−1, 1).

2. Observe that
+∞ +∞  n +∞  n−1 +∞
!
X n X 1 1X 1 1 X
= n = n = nxn−1

2n 2 2 2 2
n=1 n=1 n=1 n=1 x= 21

1
Since ∈ (−1, 1), it follows from item 1 above that
2
+∞
X n 1 1 1
= ·  = · 4 = 2.
2n 2 1− 1 2 2
n=1 2
194 CHAPTER 7. SEQUENCES AND SERIES

Example 7.7.6. Express ln(1 + x) as a power series, and give the interval in which this represen-
tation is valid.
1
Solution. Recall that Dx ln(1 + x) = . Using Example 7.7.1, we have
1+x
+∞
1 1 X
= = (−1)n xn
1+x 1 − (−x)
n=0

for |x| < 1. Integrating both sides with respect to x, we get


+∞
X (−1)n xn+1
ln(1 + x) = C +
n+1
n=0

for any x ∈ (−1, 1), and for some constant C. In particular, if we plug in x = 0 in the equation
above, we obtain C = 0. Hence,
+∞
X (−1)n xn+1
ln(1 + x) =
n+1
n=0

s
for any x ∈ (−1, 1).

ic
at
Example 7.7.7. Express tan−1 (x) as a power series, and give the interval in which this represen-

em
tation is valid. h
1
at
Solution. Recall that Dx tan−1 (x) = . Using Example 7.7.1, we have
1 + x2
M

+∞
1
of

X
= (−1)n x2n
1 + x2
e

n=0
ut

for |x| < 1. Integrating both sides with respect to x, we get


tit

+∞
(−1)n x2n+1
s

X
−1
In

tan x=C+
2n + 1
n=0
UP

for any x ∈ (−1, 1) and for some constant C. Since 0 ∈ (−1, 1) and tan−1 (0) = 0, we get
+∞ +∞
X (−1)n 02n+1 X
0=C+ =C+ 0 = C.
2n + 1
n=0 n=0

So if x ∈ (−1, 1) then
+∞
−1
X (−1)n x2n+1
tan (x) = .
2n + 1
n=0
By Example 7.7.4 the series on the right has interval of convergence [−1, 1]. It can be shown that
the equality above also holds for x = ±1. See Exercise 3 for the details of the proof.

EXERCISES. Do as indicated.

1. Use the result in Example 7.7.1 and Theorem 7.7.3 to obtain a series representation for each
expression.
7.8. TAYLOR AND MACLAURIN SERIES 195

1 x2 c. ln(3 − x), for x ∈ (−3, 3)


a. b.
3−x (3 − x)2

2. a. Use Theorem 7.7.3 to find a series representation for ln(1 + x) for x ∈ (−1, 1).
k−1 k−1
Z 1 X !
X (−1)n n n
b. Show that = (−1) t dt.
n+1 0
n=0 n=0
k Z 1 k
X (−1)n−1 k+1 t
c. Use (b.) to show that = ln(2) + (−1) dt.
n 0 1+t
n=1
 Z 1 k 
t
d. Use the Squeeze Theorem to show that lim (−1)k+1 dt = 0. Conclude that
k→+∞ 0 1+t
+∞
X (−1) n−1
= ln(2), and that the representation in (a.) is valid for x ∈ (−1, 1].
n
n=1
+∞
X (−1)n
3. a. Using a method similar to Exercise 2, show that tan−1 (1) = .
2n + 1
n=0

s
+∞

ic
X (−1)n
b. Show that π = 4 . (This is known as the Gregory-Leibniz series for π.)

at
2n + 1
n=0

7.8 Taylor and Maclaurin Series


h em
at
M

We continue our study of one of the problems posed in the previous section, namely, that of finding
of

a power series representation for a given expression g(x). Under certain conditions on g(x), we can
e

define a power series, called a Taylor series, associated with g(x). In Theorem 7.8.1 it is shown that
ut

if g(x) has a power series representation then this series is a Taylor series of g(x). From Theorems
tit

7.9.3 and 7.9.4 we obtain a method for determining whether g(x) is equal to its Taylor series, and
s
In

for which values of x this representation is valid.


UP

Theorem 7.8.1. Assume that the function g has a power series representation about the number
a ∈ dom(g). Then the derivatives of g of all orders exist at a, and
+∞ (n)
X g (a)
g(x) = (x − a)n
n!
n=0

for all x in the interval of convergence of the series, where g (0) := g.


+∞
cn (x − a)n , where ci ∈ R for all i. By Theorem 7.7.3
P
Proof. Let a ∈ R and assume that g(x) =
n=0
+∞
X
g 0 (x) = n · cn (x − a)n−1 .
n=1

It can be shown that for any k ∈ Z+ ,


+∞
X
g (k) (x) = n(n − 1) · · · (n − k + 1) · cn (x − a)n−k .
n=k
196 CHAPTER 7. SEQUENCES AND SERIES

Thus, if g (0) (x) := g(x), then for any nonnegative integer k

g (k) (a) = k! ck .

Therefore g (k) (a) exists for all k, and


g (k) (a)
ck = .
k!
The result follows.

Definition 7.8.2. The series in Theorem 7.8.1 is called the Taylor series for g about the number
a. The Taylor series for g about 0 is called the Maclaurin series for g.
1
Example 7.8.3. Find the Taylor series for g(x) = about 1, and determine the interval of
x
convergence of the Taylor series.
−1 00 (−1)2 2 · 1 3
000 (x) = (−1) 3 · 2 · 1 . If g (k) (x) =
Solution. Observe that g 0 (x) = , g (x) = , and g
x2 x3 x4
(−1)k k!
then

s
xk+1

ic
(−1)k k! · −(k + 1) (−1)k+1 (k + 1)!

at
g (k+1) (x) = = .
xk+2 xk+2

em
So g (n) (1) = (−1)n n! for all nonnegative integers n, and the Taylor series for g about 1 is
h
at
+∞ +∞
(−1)n n!
M

X X
(x − 1)n = (−1)n (x − 1)n .
n!
n=0 n=0
of

This series is geometric with common ratio −(x−1), is convergent when |x−1| < 1, and is divergent
e
ut

when |x − 1| ≥ 1. So the interval of convergence of the series is (0, 2).


tit

Example 7.8.4. Find the Maclaurin series for ex and its interval of convergence.
s
In

1
Solution. Let g(x) = ex . Then for all n we have g (n) (0) = ex x=0 = 1, so cn =
UP

. Hence the
x
n!
Maclaurin series for e is
+∞ n
X x
.
n!
n=0
It was shown in Example 7.6.9 that this series has interval of convergence R.

Warning. It is sometimes true that a function is equal to its Taylor series at all points in the
interval of convergence of the series, but not always.

Example 7.8.5. Show that ex is equal to its Maclaurin series for all x ∈ R.
+∞ n
X x
Proof. Let h(x) = . By Theorem 7.7.3,
n!
n=0

+∞ +∞ ∞
X n xn−1 X xn−1 X xn
h0 (x) = = = .
n! (n − 1)! n!
n=1 n=1 n=0
7.8. TAYLOR AND MACLAURIN SERIES 197

So h satisfies the differential equation h(x) = h0 (x), which implies that h(x) = ex + C for some
constant C. Letting x = 0 yields

1 = h(1) = e0 + C = 1 + C,
+∞ n
X x
and thus C = 0. Therefore h(x) = ex , which proves that ex = , as required.
n!
n=0

+∞
X 1
Example 7.8.6. Use the result of Example 7.8.5 to find the sum of .
n!(n + 2)
n=1

Solution. Consider the expression xex . From Example 7.8.5 we have


+∞ n +∞ n+1
X x X x
xex = x = .
n! n!
n=0 n=0

By Theorem 7.7.3,
+∞ n+1 +∞ Z +∞
!
t t t
xn+1 tn+2


s
Z Z
x
X x X X

ic
xe dx = dx = dt = ,
n! n! n!(n + 2)

at
0 0 n=0 n=0 0 n=0

em
and since the Maclaurin series for ex
converges for any real number, so does this series. On the
other hand, integrating by parts gives
h
at
M

Z t t Z t
x x
xe dx = xe − ex dx = tet − 0 − et + 1 = (t − 1)et + 1.
of

0 0 0
e

Thus we have
ut

+∞
X tn+2
(t − 1)et + 1 =
tit

.
n!(n + 2)
n=0
s
In

Evaluating both sides at t = 1 (which is valid since the series on the right converges for any t ∈ R)
UP

we get
+∞ +∞
X 1 1 X 1
1= = + .
n!(n + 2) 2 n!(n + 2)
n=0 n=1
Therefore
+∞
X 1 1 1
=1− = .
n!(n + 2) 2 2
n=1

Example 7.8.7. Find the Maclaurin series for sin x.

Solution. Let g(x) = sin x. Observe that the derivatives of sin x repeat after a cycle of four. For
any k ∈ N ∪ {0}, 
sin x

 , if n = 4k



cos x , if n = 4k + 1
g (n) (x) =
− sin x , if n = 4k + 2




− cos x , if n = 4k + 3.

198 CHAPTER 7. SEQUENCES AND SERIES

Thus, at the desired center of the power series (that is, x = 0) the only non-zero derivatives are
those with odd orders. In particular,



1 , if n = 4k + 1

g (n) (x) = −1 , if n = 4k + 3


0

, otherwise.

Hence, the Maclaurin series for sin x is


+∞
X x2n+1 x x3 x5 x7
(−1)n = − + − + ··· .
(2n + 1)! 1! 3! 5! 7!
n=0

In previous examples, we have obtained the Maclaurin series of the functions ex , ln(1+x), and sin x.
Table 7.1 shows the Maclaurins eries of some common functions and their radius of convergence.
The derivation of the Maclaurin series of the remaining functions is left as an exercise.

s
g(x) Maclaurin series Radius of convergence

ic
at
+∞
xn

em
ex
P
n! ∞
n=0 h
+∞
at
P (−1)n n+1
ln(1 + x) n+1 x 1
M

n=0
+∞
of

P (−1)n 2n+1
sin(x) (2n+1)! x ∞
e

n=0
ut

+∞
(−1)n 2n
tit

P
cos(x) (2n)! x ∞
n=0
s
In

+∞
P 1 2n+1
sinh(x) (2n+1)! x ∞
UP

n=0
+∞
P 1 2n
cosh(x) (2n)! x ∞
n=0

+∞ ∞
 if m ∈ N
m(m−1)···(m−n+1) n
x)m ,
P
(1 + m∈R n! x
n=0 
1 if m ∈
/N

Table 7.1: Maclaurin series of common functions

EXERCISES. Do as indicated.

1. Find the Maclaurin series for cos(x) using two methods:

a. using Theorem 7.8.1; and


7.9. APPROXIMATIONS USING TAYLOR POLYNOMIALS 199

b. by differentiating the Maclaurin series of sin(x). (It will be shown in the next section that
sin(x) is equal to its Maclaurin series for all x ∈ R.)

2. Find the Taylor series of ex about any a ∈ R using two methods:

a. using Theorem 7.8.1; and


b. by writing ex = ea ex−a and using Example 7.8.5.
+∞ +∞
X 1 X (−1)n
3. Use Example 7.8.5 to find the sum of and of .
n! n!
n=0 n=0

4. Derive the Maclaurin series for the remaining functions in Table 7.1, and compute the radius of
convergence of each series.

5. a. Show that (1 + x)m , where m is not a nonnegative integer, is equal to its Maclaurin series for
 +∞
P m(m−1)···(m−n+1) n
all x ∈ (−1, 1). Hint: Let h(x) = n! x . Show that (1 + x) h0 (x) = m h(x)
n=0

s


ic
and solve for h.

at
1

em
b. Find the Maclaurin series for √ .
1 − x2 h
 
c. Find the Maclaurin series for sin−1 (x). Hint: Recall that Dx sin−1 (x) = √ 1
at
1−x2
.
M

d. Give a series representation for π using the Maclaurin series for sin−1 (x).
of
e
ut

7.9 Approximations Using Taylor polynomials


stit

In this section we consider further the problem of whether a given function is equal to its Taylor
In

series. For functions which are indeed equal to their Taylor series, we estimate function values
UP

using partial sums of the Taylor series and determine the accuracy of these approximations.

Definition 7.9.1. Let f be a function whose first k derivatives exist at a. The kth degree Taylor
polynomial of f about a is the polynomial Pk defined by

k
X f (n) (a)
Pk (x) = (x − a)n .
n!
n=0

The remainder is the function Rk given by

Rk (x) = f (x) − Pk (x).

Remark 7.9.2. A Taylor polynomial of f is a partial sum of the Taylor series of f . Hence
+∞ (n)
X f (a)
(x − a)n = lim Pk (x). (7.1)
n! k→+∞
n=0
200 CHAPTER 7. SEQUENCES AND SERIES

Theorem 7.9.3. Let f be a function whose derivatives of all orders exist at a, and let Pk (x) denote
the Taylor polynomial of f about a. Then f (x) is equal to its Taylor series about a if and only if
lim Rk (x) = 0.
k→+∞

Proof. By (7.1) and the definition of the remainder function,

+∞ (n)
X f (a)
(x − a)n = lim Pk (x) = lim

f (x) − Rk (x) = f (x) − lim Rk (x).
n! k→+∞ k→+∞ k→+∞
n=0

+∞ (n)
X f (a)
It follows immediately that f (x) = (x − a)n exactly when lim Rk (x) = 0.
n! k→+∞
n=0

It is often inconvenient to compute lim Rk (x) using its definition. Theorem 7.9.4 and its imme-
k→+∞
diate consequence Corollary 7.9.7 are among several results that provide us with indirect methods
of determining if this limit is equal to zero.

ic s
at
Theorem 7.9.4 (Lagrange form of Rk ). Let f be a function whose first k + 1 derivatives exist in

em
the open interval I between x and a with f (k) continuous in the closed interval between x and a.
Then for some z ∈ I, with z dependent on k,
h
at
M

f (k+1) (z)
Rk (x) = (x − a)k+1 (7.2)
of

(k + 1)!
e
ut

Remark 7.9.5. Theorem 7.9.4 is a generalization of the Mean-Value Theorem for functions which
tit

was learned in Math 21. Recall that if f is a function that is continuous on a closed interval [a, b]
s

and differentiable on the open interval (a, b), then there exists a number z ∈ (a, b) satisfying
In

f (b) − f (a)
UP

f 0 (z) = .
b−a

This can be written as


f 0 (z)
f (b) − f (a) = (b − a).
1!
Observe that the left side is equal to f (b) − P0 (b), thus implying that the right side is R0 (b), which
is a special case of (7.2).
+∞
X (x − a)k
The following is a frequently useful fact: Since is absolutely convergent for all a, x ∈ R
k!
k=0
(see Exercises 7.6 item 1),

(x − a)k
lim =0 for any a, x ∈ R. (7.3)
k→+∞ k!

Example 7.9.6. Show that ex is equal to its Maclaurin series for all x ∈ R. (See Example 7.8.5.)
7.9. APPROXIMATIONS USING TAYLOR POLYNOMIALS 201

Proof. By Theorem 7.9.4,


ez
Rk (x) = xk+1
(k + 1)!
+∞ n
X 0
for some z between 0 and x, where z depends on k. Clearly e0 = 1 = 1 + so we need only
n!
n=1
to consider x 6= 0. We divide our analysis into two cases.
Case 1. Suppose that x > 0. Then 0 < z < x, so that 0 < ez < ex and
ex
0 < Rk (x) < xk+1 .
(k + 1)!

Applying (7.3) we get

ex xk+1
lim xk+1 = ex lim = ex · 0 = 0.
k→+∞ (k + 1)! k→+∞ (k + 1)!

It follows from the Squeeze Theorem for functions that lim Rk (x) = 0. So ex is equal to its
k→+∞
Maclaurin series for all x > 0.

s
Case 2. Suppose that x < 0. Then x < z < 0 and 0 < ez < 1, so that

ic
at
|x|k+1

em
0 < |Rk (x)| < .
h (k + 1)!
at
Again applying (7.3) and the Squeeze Theorem yields lim Rk (x) = 0. Thus ex is equal to its
k→+∞
M

Maclaurin series for all x < 0.


of

Therefore ex is equal to its Maclaurin series for all x ∈ R.


e
ut
tit

Corollary 7.9.7. Assume the hypotheses of Theorem 7.9.4. If there is a constant M satisfying
s

(k+1)
(z) ≤ M for any z ∈ I, then
In

f
UP

M
|Rk (x)| ≤ |x − a|k+1 .
(k + 1)!

Example 7.9.8. Show that sin(x) is equal to its Maclaurin series for all x ∈ R.

Proof. Let f (x) = sin(x). By Theorem 7.9.4,

f (k+1) (z) k+1


Rk (x) = x
(k + 1)!

for some z (dependent on k) between 0 and x. Now f (k+1) (z) is one of cos(z), − cos(z), sin(z), and

− sin(z), so f (k+1) (z) ≤ 1. Hence, by Corollary 7.9.7,

|x|k+1
|Rk (x)| ≤ .
(k + 1)!

Applying (7.3) and the Squeeze Theorem, we get lim |Rk (x)| = 0, and thus lim Rk (x) = 0.
k→+∞ k→+∞
Therefore sin(x) is indeed equal to its Maclaurin series for any x ∈ R.
202 CHAPTER 7. SEQUENCES AND SERIES

Remark 7.9.9. For a fixed degree k, Corollary 7.9.7 suggests that the approximation Pk (x) im-
proves (that is, |Rk (x)| decreases) the closer a is to x. Likewise, for a fixed a (suitably close to x),
the approximation Pk (x) improves by increasing the degree k, which is the same as increasing the
number of terms.

Example 7.9.10. Use the Maclaurin polynomial of degree 7 for sin(x) to estimate sin(1) and
sin(2). In each case, determine the number of decimal places to which the approximation is correct.

Solution. The Maclaurin polynomial of degree 7 for sin(x) is


3
X (−1)n
P7 (x) = x2n+1 .
(2n + 1)!
n=0

By Corollary 7.9.7 and the fact that |Dk sin(x)| = 1 for any k, we have
1 8
|R7 (x)| ≤ x .
8!
1. When x = 1,

s
3
(−1)n

ic
X 1 1 1
sin(1) ≈ P7 (1) = = 1 − + − ≈ 0.841468254

at
(2n + 1)! 3! 5! 7!
n=0

and
18
≈ 2.5 × 10−5 = 0.000025.
|R7 (1)| ≤
em
h
at
8!
M

Therefore the approximation for sin(1) is accurate to at least four decimal places.
of

2. When x = 2,
e
ut

3
X (−1)n 22n+1 23 25 27
tit

sin(2) ≈ P7 (2) = =2− + − ≈ 0.907936508


(2n + 1)! 3! 5! 7!
n=0
s
In

and
28
UP

|R7 (2)| ≤ ≈ 0.006.


8!
Therefore the approximation for sin(2) is accurate to at least two decimal places.

Example 7.9.11. Find the second degree Taylor polynomial of f (x) = 4x about 1 and use it to

approximate the numercial value of 4.08.
√ 2 4
Solution. If f (x) = 4x then f 0 (x) = √ and f 00 (x) = − p . At the given center, we have
4x (4x)3
f (n) (1)
f (1) = 2, f 0 (1) = 1, and f 00 (1) = − 21 . Using the formula cn = , the coefficients of the Taylor
n!
polynomial are c0 = 2, c1 = 1, and c2 = − 14 . Thus, second degree Taylor polynomial of f about 1
is
1
P2 (x) = 2 + (x − 1) − (x − 1)2 .
4

Moreover, 4.08 = f (1.02) and so
√ 1
4.08 ≈ P2 (1.02) = 2 + (1.02 − 1) − (1.02 − 1)2 = 2 + 0.02 − 0.0001 = 2.0199.
4
7.9. APPROXIMATIONS USING TAYLOR POLYNOMIALS 203

+∞
an , where an = (−1)n+1 bn or an =
P
Theorem 7.9.12. Given a convergent alternating series
n=1
(−1)n bn , with bn > 0 and bn+1 < bn for all n, and lim bn = 0. If S is the sum of the series, then
n→+∞

Xk
n+1
S − (−1) bn < bk+1 .



n=1
+∞
cn (x0 −a)n , where the series on the right is a convergent alternating
P
Remark 7.9.13. If f (x0 ) =
n=0
series satisfying the conditions of Theorem 7.9.12 with bn = |cn (x0 − a)n |, then

|Rk (x0 )| < ck+1 (x0 − a)k+1 .

Z 1
sin x2 dx to four decimal places.

Example 7.9.14. Estimate
0
Solution. Since sin(x) is equal to its Maclaurin series for all x ∈ R, we have
+∞ +∞
Z t Z t X !
2
 (−1)n 2 2n+1
 X (−1)n t4n+3
sin x dx = x dx = ·
0 0 (2n + 1)! (2n + 1)! 4n + 3
n=0 n=0

ic s
so that

at
1 +∞
(−1)n
Z X
2


em
sin x dx = .
0 (2n + 1)!(4n + 3)
n=0h
The series on the right is a convergent alternating series satisfying the conditions of Theorem
at
7.9.12, whose first four terms, rounded off to five decimal places, have the following absolute values:
M

0.33333, 0.02381, 0.00076, 0.00001. It follows from Remark 7.9.13 that


of

|R2 (1)| < 0.00001


e
ut

and so to four decimal places,


tit

2
s

Z 1
(−1)n 1 1 1
In

X
sin x2 dx ≈

= − + ≈ 0.3103.
0 (2n + 1)!(4n + 3) 3! 3! · 7 5! · 11
n=0
UP

EXERCISES. Do as indicated.

1. Let f (x) = 3 9x − 1.

(a) Find the second degree Taylor polynomial of f about 1.



(b) Use (a) to approximate 3 8.09.

2. Let g(x) = e2 sin x .

(a) Find the third degree Maclaurin polynomial of g


(b) Use (a) to approximate g(0.02).

3. Let h(x) = ln(4 − x).

(a) Find the fourth degree Taylor polynomial of h about 3.


(b) Use (a) to approximate ln(1.04).
204 CHAPTER 7. SEQUENCES AND SERIES

7.10 Chapter Exercises


+∞
X
I. Suppose that a series an has the property that for all k ∈ {1, 2, 3, . . .},
n=1

k2 + k
Sk = a1 + a2 + · · · + ak = .
k2 + cos k
+∞
X
1. Does the series an converge? If it converges, find its sum.
n=1
2. Does the sequence {an }+∞
n=1 converge? If it converges, find its limit.
+∞
X
3. Does the series Sk converge? If it converges, find its sum.
k=1

+∞ √
X
n n
II. Show that the series (−1) is conditionally convergent.
n+5
n=1

ic s
+∞

at
X n
III. Given the series .
en2
em
n=1
h
1. Verify that the series satisfies the conditions of the Integral Test.
at
M

2. Use the Integral Test to determine whether the series converges or diverges.
of

+∞
X 1
IV. Given the series √ .
e

3
n + sin2 n
ut

n=1
tit

1. Explain why the Comparison Test fails for the given series when it is compared with the
+∞
s

1
In

X
series √
3
.
n=1
n
UP

2. Use the Comparison Test to determine if the given series converges or diverges.

V. Determine if the given series converges or diverges.

+∞ +∞ +∞ 1
X 8n X n cos n X 1 · 22 · 32 · · · n2
1. 2n+1
3. 5.
3 + n3 n5 + n (2n)!
n=1 n=1 n=1
+∞ +∞ 2 +∞
X (−1)n n2n X (−1)n 2n X (−1)n n
2. 4. 6.
n=1
2n2 n=2
(ln n)n
n=2
3
n2 + 1

+∞ n
X 4 (2x − 1)n
VI. Find the interval of convergence of the power series √ .
n=0
1 + n

+∞ n
X x
VII. For all x ∈ R, ex = .
n!
n=0
7.10. CHAPTER EXERCISES 205

1. Express g(x) = xe2x as a power series.


2. Find g (100) (0), the 100th derivative of g at 0.
3. Differentiate the identity in (1) and assign a suitable value of x to find the sum of
+∞ n
X 2 (n + 1) (2)(2) (3)(4) (4)(8) (5)(16)
= + + + + ··· .
n! 1! 2! 3! 4!
n=1

+∞ 1
(−1)n
X Z
VIII. Given that cos x = x2n , for all x ∈ R. Use the integral x cos x dx to show that
(2n)! 0
n=0

+∞
X (−1)n
= cos 1 + sin 1 − 1 .
(2n)!(2n + 2)
n=0

IX. Approximate the numerical value of ln(1.6) using the second degree Taylor polynomial for
f (x) = ln(3x − 2) about 1.

ic s
at
h em
at
M
of
e
ut
s tit
In
UP
Bibliography

[1] H. Anton, I. Bivens, S. Davis, Calculus: Early Transcendentals, John Wiley and
Sons, 7th Edition, 2002.

[2] L. Leithold, The Calculus 7, Harpercollins College Div., 7th edition, December 1995.

[3] J. Stewart, The Calculus: Early Transcendentals, Brooks/Cole, 6th Edition, 2008.
Institute of Mathematics, College of Science
University of the Philippines Diliman

S-ar putea să vă placă și